Vous êtes sur la page 1sur 174

Question 1

1 out of 1 points

In the accompanying diagram of a bacterial cell envelope, identify the


overall structure marked as 2.

Answers:
Peptidoglycan layer
Outer membrane
Mesosomes
Inner cell membrane
Capsule

Response Structure 2 is the cell wall composed of peptidoglycans and is


Feedback also known as the peptidoglycan layer. The cell wall is involved
:
in maintaining the overall shape of a bacterial cell. Most Grampositive bacteria have a relatively thick cell wall composed
largely of peptidoglycan. In contrast, the peptidoglycan layer in

the Gram-negative bacteria is relatively thin. This structure


provides the basis for the gram reaction.
Outside the peptidoglycan layer in the Gram-negative envelope
is a structure called the outer membrane. This membrane
structure is anchored non-covalently to lipoprotein molecules
(Braun's lipoprotein), which in turn are covalently linked to the
peptidoglycan. The lipopolysaccharides of the Gram-negative
cell envelope form part of the outer leaflet of the outer
membrane structure.
The capsule is a region of material that extends beyond the
outer membrane of some bacteria. Bacterial capsule contributes
directly to the pathogenicity or virulence of an organism. They
confer resistance to phagocytosis; therefore, they provide
protection to the bacterial cell against host defenses to
invasion.
Encapsulated species are found among both Gram-positive and
Gram-negative bacteria.
The inner cell membrane is a lipid bi-layer that surrounds and
contains the cytosol.
Mesosomes are invaginations of the plasma membrane/inner
cell membrane. These structures are much more prominent in
Gram-positive than in Gram-negative organisms.
References:
1. Jennifer K. Wagner, Sima Setayeshgar, Laura A. Sharon, James
P. Reilly, and Yves V. Brun; A nutrient uptake role for bacterial
cell envelope extensions; PNAS; 2006, 103(31): 11772-11777.
2. Ghuysen J-M, Hakenbeck R: Bacterial cell wall. Elsevier, 1994
Question 2
1 out of 1 points

A 24-year-old Indian man presents to his physician with 4-day history of


progressive weakness in the legs. The patient states that he is now also
having weakness in his arms. History is significant for gastroenteritis 1 week
ago. A spinal tap is performed and a CSF analysis is pending. Which of the
following is true regarding the organism that is associated with this disease?
Answers:

Gram positive anaerobic rod

Gram positive aerobic rod


Gram negative rod
Gram negative diplococci
Gram positive cocci

Respons
e
Feedbac
k:

The correct answer choice is Gram negative rod. This patient


presents with ascending paralysis and a notable history of
gastroenteritis. This patient most likely has Guillain Barre
Syndrome (GBS). Campylobacter jejuni is the most commonly
associated organism with the preceding gastroenteritis that is
associated with GBS. C. jejuni is a Gram negative curved rod.
Clostridium tetani is an example of a Gram positive anaerobic
rod.
Corynebacterium diptheria is an example of a Gram positive
aerobic rod.
Neisseria are Gram negative diplococci.
Staphylococcus are Gram positive cocci.
References:
www.ninds.nih.gov/disorders/gbs/gbs.htm
Campylobacter jejuni. FDA website.Available at
www.cfsan.fda.gov/~mow/chap4.html, last accessed Nov 13,
2007
Denyer SP, Hugo WB, Hodges NA, et al.Hugo and Russels
Pharmacological microbiology. Blackwell publishing.2004

Question 3
0 out of 1 points

Case:
A 25-year-old woman presents with severe orbital pain. She claims to have
difficulty smelling and tasting. This symptom has been present for 2 weeks.
An examination of her throat is significant for the presence of sinus
drainage. There is no redness of the throat area. The ears are clear. The
patient has a body temperature of 38.6 C and an elevated WBC with a left

shift. Sinus material was obtained for culture and gram stain. The gram stain
was positive for numerous neutrophils with no organisms seen. On day 2,
the culture was positive for 4+ gram negative rods that were sensitive to
kanamycin and colistin, resistant to vancomycin, and catalase negative.
Morphologically, the organism Gram stained as long and pointy rods.
Question:
What organism is causing the infection?
Answers:

Clostridium perfringens
Bacteroides fragilis

Fusobacterium spp
Veillonella spp
Propionibacterium acnes

Respons
e
Feedback
:

Fusobacterium spp. are long, thin anaerobic Gram-negative rods


with pointed ends. They are usually arranged in end-to-end
pairs. The organism is indole variable, catalase negative, grows
in 20% bile, and is sensitive to kanamycin and colistin but
resistant to vancomycin. The organism is a cause of brain
abscesses, sinusitis, odontogenic infections, pleuropulmonary
infections, bacteremia, and endocarditis. Most Fusobacterium
spp. are sensitive to penicillin.
Bacteroides fragilis are anaerobic, Gram-negative bacilli that
grow in 20% bile; they are resistant to kanamycin, vancomycin,
and colistin and are catalase and indole positive. It is the most
common anaerobic isolate from intra-abdominal abscesses. The
organism is part of the normal gastrointestinal flora, though
usually only 0.5% of the colonic microflora. Anaerobic infections
are usually polymicrobial with a mixture of aerobes and
anaerobes. Metronidazole, clindamycin, and chloramphenicol
are some of the antibiotics used to treat Bacteroides fragilis.
Clostridium perfringens is an anaerobic, Gram-positive, sporeforming bacillus. The organism is nonmotile and has a distinctive
"box car" appearance on gram stain. It produces oval central
spores, but they are rarely seen in clinical or cultures. All types
produce lecithinase, which is lytic to neutrophils. Because of the

action of lecithinase on neutrophils, gram stains of blister


aspirates and other drainage characteristically contain grampositive rods but little or no neutrophils. Clostridium
perfringens is the main causative agent of gas gangrene, and
diagnosis is made by microbiological findings, clinical findings,
and the demonstration of myonecrosis at surgery. Surgical,
antibiotic, and hyperbaric oxygen treatments are used in
treating gas gangrene.
Veillonella spp. is anaerobic Gram-negative cocci that are found
as part of the normal flora in the oral cavity, upper respiratory
tract, intestine, and vagina. They have a generally low virulence
but can cause significantly serious infections given the right
circumstances, such as steroid therapy, previous injury, foreign
bodies, and IV drug abuse. They are generally susceptible to
most of the antibiotics used to treat other anaerobic infections.
is a Gram-positive anaerobic cocci. It is part of the normal skin
flora. Though the organism is usually a contaminant of blood
and wound cultures, it can be the cause of serious lifethreatening infections in the proper conditions. These infections
include brain abscesses, osteomyelitis, subdural empyema,
cerebral shunt infection, infective endocarditis, and parotid and
dental infections.Propionibacterium acnes
Question 4
1 out of 1 points

The most common evidence for the existence of a genetic predisposition to


autoimmune disease is in the higher incidence of the disease in monozygotic
twins, with a lower but still increased incidence in dizygotic twins and family
members when compared with an unrelated population. Although familial
tendencies occur, the pattern of inheritance is complex and indicates that
inheritance is
Answers:

Latent
Anergic
Polygenic
Apoptotic
Tolerant

Respons
e
Feedbac
k:

Gene families like HLA are associated with autoimmune


diseases, but inheritance of the predisposition to autoimmune
disease is polygenic. This means that no individual gene is
sufficient to elicit the disease and many genes may interact with
one another. This interaction may result in a synergistic effect,
an additive one, or an inhibitory consequence if a gene codes for
a product that cancels out the effect of other genes. Many
autoimmune diseases are genetically heterogeneous, that is, the
same clinical disease may result from the combined effect of
different genes. The fact that predisposing genes are usually
common in the general population adds to the difficulty in
studying genetic susceptibility. Eliciting the disease may require
specific environmental triggers.
Some viruses can enter a state known as latency in which the
virus is not being replicated. In the latent state, the virus does
not cause disease, but because there are no viral peptides to
signal its presence, the virus is not eliminated. Latent infections
can be reactivated.
Anergy is a state of unstable metabolic arrest affecting
lymphocytes that can lead to apoptosis. It occurs when a
lymphocyte receives an antigenic signal without the normal
necessary co-stimulatory second signal.
Apoptosis represents physiological as opposed to pathological
(necrotic) cell death - it is the cellular equivalent of suicide.
Apoptotic cells are recognized by phagocytes and removed.
is a state in which no specific antibody is made to an antigen,
while normal immune responsiveness to other antigens remains.
T cells from a tolerant animal will suppress the immune
response to the antigen if the cells are adoptively transferred to
another animal; the cells responsible for this transfer
are suppressor T cells. Tolerance can be induced any time in the
ontogeny of the cell, provided it expresses a receptor for the
antigen.Tolerance

Question 5
1 out of 1 points

Case:
A 6-month-old boy has a peculiar facies. He also has developmental
abnormalities to his face and ears. He is constantly getting sick with viral and
fungal infections and is beginning to have bizarre spasms in his hands that
frighten his mother. She takes him to his pediatrician, who orders some tests.
His lab results are as follows:
TEST

RESULTS

RBC

4.8 x 106/l

platelets

250,000/mm3

WBC

5.4 x 103/mm3

neutrophils

68

lymphocytes

19

monocytes

eosinophils

basophils

sodium

140 mEq/L

glucose (fasting)

91 mg/dL

calcium

7.9 mg/dL

Serum electrophoresis of the immunoglobulins was within normal limits.


Question:
This patient is found to have hypoplasia of his parathyroid gland. What other
organ will be hypoplastic?

Answers:
Thymus
Thyroid
Adrenal cortex
Adrenal medulla
Pituitary

Respons
e
Feedbac
k:

This patient has hypocalcemia as a consequence of DiGeorge


syndrome. DiGeorge syndrome consists of thymic hypoplasia and
hypoparathyroidism. There will be a T-cell deficiency because of the
thymic hypoplasia. A lack of T-cells would lead to viral and fungal
infections. The lymphocyte count can be within normal limits;
however, all the lymphocytes present would be B-lymphocytes. The
parathyroid glands are also affected in DiGeorge syndrome and are
hypoplastic (or sometimes completely absent). Because of the
hypoplastic parathyroid glands, hypoparathyroidism is present. The
hypoparathyroidism will result in hypocalcemia, which in turn can
result in tetany. Developmental defects are often seen in children
with DiGeorge syndrome.
The thyroid, adrenal cortex, adrenal medulla, and pituitary would
not be affected in DiGeorge syndrome.
References:
M. Louise Markert, Marilyn J. Alexieff, Jie Li, Marcella Sarzotti, Daniel
A. Ozaki, Blythe H. Devlin, Debra A. Sedlak, Gregory D. Sempowski,
Laura P. Hale, Henry E. Rice, Samuel M. Mahaffey, and Michael A.
Skinner. Postnatal thymus transplantation with immunosuppression
as treatment for DiGeorge syndrome. Blood. 15 October 2004, Vol.
104, No. 8, pp. 2574-2581. Prepublished online as a Blood First
Edition Paper on April 20, 2004; DOI 10.1182/blood-2003-08-2984.
Applying Public Health Strategies to Primary Immunodeficiency
Diseases: A Potential Approach to Genetic Disorders. MMWR. Jan
16, 2004. 53(RR01);1-29.

Question 6
1 out of 1 points

Many of the microbes that are most important in human infectious diseases
multiply in the extracellular spaces of the body. Most intracellular pathogens
must spread by moving from cell to cell through the extracellular fluids.
Viruses and intracellular bacteria, which need to enter cells in order to grow,
spread from cell to cell by binding to specific molecules on their target cell
surface. Antibodies that bind to the pathogen can prevent this. What is this
process, which is also important in protection from bacterial toxins, called?
Answers:

Opsonization
Neutralization
Phagocytosis
Pinocytosis
Endocytosis

Respons
e
Feedbac
k:

The simplest and most direct way in which antibodies can


protect from pathogens or their toxic products is by binding to
them and thereby blocking their access to cells that they may
infect or destroy. This is known as neutralization and is
important for protection against bacterial toxins and against
pathogens such as viruses, which can thus be prevented from
entering cells and replicating.
The coating of pathogens and foreign particles by
antibodies is opsonization. Macrophages and other phagocytic
cells have receptors for the Fc portion of the antibody molecule.
These receptors provide a "handle" for phagocytes to bind
coated particles.
Phagocytosis is the ingestion of these coated pathogens or
particles by macrophages or other phagocytes, followed by
internalization and destruction by intracellular digestion.
Pinocytosis is the simple cellular uptake of fluid and solutes. It is
a much simpler process than phagocytosis.
Some enveloped viruses enter the cell
using endocytosis, the invagination of the plasma
membrane into smaller virus-sized depressions coated on the
cytoplasmic side with a cellular protein known as clathrin. These
detach from the plasma membrane and become vesicles free in

the cell's cytoplasm.


Question 7
1 out of 1 points

Which of the following media are considered to be most useful for the
cultivation of fastidious pathogens?
Answers:
Chocolate agar
MacConkey agar
Colistin-nalidixic acid agar
Eosin methylene blue agar

Response Chocolate agar is a highly enriched medium composed of


Feedback heated sheep blood in which the red cells have lysed due to
:
gentle heating, spilling their nutritious contents into the basal
medium. This agar is normally incubated in a carbon dioxide
atmosphere to further enhance the growth of fastidious
pathogens. MacConkey agar and Eosin methylene blue are
designed only for the isolation of enteric, gram-negative rods.
Colistin-nalidixic acid agar is an inhibitory medium that prevents
the growth of gram-negative organisms and allows grampositive organisms to grow. Thioglycollate is a broth medium
designed to encourage the growth of anaerobic organisms.

Question 8
1 out of 1 points

Future vaccines will be formulated using highly purified, recombinant, or


synthetic peptide antigens, and vaccination with DNA has certain
advantages. These vaccines may be safer and may elicit specific immune
responses, but these vaccines are usually less immunogenic than whole
killed or live vaccines. Even some inactivated vaccines require enhancement
to be sufficiently immunogenic. Materials that increase the immune

responses to a given antigen without being related to it are called


Answers:

Immunotoxins
Streptolysins
Immunophilins
Endotoxins
Adjuvants

Response Even conjugate vaccines are not usually strongly immunogenic


Feedback on their own; most require the addition of adjuvants, which are
:
substances that enhance the immunogenicity of antigens. Some
commonly used adjuvants are listed.
Oil Adjuvants
Freund's complete adjuvant and incomplete adjuvant
Mineral Salts
Aluminum phosphate or hydroxide, calcium alginate
Double-stranded nucleic acids
Poly (IC), Poly (AU)
Other substances
Saponin, levamisole
In humans, the most widely used adjuvants are aluminum
compounds (alums). These form a precipitate with protein
antigens and result in slow release of the antigen. Alums are
present in vaccines such as tetanus toxoid and diphtheria
toxoid.Biodegradable polymers can be used as delayed-release
capsules, dissolving weeks after injection to release a booster
dose of antigen. A glycoside adjuvant called Quil A extracted
from the bark of an Amazonian oak tree readily forms micelles
and can be used to make an immunostimulatory complex
containing immunizing peptides with an built-in adjuvant.
Linking a tumor-specific or tumor-selective monoclonal antibody
to a toxin is a way to create an immunotoxin, a reagent that can
by used to destroy all or part of a tumor. When the labeled
antibody is internalized, the toxin is cleaved from the antibody
in the cell's endocytic compartment, allowing the toxin to
penetrate and kill the cell.
Certain groups of streptococci secrete a variety of streptolysins
or hemolysins that lyse red blood cells and are much more

active weight for weight than hemolysins such as bile salts or


saponin, but which have a more important toxic action on
polymorphs and macrophages. Various hemolysins are released
also by pathogenic staphylococci, and these can kill phagocytes.
The immunophilins are a family of intracellular proteins that
bind to certain compounds forming complexes that interfere
with signaling pathways important for the clonal expansion of
lymphocytes.
are part of the outer membrane of Gram-negative bacteria.
Some of the diseases in which endotoxins may play a part are
typhoid fever, tularemia, plague and brucellosis, and a variety of
hospital-acquired infections caused by opportunistic Gramnegative pathogens. Endotoxin is a complex lipopolysaccharide
that displays a large array of biological effects.Endotoxins
Question 9
1 out of 1 points

The antigen-specific arm of cell-mediated immunity consists of the T


lymphocytes. Unlike B lymphocytes, which produce soluble antibody that
circulates to bind its specific antigens, each T cell, bearing many identical
antigen receptors called T cell receptors, circulates directly to the site of
antigen and performs its function when interacting with antigen. Mammalian
T cell maturation and differentiation take place in the
Answers:

Spleen
Peyer's patches
Thymus
Bone marrow
Primary lymphoid organ

Response The thymus provides a specialized and architecturally organized


Feedback microenvironment for the development of mature T cells.
:
Precursors of T cells migrate from the bone marrow and mature
in the thymus, passing through a complex multistep process. T
cell development is accompanied by extensive cell death,
reflecting the intense selection of T cells and the elimination of

those with inappropriate receptor specificities. Most steps in T


cell differentiation occur in the cortex of the thymus. The thymic
medulla contains mainly mature T cells.
The spleen is a fist-sized organ just behind the stomach that
collects antigen from the blood. It also collects and disposes of
senescent red blood cells. The spleen is the major organ in the
body in which antibodies are synthesized and from which they
are released into the circulation.
Named after a Swiss anatomist, Peyer's patches are oval
elevated areas of lymphoid tissue on the mucosa of the small
intestine, composed of many lymphoid follicles closely packed
together. Antigen is collected by specialized epithelial cells
called M cells. The lymphocytes form a follicle consisting of a
large central dome of B lymphocytes surrounded by a smaller
number of T lymphocytes.
In embryonic life, B lymphocytes differentiate from
hematopoietic stem cells in the fetal liver. After birth, and for
the life of the individual, this function moves to the bone
marrow, a structure that is considered to be a primary lymphoid
organ. The mature B cells are transported to the secondary
lymphoid organs, where they encounter and respond to foreign
antigens.
There are two major primary lymphoid organs, one in which the
T cells develop and the other in which the B cells develop. The
thymus where T cells mature and the bone marrow where B
cells develop are the primary lymphoid organs.
Question 10
1 out of 1 points

Tolerance to self is a normal state that is maintained primarily by clonal


deletion of developing T and B cells and clonal deletion or inactivation of
mature peripheral T and B cells. When the state of self-tolerance is
disrupted, autoimmunity can result. The immune response to natural
infection occasionally elicits destructive T cells that cause autoimmune
disease by cross-reacting with peptides of self-antigens. In this case,
microbial antigens stimulate antibody responses that react not only with the
pathogen but also with host antigens. This phenomenon is called
Answers:

Anergy

Apoptosis
Molecular pathogenesis
Molecular mimicry
Antigenic variation

Response In molecular mimicry, an antigen of a microorganism resembles


Feedback a self-molecule and can induce a cross-reactive autoimmune
:
response. Several autoimmune diseases occur after specific
infections and are thought to be triggered by the infection.
Some examples are shown in the table.
Infection Consequence
Group A Streptococcus Rheumatic fever
Chlamydia trachomatis Reiter's syndrome
Salmonella typhimurium Shigella flexneri Campylobacter jejuni
Reactive arthritis
Borrelia burgdorferi Chronic arthritis in Lyme disease
Disease susceptibility in these cases is determined by MHC
genotype. Other environmental initiators of autoimmunity can
act like infections by causing tissue damage, such as sunlight in
lupus erythematosus.
Anergy is a state of unstable metabolic arrest affecting
lymphocytes that can lead to apoptosis. It occurs when a
lymphocyte receives an antigenic signal without the normally
necessary co-stimulatory second signal.
Apoptosis represents physiological as opposed to pathological
(necrotic) cell death - it is the cellular equivalent of suicide.
Apoptotic cells are recognized by phagocytes and removed.
Most infectious agents show a significant degree of host
specificity, causing disease only in one or a few related species.
The molecular mechanism of host specificity is an area of
research interest known as molecular pathogenesis.
One way in which an infectious agent can evade immune
surveillance is by altering its antigens; this is particularly
important for extracellular pathogens, against which the
principal defense is the production of antibody against their
surface structures.Antigenic variation can allow pathogens to
escape form the immune response.

Question 11
1 out of 1 points

Case:
A 50-year-old industrial worker engaged in handling animal products
develops a painless papule on his right forearm where he had a minor
abrasion 5 days earlier. The papule became a vesicle after 48 hours and
ruptured, leaving an ulcer with a black necrotic area in the center and
surrounding edema. Case: Gram stain of smear from the skin lesion showed
Gram-positive bacilli (1-1.5 x 3-5 microns) that appeared encapsulated.
Aerobic culture done from the lesion on 5% sheep blood agar yielded pure
growth of non-hemolytic colonies 2-5 mm diameter with wavy border and
ground glass appearance after overnight incubation. The isolate was nonmotile, produced spores, and was sensitive to penicillin.
Question:
What is the Gram-positive bacillus most likely to be?"
Answers:

Corynebacterium ulcerans
Bacillus cereus
Clostridium perfringens
Bacillus anthracis
Erysipelothrix rhusiopathiae

Response Bacillus anthracis is the bacterium isolated. The skin lesion is


Feedback likely to be acquired as a result of infection from animal
:
products. The Gram-positive large encapsulated bacilli
demonstrated in a smear from the lesion, its rapid aerobic
growth, the characteristic appearance of the colonies, the
absence of motility, and its sensitivity to penicillin support its
presumptive identification as Bacillus anthracis. The organism is
an aerobic spore-bearer that produces a poly-gamma-d-glutamic
acid capsule in vivo. The capsule is anti-phagocytic and protects
the bacteria from complement-mediated lysis. Spores are highly
resistant and are not produced in living tissue. Carbon dioxide
levels within the body inhibit sporulation. Exposure to free
oxygen is necessary for formation of spores. Absence of motility
and absence of hemolysis in culture help to differentiate the

organism from other bacillus species often encountered as


contaminants in culture.
Bacillus anthracis causes 3 clinical forms of anthrax: cutaneous,
inhalational, and gastrointestinal.
Cutaneous form is the most common. The development and
appearance of the skin lesion in the patient is characteristic of
cutaneous anthrax (malignant pustule).
Inhalational anthrax is the most severe form.
Complications like septicemia and meningitis can occur. Tests
used to confirm identification of the organism in reference
laboratories are lysis by gamma phage, capsule-specific
staining, and polysaccharide cell wall antigen staining.
Polymerase Chain Reaction (PCR) for Bacillus
anthracis chromosomal markers and Enzyme-Linked
Immunosorbent Assay (ELISA) for antigen detection are rapid
tests for the direct detection in clinical samples. Penicillin,
ciprofloxacin, or doxycycline is usually recommended for
treatment of anthrax. Virulence factors of bacillus anthracis are
anthrax toxin and capsule. These are encoded on 2 large
plasmids pXO1 and pXO2, respectively. Strains lacking either of
these plasmids have greatly reduced virulence. Anthrax toxin
consists of 3 protein components termed protective antigen
(PA), edema factor (EF), and lethal factor (LF). Separately, none
of these proteins is toxic. They act synergistically in binary
combinations. PA is the cell-binding moiety, and it mediates
entry of EF and LF into the host cell. EF in combination with PA
forms the Edema toxin. LF with PA forms the Lethal toxin, which
is a major virulence factor of anthrax bacillus. Anthrax vaccine
for humans contains the protective antigen. B.
anthracisis considered an important agent of bioterrorism
because of its virulence factors and physical properties.
Corynebacterium ulcerans is a Gram-positive non-motile
bacillus; it is club-shaped, non-sporing, and non-capsulated. It
possesses metachromatic granules, which give the rod a beaded
appearance. It produces slightly hemolytic colonies on blood
agar. The organism is closely related to Corynebacterium
diphtheriae and can carry the same bacteriophage that codes
for diphtheria toxin; therefore, it can cause disease similar to
clinical diphtheria. The organism is primarily a veterinary
pathogen, and it causes mastitis in cattle and infections in other
domestic animals. Sporadic cases of diphtheria caused by C.
ulcerans have been reported in humans who come in contact

with farm animals or consume un-pasteurized dairy products.


Human to human transmission is not documented. Cutaneous
infection is extremely rare.
Bacillus cereus is an aerobic spore-forming bacillus related to B.
anthracis, and it is a ubiquitous organism. It is motile and
produces hemolysis on sheep blood agar. It produces beta
lactamase and is therefore resistant to penicillin and other beta
lactam antibiotics. It causes food poisoning: emetic type
following consumption of preformed toxin in rice dishes and
diarrheal type following consumption of contaminated meat or
dairy products. B. cereus can also cause severe diseases when
combined with predisposing factors in the patient, such as drug
addiction, immunosuppression, and prosthetic implants.
Endophthalmitis, osteomyelitis, pneumonia, and endocarditis
are examples of severe infections.
Clostridium perfringens (clostridium welchii) is a large Grampositive, nonmotile, anaerobic spore-bearing bacillus. It causes
invasive infections, myonecrosis, and gas gangrene, often
following wound contamination. Several toxins with lethal,
necrotizing, and hemolytic properties contribute to
pathogenesis. Some strains produce a powerful enterotoxin and
cause food poisoning.
Erysipelothrix rhusiopathiae are Gram-positive, non-motile, nonsporing, non-capsulated slender rods arranged singly, in short
chains, or forming long filaments. The organism produces
transparent glistening colonies on blood agar. E. rhusiopathiae is
an occupational pathogen that typically causes non-pyogenic
cutaneous lesions known as erysipeloid. The organism can be
cultured from biopsy of inflamed edge of lesion. It is highly
susceptible to penicillin and intrinsically resistant to
vancomycin. Erysipeloid lesions follow inoculation of the
organism at the site of abrasion, and they usually occur on the
hand or fingers of persons handling animals, fish, or animal
products. The lesions are painful, edematous, and
erythematous; they often involve the local lymph glands. E.
rhusiopathiae can cause a wide spectrum of diseases, namely
polyarthralgia, septic arthritis, peritonitis, bacteremia, and
endocarditis.

References:
1. N Engl J Med-29-NOV-2001; 345 (22) : 1621- 26.
2. Centers for Disease Control and Prevention; Basic laboratory

Protocols for the Presumptive Identification of bacillus


anthracis . OCT-2001.
3. Jawetz, Melnick and Adelberg's Medical microbiology ,
23rd ed;2004 Mcgraw Hill, 202-18.
4. Journal of Clinical Microbiology ; SEPT-2005 : 43 (9) : 43774381.
5. Applied Environ Microbiol 2001 ; 67 : 3720 - 7.
6. Methods Mol Biol 2004 ; 268 : 199 - 205.
Question 12
1 out of 1 points

Patients with hyposplenism or those who have undergone splenectomy are


at risk for overwhelming pneumococcal infection. This is because an immune
cell population is important in the production of protective antibody against
pneumococcal cell wall and other carbohydrate antigens. This cell
population consists of what splenic element?

Answers:

Dendritic cells
Basophils
Eosinophils
B lymphocytes
T lymphocytes

Response Splenic B lymphocytes are critical in secreting IgG2 against the


Feedback cell wall antigens and other carbohydrate antigens associated
:
with pneumococcal species. The polysaccharide capsule of the
pneumococcus is associated with resistance to phagocytosis
and with virulence. Pathogenic bacteria with similar
polysaccharide capsules include Hemophilus
influenzae and Klebsiella pneumoniae. Pneumococcal vaccine
contains polysaccharide antigens of the most common
serotypes. Immunization offers some protection in these
patients and should be performed.

Dendritic cells are cells with a distinctive branched morphology


found in the T cell areas of lymphoid tissue. These cells trap
antigen in the periphery and migrate to lymphoid tissues, where
they present antigens to T cells. Because of this, and because of
their ability to deliver activating signals, dendritic cells are
known as professional antigen-presenting cells.
Basophils are the smallest circulating granulocytes with the
least known function. They arise in the bone marrow, and
following maturation and differentiation, are released into the
blood circulation. Like mast cells, basophils have receptors on
their cell membranes that bind with high affinity to the Fc
portion of IgE.
Eosinophils are terminally differentiated, end-stage leukocytes
that reside primarily in submucosal tissue and are recruited to
sites of specific immune responses. They are granulocytes that
can express Fc receptors when activated. Once these receptors
are triggered they degranulate, releasing leukotrienes that
cause muscle contraction.
There are two main classes of T lymphocytes. One class
differentiates on activation into cytotoxic T cells that kill cells
infected with viruses, and the second class of T cells
differentiates into cells that activate other cells such as B cells
and macrophages. These T lymphocytes may provide a helper
function, but the immunoglobulin critical for defense against
pneumococcal species is secreted by B-lymphocytes.
Question 13
1 out of 1 points

The main cause of bacterial pharyngitis is


Answers:

Streptococcus anginosus
Streptococcus bovis
Streptococcus pyogenes
Streptococcus agalactiae
Streptococcus pneumoniae

Question 14

1 out of 1 points

Case:
The patient is a 90-year-old patient in an extended care facility who has an
elevated temperature of 39.9C. The patient charts show spiking
temperatures began the previous night. There was a nursing note of the
patient complaining of lower chest pain upon respiration. Blood cultures and
sputum cultures were collected. A CBC was significant for a white cell count
of 28,000/ul. Later that afternoon the patient became hypotensive. He was
transferred to a hospital were he was intubated because of septic shock,
acute renal failure, and respiratory failure. Chest radiographs were obtained
and were significant for the presence of a left lower lobe infiltrate with
pleural effusion. The sputum Gram stain was significant (refer to the image).
Sputum and blood cultures were positive for beta hemolytic gram-positive
cocci that were catalase and coagulase positive.
Question:
This patient has sepsis and what infection?

Answers:

Mycobacterium tuberculosis
Francisella tularensis
Pneumocystis carinii

Aspergillus
Haemophilus influenzae
Staphylococcus aureus

Respons
e
Feedbac
k:

Pneumocystis carinii is clssified as a fungus based on aspects of


it biochemistry, but its drug sensitivity is that of a protozoan.
The organism is capable of causing a fulminant pneumonia in
patients that are immunocompromised or debilitated. In the
normal individual the natural host defenses are intact and the
organism cannot cause disease. The organism is widely spread
and up to 90% of children develop antibody against it by the age
of 4 years. The pneumonia that the organism produces is rapid
in its development and is life threatening. Since the organism is
not sensitive to the normal prophylactic regimen provided to
patients suspected of bacterial pneumonia, and the aggressive
nature of a Pneumocystis infection, the definitive identification is
needed to successfully treat the infection. The drug of choice is
trimethoprim/sulfamethoxazole. Diagnosis is made by
demonstrating Pneumocystis organisms in pulmonary material.
Mycobacterium tuberculosis is an acid-fast bacilli and is the
causative agent of tuberculosis (also caused by Mycobacterium
bovis and africanum). The organism is found worldwide and
found especially in developing countries. It is spread from
person to person via droplet nuclei (caused by coughing or
sneezing). The disease usually has an insidious onset with the
first symptoms being night sweats, low-grade fever, and chronic
weight loss. The progression of the disease leads to a chronic
cough that produces sputum and hemoptysis. The disease is
primarily pulmonary, but any organ can be attacked. The
organism is best detected in sputum, which can be cultured and
stained with an acid-fast stain. Mycobacteria will stain intensely
pink while non-acid-fast organisms will stain blue. Treatment is
with two or more of the following drugs for up to 6 months or
more (isoniazid, rifampin, pyrazinamide, ethambutol, and
streptomycin).
Francisella tularensis is coccoid to ellipsoid in shape and very
small. It is gram-negative and non-motile, appearing singly. The
organism is a strict aerobe. The organism is the causative agent
of tularemia (rabbit fever, deerfly fever, market men's fever).
The onset of the disease is sudden and includes flu-like
symptoms as well as characteristic ulcer with regional

lymphadenopathy. The source of infection is the exposure to


infected wild and domestic animals or their products,
contaminated water, ingestion or inhalation, and blood sucking
arthropods. The organism is highly infectious and is the third
most commonly reported laboratory associated infection. Direct
isolation is best done by culturing the ulcer, pharynx,
conjunctiva, lymph node biopsy, and gastric washing.
Streptomycin is the drug of choice.
Aspergillus is a fungus and contains the species fumigatus,
flavus, niger, and terreus. It is a fungus that can cause allergic
conditions, fungus ball formation in the lungs, and
mycotoxicoses, tissue invasion. Infections generally are
opportunist in nature with the immunocompromised and
debilitated most at risk. The infection can either present as an
acute or chronic infection. Inhalation is considered the primary
route of infection. Invasive disease is found primarily in those
cases where an underlying primary disorder compromises cellmediated immunity. Amphotericin B is the drug of choice.
Haemophilus influenza is a gram-negative cocco-bacillus. It is a
major cause of meningitis, mostly in young children. When it
occurs in adults, it is usually due to an underlying condition such
as paranasal sinusitis, remote head trauma, or otitis. The
organism is also a major cause of pneumonia and can be found
concomitantly with Streptococcus pneumoniae. The organism
will not grow on blood agar (but grows well on chocolate agar)
and requires the presence of growth factors (NAD and hemin) for
growth. Ampicillin is the drug of choice.
is a gram-positive cocci that is catalase positive and coagulase
positive. On Gram stain it morphologically appears in clusters.
The organism is usually beta hemolytic on 5% sheep blood agar
and the colonies have a yellowish color. Staphylococcus
aureus is the most commonly isolated clinically significant grampositive organism. It has a host of virulent factors that make it a
potent pathogen. Along with pneumonia it can cause urinary
tract infections; wound infections; deep tissue abscesses;
septicemia; gastroenteritis; etc. The pneumonia is generally
described as a cavitating pneumonia with
hemoptysis.Staphylococcus aureus
Question 15
1 out of 1 points

All adaptive immune responses are mediated by lymphocytes and


macrophages. In order to study these cells it is necessary to separate them
so that their behavior can be analyzed in vitro. A useful technique for
isolating these immune cells from blood is density centrifugation over a step
gradient consisting of a mixture of a carbohydrate polymer and the dense
iodine-containing compound metrizamide. This yields a population of cells
called
Answers:

Activated cytotoxic cells


Peripheral blood mononuclear cells
Long-term memory lymphocytes
Small resting B cells
Fully differentiated tissue macrophages

Response In this simple procedure, peripheral blood is diluted with buffer


Feedback and carefully layered over the gradient and
:
centrifuged. Peripheral blood mononuclear cells are separated in
a layer at the interface that is gently removed. These cells
are depleted of red blood cells and most polymorphonuclear
leukocytes or granulocytes, and consist mainly of lymphocytes
and monocytes. To obtain a more purified population of
lymphocytes or monocytes, further purification based on
characteristics such as cell-surface receptors is required.
Question 16
1 out of 1 points

The botulinum toxin acts by:


Answer Blocking the release of acetylcholine at the synapse and thereby
s:
producing paralysis
Affecting cGMP activity
Blocking protein synthesis
Activating the complement cascade

Response Botulinum toxin produced by Clostridium botulinum is a


Feedback neurotoxin that blocks the release of acetylcholine at the
:
synapset, producing paralysis. The genes for this toxin are
encoded by a temperate bacteriophage. It is composed of 2
polypeptide subunits held together by disulfide bonds and one
of the subunits binds to a receptor on the neuron.
Question 17
1 out of 1 points

A 10-year-old boy was diagnosed with a zoonotic infection endemic in the


U.S. that is recognized as a frequent cause of benign lymphadenopathy in
children and is caused by a small pleomorphic Gram-negative bacillus.
Following a scratch from his pet cat, the boy had developed an
erythematous papule on his arm and a few weeks later, regional
lymphadenitis. The etiological agent of the boy's infection is most likely to
belong to what genera?

Answers:

Brucella
Bartonella
Bordetella
Burkholderia
Borrelia

Response The boy's history of 'cat scratch', typical presentation with


Feedback regional lymphadenitis, and the features of the zoonotic
:
infection described are suggestive of Cat Scratch Disease (CSD).
The infection is endemic in the U.S. and about 24,000 people
have CSD annually, 80% of whom are children. The main agent
of CSD is Bartonella henselae. Bartonella clarridgeiae and
Bartonella quintana have also been reported to be associated
with a few cases of CSD. Thus, the correct answer is Bartonella.
B.henselae is a zoonotic pathogen capable of causing a broad
range of clinical manifestations in humans. CSD is usually a self-

limited infection in immunocompetent children. Other


manifestations of B.henselae infection include bacillary
angiomatosis-peliosis, bacteraemia, encephalitis, endocarditis,
and neuroretinitis. Bacillary angiomatosis-peliosis is
characterized by the formation of vasoproliferative tumors
resulting from bacterial colonization and activation of human
endothelial cells. Ability to induce endothelial cell proliferation is
a common characteristic of Bartonella species pathogenic to
humans. Systemic diseases mostly occur in
immunocompromised persons.
Cats are the main reservoirs of B.henselae though other animals
like dogs and rabbits can occasionally serve as sources of
infection. Arthropods, mainly fleas, transmit the infection among
cats.
B.henselae is a very fastidious bacterium. Once isolated from
the lymph nodes of CSD patients, it needs prolonged incubation
in enriched blood media in the presence of carbon dioxide, so
detection is often unsuccessful. Histological examination is
helpful and the bacteria can be detected in lymph node biopsy
smears and sections by Warthin-Starring silver impregnation
staining. Serological tests like indirect immunofluorscence
assays are very useful for diagnosis except in early stages of the
disease. Detection of B.henselae DNA in the lymph node tissue
samples by polymerase chain reaction (PCR) assay has high
specificity and is found valuable in the diagnosis of CSD.
Carrying out laboratory tests to exclude other causes of
lymphadenitis is important.
All other genera listed also contain species that cause zoonotic
infections. All species of brucellae, Bordetella bronchiseptica,
Burkholderia pseudomallei, and the spirochete Borrelia
burgdorferi that causes Lyme disease are zoonotic pathogens.
Cats are not common reservoirs of these bacteria and mode of
transmission and typical manifestations also differ.
References:
Jawetz, Melnick & Adelberg's Medical Microbiology -23rd Ed
2004; Copyright McGraw-Hill Co: 314-316.
MMWR Morb Mortal Wkly Rep-MAR-2002; 51(10): 212-214.
Journal of Clinical Microbiology-AUG-2005; 43(8): 3800-3806.
Emerg Infect Dis-MAR-2006; 12(3): 389-394.

BMJ-MAY 18-2002; 324: 1199-1200.


American Family Physician-NOV 1-2007; 76(7): 1314-1322.
Question 18
1 out of 1 points

The laboratory you are working in is studying the regulation of


immunoglobulin synthesis. You know that in order to produce the complete
immunoglobulin protein, changes must occur at the DNA level. The antigen
binding site of the immunoglobulin molecule is contained in the

Answers:

Constant region
Variable region
Fc fragment
Heavy chain only
Hinge region

Response Antibody molecules are composed of 4 polypeptide chains, 2


Feedback heavy chains, and 2 light chains. These chains are covalently
:
linked by disulfide bonding. Each heavy chain is associated with
a light chain with the association occurring at the aminoterminal end of the heavy chain. Each light and heavy chain
consist of constant and variable regions. Each chain type
contains 3 hypervariable regions that are required to form the
antigen-binding site. The carboxy-terminal 3/4 of the heavy
chain and the carboxy-terminal half of the light chain are
homologous and called the constant region. The Fc fragment is
generated in vitro by cleavage of the antibody molecule with
the enzyme papain. This fragment does not bind antigen. The
hinge region is the domain between the amino and carboxyterminal regions and is the "branch point" in the "Y" model of
antibody structure.
Question 19
1 out of 1 points

Case:
A 25-year-old woman residing in a rural area of Alaska delivered her first
born at home (home birth). Within 48 hours, the new born baby boy
developed lethargy, poor feeding, and respiratory distress and was brought
to the pediatric emergency room of the nearest hospital. He was admitted to
the neonatal ICU with a clinical diagnosis of neonatal sepsis. Samples were
collected for laboratory investigations including blood for culture. The infant
was placed on empirical antibiotic therapy and other supportive measures.
After overnight incubation at 37 degrees C, gram-stain of blood culture
showed gram-positive cocci in chains. Subculture on 5% sheep blood agar
medium grew grayish white colonies with narrow zone of beta hemolysis.
Further studies of the isolate showed that it was catalase and oxidase
negative, hydrolized hippurate, and was bile-aesculin negative. It was
resistant to bacitracin and gave a positive CAMP test (Christie Atkins-MunchPetersen test). Presumptive identification of the organism could be made
based on these tests. Serological test was used to confirm identification.
Question:
What is the most likely bacterium causing early onset neonatal infection?"
Answers:

Staphylococcus aureus
Streptococcus pyogenes
Streptococcus pneumoniae
Staphylococcus epidermidis
Streptococcus agalactiae

Respons
e
Feedbac
k:

Streptococcus agalactiae is a Group B Streptococcus (GBS). GBS


is a major cause of neonatal sepsis in the US and other western
countries. The isolate from the blood culture has the typical
characteristics of this bacterium. Camp factor produced by the
organism is responsible for giving a positive CAMP test. CAMP
test demonstrates synergistic hemolytic activity between GBS
and staphylococcus aureus. An accentuated zone of hemolysis is
seen when GBS is inoculated perpendicular to a streak of
staphylococcus aureus on blood agar. S.aureus produces beta
lysin, which partially lyses sheep RBCs and the camp factor
enhances this hemolytic activity.

In developing countries with limited availability of streptococcal


grouping sera, a positive CAMP test and positive hippurate
hydrolysis are often used for presumptive identification of GBS
and to differentiate it from streptococcus pyogenes (Group A
Streptococcus), the most common hemolytic streptococcus
associated with human infections.
Streptococcus pyogenes is usually bacitracin sensitive, and gives
a negative CAMP test. Streptococcus
pneumoniae (Pneumococcus) can be differentiated from GBS by
its typical lanceolate diplococcal appearance and production of
alpha hemolytic colonies on blood agar. Staphylococcus
aureus and staphylococcus epidermidis appear as grape-like
clusters of gram-positive cocci and these are catalase positive.
The newborn infant referred to is having early onset neonatal
sepsis caused by GBS. (EOGBS disease). Neonates are
considered to have EOGBS disease when clinical illness develops
within 7 days after birth accompanied by GBS isolation from
normally sterile sites. Late onset infections occur in infants in
their 2nd-12th week of life and are often obtained from the
environment. In EOGBS disease, infection of the fetus results
from ascending spread of GBS from the vagina of the mother,
who is an asymptomatic carrier colonized with the organism.
Gastro-intestinal tract serves as the natural reservoir of GBS and
is the likely source of colonization. Infection may cause
septicemia, meningitis, or pneumonia in the new born. Preterm
infants are more susceptible to GBS infections. Conditions in the
mother such as preterm labor, premature rupture of membranes,
prolonged labor after the rupture of membranes, and GBS
bacteriuria during pregnancy favor fetal infection. Infants can get
infected during passage through the birth canal.
GBS colonization of the vagina is often transient. It can be also
intermittent or persistent. Collection of cultures between 35 and
37 weeks of gestation by swabbing both lower vagina and
rectum for detection of colonization by GBS in pregnant women
and subsequent intrapartum antibiotic prophylaxis of colonized
mothers is recommended by the Centers for Disease Control and
Prevention (CDC) for prevention of EOGBS infections.
Intrapartum antibiotic prophylaxis, administered at least 4 hours
before delivery, is found to reduce the incidence of EOGBS
disease considerably. Intravenous penicillin G is the antibiotic of
choice because of its narrow spectrum of activity. Ampicillin is
considered as an alternative. In individuals at high risk of
anaphylaxis to penicillin, erythromycin or clindamycin can be
used. Cefazolin is used in women having only mild allergic

reaction to penicillin. Vancomycin is reserved for those colonized


with GBS strains resistant to erythromycin and clindamycin and
with history of anaphylactic reaction to penicillin.
The conventional culture method to detect GBS colonization
requires 36 to 72 hours. To overcome this slow turnaround time,
antigen detection assays and nucleic acid detection assays have
been explored. A PCR-based assay targeting cfb gene which
codes for the CAMP factor is reportedly useful for detecting GBS
DNA directly in vagino-rectal specimens. This test has the
advantage of a turn around time of 1-2 hours, but the
disadvantage of its requirement for costly equipments. A PCR
assay targeting scpB gene, which encodes C5a peptidase,is also
reported to have given accurate results. A PCR-based assay for
detection of GBS in neonatal blood samples is found to show
good sensitivity and specificity.
Though increased use of intrapartum antibiotic prophylaxis has
caused significant decline in EOGBS disease, GBS still remains a
leading cause of morbidity and mortality in the newborn.
Pntrapartum antibiotic prophylaxis also causes concern over the
possibility of emergence of antibiotic resistance, and neonatal
infections by resistant gram-negative bacteria. The need for a
vaccine is felt and there is ongoing research in this field.
GBS strains associated with human infections possess a
polysaccharide capsule which confers virulence to the organism.
9 capsular types have been identified, antibodies to which confer
type specific protection. Glycoconjugate vaccines against all 9
GBS serotypes have been shown to be immunogenic in animal
models. Conjugate vaccines have been prepared using the
capsular polysaccharides of GBS serotypes common in the US
and have been shown to be highly immunogenic in healthy
adults. It is reported that such vaccines are likely to be effective
in preventing GBS disease in the at-risk populations including
neonates born to colonized mothers, peripartum women,
diabetics, and the elderly with underlying illnesses. Surface
proteins of GBS are found to elicit protective immunity and
therefore thought to be of relevance in vaccine production.
Attempt to develop a universal protein-based candidate vaccine
with a broader protective activity against various circulating
serotypes of GBS is also in progress.
Staphylococcus aureusis a Gram-positive coccus that occurs in
clusters. It produces large opaque beta hemolytic colonies with
golden yellow pigment. It is catalase positive, coagulase positive,
and can be easily differentiated from streptococci.

Staphylococcal infections are among the most common bacterial


infections ranging from trivial superficial infections to lifethreatening conditions such as septicemia and toxic shock
syndrome. It is one of the common agents of nosocomial
infections. Staphylococcus aureus may cause early onset
neonatal infections, though rarely.
Streptococcus pyogenes (Group A streptococcus) possesses a
group-specific cell wall carbohydrate different from that of GBS.
It can be distinguished from GBS by serological method in
addition to the presumptive methods such as bacitracin
sensitivity and CAMP test. The organism produces various types
of enzymes and toxins, which contribute to its virulence and are
responsible for a wide variety of suppurative infections of skin
and subcutaneous tissue, septicemia and pyemia, and the
nonsuppurative sequelae, acute rheumatic fever, and acute
glomerulonephritis. Early onset neonatal infections associated
with Streptococcus pyogenes are exteremely rare.
Streptococcus pneumoniaeis a small Gram-positive lanceolate
diplococcus, often capsulated, the capsule being one of the
major virulence factors of the bacterium.
It produces alpha hemolytic colonies on blood agar medium.
Pneumococci colonize the human nasopharynx and can cause
middle ear infections, sinusitis, conjunctivitis, and pneumonia.
Meningitis is the most serious form of pneumococcal infections.
The bacterium is also associated with acute exacerbations of
chronic bronchitis. Isolation of streptococcus pneumoniae from
early onset neonatal infections is very rare.
Staphylococcus epidermidisis a coagulase-negative
staphylococcus and is invariably present on the normal human
skin. It is an opportunistic pathogen, causing infections when the
host defenses are breached. It is implicated in causing
nosocomial or late-onset neonatal sepsis. It is known to form
biofilms on prosthetic devices and survive within them,
inaccessible to antibiotics and host defenses. It can cause
various infections, including cystitis, bacteremia, and
endocarditis. Hospital strains are usually multidrug resistant.

References:
Matern Child Health- JAN-2007; 11(1): 91-95.
CDC: MMWR.Recommend Rep - AUG 16 - 2002; 51 (RR-11): 1-22
CDC on Group B streptococcus: available at

www.cdc.gov/groupbstrep/lab/lab_pers_photos.htm - last
modified Oct 4, 2006.
Am Fam Physician-MAR-2005; 71 (5): 903-910.
Pediatrics- May-2005; 115 (5): 1240-1246.
Journal of Clinical Microbiology-MAR-2006; 44 (3); 725-728.
Science-JULY- 2005; 309 (5731): 148-150.
Expert Opin Biol Ther -SEPT-2003; 3(6): 975-984.
Expert Rev Vaccines-APR-2005; 4(2): 207-218.
Jawetz, Melnick & Adelberg's Medical Microbiology 23rd Ed 2004;
copyright McGraw-Hill co. pp 223- 242.
Pediatrics-OCT-2002; 110(4); 690-695
Diagn Microbiol and Infect Dis-SEPT-2004;50(1):7-13.
Clin Microbiol Rev-JAN-2005;18(1):102-127.
Question 20
1 out of 1 points

One of the major functions of the immune system is the production of


soluble proteins that circulate freely and exhibit properties that contribute
specifically to immunity and protection against foreign material. These
soluble proteins are the immunoglobulins. In the immunoglobulins there is
an area composed of a short segment of amino acids that is found between
the CH1 and CH2 regions of the heavy chains. This segment is made up
predominantly of cysteine and proline residues and is called the

Answers:

Hinge region
J chain
Light chain
Agretope
Domain

Response The hinge region of the immunoglobulin heavy chain provides


Feedback an important structural characteristic of these proteins. It
:
permits flexibility between the two Fab arms of the Y-shaped
antibody molecule. It allows the two Fab arms to open and close
to accommodate binding to two epitopes, separated by a fixed
distance, as might be found on the surface of a bacterium.
Additionally, since this stretch of amino acids is open and as
accessible as any other nonfolded peptide, it can be cleaved by
proteases, such as papain, to generate the Fab and Fc
fragments.
A 15 kDa polypeptide chain called the J chain promotes
polymerization of IgA and IgM molecules by linking to the
cysteines of the tailpiece, which is found only in the secreted
forms of the and chains. In the case of IgA, polymerization is
required for transport through epithelia. The polymerization of
these immunoglobulins is thought to be important in antibody
binding to repetitive epitopes.
There are two types of light chain found in antibodies, which are
called lambda () and kappa () chains. No functional difference
has been found between antibodies having lambda or kappa
light chains, and either type of light chain may be found in
antibodies of any of the five major classes.
The size of an epitope that binds to a specific T cell is about
eight to 15 amino acid residues and it contains an area that
binds to the MHC proteins of the antigen-presenting cell, an area
called the agretope.
Each heavy or light chain is constructed of domains of
approximately 110 amino acids held together in a loop by a
disulfide bond between two cysteine residues in the chain. The
domains have the same basic structure and many areas of
similarity in their amino acid sequences. Light chains have two
domains each, and heavy chains have four or five domains,
separated by a short unfolded stretch.
Question 21
1 out of 1 points

Case:
A 6-month-old boy has a peculiar facies. He also has developmental
abnormalities to his face and ears. He is constantly getting sick with viral and
fungal infections and is beginning to have bizarre spasms in his hands that
frighten his mother. She takes him to his pediatrician, who orders some tests.
His lab results are as follows:
TEST

RESULTS

RBC

4.8 x 106/l

platelets

250,000/mm3

WBC

5.4 x 103/mm3

neutrophils

68

lymphocytes

19

monocytes

eosinophils

basophils

sodium

140 mEq/L

glucose (fasting)

91 mg/dL

calcium

7.9 mg/dL

Serum electrophoresis of the immunoglobulins was within normal limits.


Question:
What is the most likely condition?
Answers:

DiGeorge syndrome

X-linked agammaglobulinemia
Severe combined immunodeficiency
Wiskott-Aldrich syndrome
Bruton's agammaglobulinemia

Respons
e
Feedbac
k:

This patient has hypocalcemia as a consequence of DiGeorge


syndrome. DiGeorge syndrome consists of thymic hypoplasia and
hypoparathyroidism. There will be a T-cell deficiency because of the
thymic hypoplasia. A lack of T-cells would lead to viral and fungal
infections. The lymphocyte count can be within normal limits;
however, all the lymphocytes present would be B-lymphocytes. The
parathyroid glands are also affected in DiGeorge syndrome and are
hypoplastic (or sometimes completely absent). Because of the
hypoplastic parathyroid glands, hypoparathyroidism is present. The
hypoparathyroidism will result in hypocalcemia, which in turn can
result in tetany. Developmental defects are often seen in children
with DiGeorge syndrome.
X-linked agammaglobulinemia is also called Bruton's
agammaglobulinemia. X-linked agammaglobulinemia would present
with a low level of immunoglobulins.
Severe combined immunodeficiency is characterized by a defect in
both cellular and humoral immunity. There would be a decrease in
the lymphocyte count.
Wiskott-Aldrich syndrome is an immune deficiency disorder.
Characteristics of Wiskott-Aldrich syndrome include eczema,
infections, small platelet size, and a low platelet count. It is X-linked
recessive.
References:
M. Louise Markert, Marilyn J. Alexieff, Jie Li, Marcella Sarzotti, Daniel
A. Ozaki, Blythe H. Devlin, Debra A. Sedlak, Gregory D. Sempowski,
Laura P. Hale, Henry E. Rice, Samuel M. Mahaffey, and Michael A.
Skinner. Postnatal thymus transplantation with immunosuppression
as treatment for DiGeorge syndrome. Blood. 15 October 2004, Vol.
104, No. 8, pp. 2574-2581. Prepublished online as a Blood First
Edition Paper on April 20, 2004; DOI 10.1182/blood-2003-08-2984.
Applying Public Health Strategies to Primary Immunodeficiency
Diseases: A Potential Approach to Genetic Disorders. MMWR. Jan 16,

2004. 53(RR01);1-29.
Yinzhu Jin, Cinzia Mazza, Jacinda R. Christie, Silvia Giliani, Maurilia
Fiorini, Patrizia Mella, Francesca Gandellini, Donn M. Stewart, Qili
Zhu, David L. Nelson, Luigi D. Notarangelo, and Hans D. Ochs.
Mutations of the Wiskott-Aldrich Syndrome Protein (WASP):
hotspots, effect on transcription, and translation and
phenotype/genotype correlation. Blood. 15 December 2004, Vol.
104, No. 13, pp. 4010-4019.Prepublished online as a Blood First
Edition Paper on July 29, 2004; DOI 10.1182/blood-2003-05-1592.
Question 22
1 out of 1 points

Case:
As a child, whenever you caught a cold, your Grandma prepared chicken
soup, which always made you feel better. Now, as a scientist, you decided to
find the evidence that chicken soup really helps in the treatment of the cold.
To investigate, you prepared chicken soup the same way your Grandma did
and add it to neutrophils from the peripheral blood of healthy volunteers.
Using Zymosan-activated serum as a positive chemoattractant, you find that
chicken soup is active only in the presence of chemoattractant.
Question:
You come to what conclusion about what chicken soup inhibits?
Answers:

Immunoglobulines
Cell motility
CD4
Necrotaxis
Hemokinesis

Response A cold is generally the result of transient infections of the


Feedback mucosa of the upper respiratory tract and is associated with the
:
recruitment of neutrophils to the epithelial surface of the
airways and the neutrophil chemotaxis. As chicken soup is
active only in the presence of chemoattractant, you conclude
that in vitro chicken soup attenuates inflammatory response by

inhibiting neutrophil migration.


Chicken soup in this experiment acts by inhibiting chemotaxis
using nonspecific chemoattractant. Immunoglobilines are
specific for the each type of foreign substance. Therefore,
chicken soup does not act on immunoglobulines.

You are investigating the role of neutrophils. CD4 is


a glycoprotein expressed on the surface of T helper
cells, regulatory T cells, monocytes, macrophages, and dendritic
cells that assists the T cell receptor with an antigen presenting
cell and is not involved in chemotaxis.
Necrotaxis embodies a special type of negative or positive
response to chemoattractants released from apoptotic or
necrotic cells. This is not a case in your experiment; you use live
neutrophils and Zymosan-activated serum.
Hemokinesis is the flow of blood in the body.
References:
Miceli MC, Parnes JR (1993). "Role of CD4 and CD8 in T cell
activation and differentiation".Adv. Immunol. 53: 59-122
Rennard BO, Ertl RF, Gossman GL, Robbins RA, Rennard SI,
MD, Chicken Soup Inhibits Neutrophil Chemotaxis In
Vitro. Chest. 2000 Oct;118(4):1150-7
Hu CL, Barnes FS. (1970). A theory of necrotaxis. Biophys
J. 10 (10): 958-69
Question 23
1 out of 1 points

Both B cell and T cell receptors for antigen react with structures called
epitopes or antigenic determinants. An epitope is a discrete region on or
within an antigen molecule - a particular molecular shape expressed at a
certain region of the molecule. An epitope composed of a single segment of
polypeptide chain is termed
Answers:

Conformational

Combinatorial
Polymorphic
Continuous
Immunodominant

Respons
e
Feedbac
k:

A continuous or linear epitope is one which represents a linear


peptide segment of amino acid or carbohydrate sequence. An
epitope is linear if the target of the immune response is
apparent in the series of adjacent amino acids without any
requirement for secondary or tertiary structure or folding. T cell
clones respond to peptide fragments (usually eight to 20 amino
acids in length) digested from native protein - these are linear
epitopes.
A conformational epitope is one that represents a
discrete folded region of polypeptide or polysaccharide that
forms due to the secondary and/or tertiary folding
characteristics of the protein or polysaccharide. A
conformational epitope could consist of short stretches of amino
acids from two or more different regions of a polypeptide, or
from two or more polypeptides, which are in direct apposition in
the folded protein.
A combinatorial epitope reflects protein subunit: subunit
interaction. Epitopes can form on the individual subunits and at
the site of subunit interaction that are unique for the complex.
When used in reference to proteins, polymorphic means
variation at a single genetic locus and its product within a
species. An example would be polymorphic variants of MHC
molecules.
An antigenic molecule may have several different
epitopes. Immunodominant epitopes are those that consistently
stimulate an antibody response in all individuals. This concept is
critical in vaccine design, where recipients of the vaccine
material will respond to immunodominant epitopes to the
exclusion of others.

Question 24
1 out of 1 points

A concerned mom brings her 2-year-old son into ER and tells the pediatrician
that the child has recurrent infections with bacteria, viruses, and yeast. His
blood is drawn and severe depletion of B cells and T cells are noted. What
defect is most likely responsible for this child's condition?
Answers:

A lack of Btk
A lack of RAG
A lack of NADPH oxidase
A lack of myeloperoxidase
A convoluted thymus

Response In the case of RAG deficiency, the T cells and B cells cannot
Feedback properly recombine their variable regions to produce distinct
:
idiotypes and therefore the T cell receptors and
immunoglobulins are not synthesized. These molecules are
necessary to give a survival signal to developing T cells and B
cells, so the lack of these proteins depletes one of B cells and T
cells. A lack of T cells and B cells leads to increased infections
with different types of pathogens (this is an example of severe
combined immunodeficiency (SCID)).
No Btk means B cells cannot signal during development. This
would result in severe B cells depletion, but not a severe T cell
depletion.
Lack of NADPH oxidase causes Chronic Granulomatous Disease,
which is a phagocyte deficiency. It will not result in severe
depletion of T cells and B cells.
Myeloperoxidase deficiency (a lack of hypochloride production)
is also a phagocyte deficiency, rather than a lymphocyte
deficiency.
A deficiency in thymic development will severely reduce the
number of T cells in the periphery and the ability of B cells to be
activated. The actual number of B cells, however, will not
change as significantly.
References:
Murphy Kenneth, Paul Travers, Mark Walport. Janeway's

Immunobiology, 7th ed., Taylor & Francis, Inc. 2007


David C. Rudolph's Pediatrics. 21st Edition. McGraw-Hill
Professional. 2003;795.
Schuetz C, Huck K, Gudowius S, et al. An immunodeficiency
disease with RAG mutations and granulomas. N Engl J Med.
2008 May 8;358(19):2030-8.
Question 25
1 out of 1 points

An unresponsive 20-year-old female college student is brought to the


emergency room. Respiratory rate is 50/min and systolic blood pressure is
60mm Hg. She is intubated and transferred to the ICU. The patient's
roommate reports that the patient had been feeling ill for about 2 days and
had been experiencing diarrhea. The physical examination reveals lungs that
are clear; she has an elevated body temperature of 39.8C, and there is a
desquamation of the skin around the fingernails and underlying toes. A
malodorous tampon is removed. Culture and Gram stain is obtained from
exudate material from the cervix. Blood is drawn for a CBC and for
coagulation studies. The CBC is significant for an elevated white blood cell
count with a pronounced left shift and a platelet count that is significantly
decreased. Prothrombin time is abnormally high. The Gram stain is
significant for Gram-positive cocci in clusters. The culture is positive for
Gram-positive cocci that are beta-hemolytic on 5% sheep blood agar,
catalase-positive, and coagulase-positive. Refer to the image. This patient is
diagnosed as having menstrual toxic shock syndrome due to what infection?

Answers:

Streptococcus agalactiae
Clostridium perfringens
Bacillus cereus
Streptococcus intermedius group (millerii)
Streptococcus pyogenes
Staphylococcus aureus

Respons
e
Feedback
:

Staphylococcus aureus is a Gram-positive cocci that is catalasepositive and coagulase-positive. On Gram stain, it
morphologically appears in clusters. The organism is usually
beta-hemolytic on 5% sheep blood agar, and the colonies have
a yellowish color. Staphylococcus aureus can cause a variety of
infections. It can cause toxic shock syndrome (TSS), which can
be broken down to menstrual TSS and non-menstrual TSS. TSS
was first described in 1978 in children and later in 1980 and
1981 when the introduction of a new hyperabsorbable tampon
produced an epidemic of Staphylococcus aureus TSS. Since
then, product changes have reduced the incidence. Patients with
menses-associated TSS are usually young women between the
ages of 15-25 years of age who are using tampons during their
menstrual period. Symptoms include severe myalgias, fever,
vomiting, and diarrhea. The patient usually presents as listless

and confused and may rapidly develop severe hypotension with


hypovolemic shock. Conjunctival inflammation and a "sunburn
rash" can develop within a few hours. Fluid replacement and IV
oxacillin or nafcillin are used to treat the patient.
Streptococcus intermedius group (millerii) is a Gram-positive
cocci that is catalase-negative, esculin-positive, hippuratenegative, Voges-Proskauer positive, and produces a caramel-like
odor. Colonies often require CO2. When they do grow, they are
sometimes mistaken for anaerobic streptococci. Colonies can be
beta, alpha, or gamma hemolytic on blood agar media.
Lancefield antigenic serologic reactions are of little value in
identifying these organisms because any one of a number of
antigens may be exhibited. The organism has been documented
as causing oral infections, bacteremia and endocarditis, thoracic
infections, central nervous system infections, and abdominal
infections. In drug addicts, the organism can cause a
subcutaneous abscess/cellulitis that can be mistaken as
necrotizing fasciitis.
Streptococcus agalactiae is a Gram-positive cocci that is
catalase-negative, beta-hemolytic on blood agar media, and has
a Lancefield grouping of B. The organism produces CAMP factor,
a thermostable extracellular protein that results in synergistic
hemolysin on sheep blood agar with the B-lysin
of Staphylococcus aureus (seen in 98-100% of group B
streptococcal isolates). The organism can cause a variety of
infections, such as neonatal meningitis, pneumonia,
osteomyelitis, endocarditis, and skin/soft tissue infections.
Skin/soft tissue infections are usually found in patients with
predisposing factors like diabetes mellitus. Necrotizing fasciitis
can occur due to Streptococcus agalactiae. Ampicillin and
penicillin G are the drugs of choice. Vancomycin is an alternative
for penicillin allergic adults.
Clostridium perfringens is a Gram-positive anaerobic rod that
produces spores, has a boxcar appearance on Gram stain,
produces lecithinase, and produces a double zone of hemolysis
on blood agar. The organism can cause a variety of infections,
such as bacteremia, pleuropulmonary infections, biliary tract
infections, intra-abdominal infections, food poisoning, clostridial
myonecrosis (gas gangrene), and various other soft tissue
infections (crepitant cellulitis, suppurative myositis, and
common soft tissue infections in which they are often a
component of polymicrobial flora).
Bacillus cereus is an aerobic Gram-positive rod that forms

spores, and is beta-hemolytic on blood agar. The organism can


cause a variety of infections, such as bacteremia, endocarditis,
food poisoning, ophthalmitis, osteomyelitis, soft tissue
infections, and necrotizing fasciitis. Necrotizing fasciitis has
been reported in a leukemic patient and a patient with sickle cell
disease. As in all cases of necrotizing fasciitis, antibiotic therapy
alone is not sufficient in treating the infection, and multiple
surgical debridement is usually required. Amputation is
sometimes necessary.
is a Gram-positive cocci that is catalase-negative, betahemolytic on blood agar, and had a Lancefield grouping of A.
The organism can cause an infection called necrotizing fasciitis
(streptococcal gangrene). The infection involves the deeper
tissues and fascia. Extensive and rapid spreading of necrosis, as
well as gangrene of the skin and underlying structures, is
characteristic. The infection usually begins at a site of trivial or
inapparent trauma. Prompt antibiotic treatment and surgical
measures to contain the infection (amputation/debridement) is
necessary to prevent life threatening complications, such as
toxic shock. Mortality rates are generally high.Streptococcus
pyogenes
Question 26
1 out of 1 points

Which of the following best defines an antigen?


Answer Any protein or carbohydrate that enters the body
s:
Any substance that is able to combine with T or B cells
A substance that binds to a specific site on the Fab fragments of
antibodies
"A simple substance that is not individually immunogenic, but can
react with a specific antibody"
Any substance that can evoke an immune response

Response The following are definitions for some basic terms.

Feedback Immunogen: Any substance that can induce an immune


:
response. Most immunogens are proteins, polypeptides, and
polysaccharides.
Antigen: Any substance that can bind to a specific antibody, but
not necessarily elicit antibody production by itself. From this,
and previous definition, it follows that all immunogens are
antigens, but not all antigens are immunogens. Proteins,
peptides, polysaccharides, nucleic acids, and lipids can be
antigens. Protein and peptide antigens can be processed by socalled antigen presenting cells and presented to T cells. The
binding of antigens with the surface immunoglobulins on B cells,
or processed antigens with T cell receptors, may lead to a
positive or negative activation of those lymphocytes.
Epitope: Or antigenic determinant, is the part of an antigen that
binds the antibody molecule or T cell receptor.
Haptens: Amall substances that are not immunogenic
themselves, but can react with antibodies of corresponding
specificity. Antibodies can be generated against haptens, if
immunization is performed with a hapten attached to a carrier
protein.
Adjuvant: A substance that increases immunogenicity of an
immunogen, when mixed with it.
Question 27
1 out of 1 points

The heat-labile enterotoxin produced by E.coli causes diarrhea by


Answers: Stimulating adenylate cyclase activity in cells in the small
intestine
Stimulating guanylate cyclase
Blocking protein synthesis
Blocking release of acetylcholine

Response The heat-labile enterotoxin produced by E.coli causes diarrhea


Feedback by stimulating adenylate cyclase activity in cells in the small
:
intestine. The resulting increase in the concentration of cyclic

adenosine monophosphate (cAMP) causes excretion of the


chloride ion, inhibition of sodium ion absorption, and significant
fluid and electrolyte loss into the lumen of the gut.
Question 28
1 out of 1 points

Case:
A 72-year-old man was brought to the hospital with complaints of sudden
onset of rigors and high fever (38.8C), vomiting, and confused mental
status. The patient had history of fever and diarrhea about 3 weeks earlier
from which he recovered without any specific treatment. While being
examined, he developed generalized seizures. Lumbar puncture was done.
CSF was turbid. CSF and blood samples were sent for microbiological
investigations and the patient was placed on intravenous ceftriaxone as
empirical therapy.
The CSF total WBC count was 1000/cmm. Differential count showed 63%
mononuclear cells with predominance of lymphocytes. In Gram stain done
with centrifuged deposit of CSF, very few gram-positive coccobacilli could be
observed. Kinyoun's and modified acid fast stains showed non-acid fast
coccobacilli. After 48 hours of incubation, minute round smooth translucent
beta hemolytic colonies of short gram positive bacilli appeared on aerobic
sheep blood agar. The bacteria showed tumbling motility when grown in
broth at 25C and were non-motile at 37C. The isolate was catalase positive
and fermented sugars with production of acid only. Antibiotic susceptibility
pattern showed sensitivity to penicillin, trimethoprim/sulphamethoxazole,
vancomycin, and aminoglycosides as well as resistance to quinolones and
third generation cephalosporins. Gram-positive bacillus with same
characteristics was grown from blood culture samples. Based on the
microbiology report, the patient was placed on a prolonged course of
combination therapy with ampicillin and gentamicin.
Question:
Which of the following organisms caused meningitis in the patient?
Answers:

Flavobacterium meningosepticum
Listeria monocytogenes
Corynebacterium jeikeium
Rhodococcus equi

Erysipelothrix rhusiopathiae

Respons
e
Feedbac
k:

Listeria monocytogenes: The mononuclear predominance and


presence of gram-positive coccobacilli in the CSF are in favor of
L.monocytogenes infection. Morphology in gram-stained smear
and cultural characteristics of the isolate from the clinical
samples are typical of Listeria though they are not sufficient to
differentiate from corynebacterium,which the organism closely
resembles. The tumbling motility shown by the bacterium when
grown at 25C and its absence at 37C is highly characteristic of
L.monocytogenes and is an important differentiating feature.
Antibiotic sensitivity pattern is also in accordance with that of
L.monocytogenes, which is known to be resistant to third
generation cephalosporins.
Listeria monocytogenes causes severe invasive infections such
as meningitis and meningoencephalitis in neonates, pregnant
women, elderly, transplant patients, and others with impaired
cell-mediated immunity. Listeria can also cause infections in
normal individuals. Infection is usually food-borne and is
transmitted by consumption of unpasteurized dairy products and
uncooked meat and vegetables. Outbreaks of listeriosis
associated with contaminated food have occurred, though rarely.
The bacterium possesses ability to cross intestinal, placental,
and blood-brain barriers leading to gastroenteritis, maternofetal
infections, meningitis, and meningoencephalitis. For invasive
illness, the incubation period is 20-30 days and for
gastroenteritis about 20 hours. Important sero varieties, which
are associated with human infections, are 1a, 1b, and 4b. Some
of the factors, which attribute to the virulence of Listeria
monocytogenes, are its cell wall surface proteins called
Internalins, hemolysin (listeriolysin O), phospholipase enzymes,
and ActA protein that are responsible for the actin-based
intracellular motility of the bacterium. Prolonged treatment (2-3
weeks) with high doses of antibiotics is required for invasive
infections because of the intracellular habitat of the bacterium.
Combined therapy with ampicillin and gentamicin is considered
as the best option. Trimethoprim-sulfamethoxazole is used in
those who cannot take pencillin.
Flavobacterium meningosepticum (Chrysobacterium
meningosepticum) is a non-motile gram-negative bacillus, which
often produces yellow-pigmented colonies on blood agar
medium. It can cause meningitis and sepsis, especially in the
newborns and in adults with underlying illness. It is often

associated with nosocomial infections such as ventilatorassociated pneumonia. It may also cause endocarditis. In the
hospital environment the bacterium exists in water systems on
wet surfaces of medical tools and equipment. Vancomycin has
been successfully used for treatment of meningitis caused by
this gram-negative bacterium. Newer quinolones garenoxacin,
gatifloxacin, and levofloxacin have been reported to be most
effective antibiotics for treating infections by Chrysobacterium
meningosepticum. The bacterium is known to be resistant to
aminoglycosides, tetracyclines, erythromycin, clindamyciin,
teicoplanin, and most beta-lactams including carbapenems.
Chromosomal genes that encode for production of metallobetalactamases have been identified in this bacterium.
Dissemination of these genes to other gram-negative bacteria of
greater clinical significance can pose a problem.
Corynebacterium jeikeium may resemble listeria in Gram stain,
but is non-motile and non-hemolytic. It can cause severe
infections such as endocarditis in immunocompromised
individuals and is known to be resistant to most of the
commonly used antimicrobials. Vancomycin is the antibiotic of
choice for treatment. Macrolide resistance of C.jeikeium has
been observed as part of its multi-drug resistance. Genetic
studies have shown that macrolide resistance is conferred by
erm (X) cj gene, which is integrated within the chromosome of
this bacterium. It is suggested that C.jeikeium may be an
important reservoir of drug resistant genes and presence of this
organism in the hospital environment can be a cause for
concern.
Rhodococcus equi (Corynebacterium equi) is a gram-positive
pleomorphic coccobacillus. It is weakly acid fast when stained
with modified acid-fast stain. On blood agar it produces nonhemolytic, mucoid colonies with salmon pink pigmentation. It is
primarily a pathogen of animals and produces severe
pneumonia in foals. It is an intracellular pathogen and in humans
an important cause of AIDS-associated pneumonia. R.equi is also
known to cause infections in persons having other conditions
with impaired cell-mediated immunity such as organ
transplantation and malignancy. In immunocompetent persons,
R.equi infection is extremely rare. Virulence associated antigens
(VapA and VapB) and virulence plasmids of R.equi have been
discovered. The majority of R.equi isolates from patients with
AIDS have been shown to harbor virulence plasmids and either
of the 2 virulence associated antigens. R.equi is resistant to
penicillins and cephalosporins and sensitive to vancomycin,

rifampin, aminoglycosides, erythromycin, and imipenem.


Erysipelothrix rhusiopathiae is a gram-positive bacillus that may
appear singly, in short chains, or as long branching filaments. It
grows slowly on blood agar medium producing small transparent
glistening colonies either non-hemolytic or alpha hemolytic. It is
catalase negative, highly susceptible to Penicillin G, and
intrinsically resistant to vancomycin. It causes erysipelas in
swine. In humans, the bacterium produces erysipeloid, a
localized cutaneous lesion. The lesion follows inoculation of the
organisms at the site of a cut or abrasion and usually occurs on
the hands or fingers of persons handling animals, fish, or animal
products. It is painful, edematous, and erythematous. Often the
local lymph glands are involved. Generalized cutaneous lesion
may occur due to spread from the initial site of infection. In rare
cases, the bacterium causes septicemia associated with
endocarditis. Penicillin G is the drug of choice for severe
infections. Mortality rate of septicemia is high because
vancomycin is often used as empirical therapy; therefore, early
diagnosis is important. Polymerase chain reaction (PCR) assays
have been recently developed for detection and identification of
E.rhusiopathiae.
References:
1. Jawetz, Melnick, & Adelberg's Medical Microbiology. 23rd Ed;
Copyright 2004: The McGraw-Hill Companies, 217-218.
2. Clincal Microbiology Reviews -JULY-2001; 14 (3): 584- 640.
3. FEMS Immunol Med Microbiol-APRIL-2003; 35 (3): 173-175,
203-205.
4. Journal of Clinical Pathology- APRIL - 2005; 58 (4): 423-425.
5. Journal of Clinical Microbiology- JAN- 2004; 42 (1): 445-8.
6. Antimicrob Agents Chemother- JUN - 44 (6) :1448-1452.
7. Journal of Clinical Microbiology-AUG- 2006; 44(8): 3037-9.
8. Disease listingListerosis, CDC, last modified Oct 2005.
9. Antimicrob Agents Chemother-JULY-2001; 45(7): 1982-1989.
10. Methods Mol Biol-2004; 268:199-205
11. Am J Trop Med Hyg-JAN-2002; 66(1): 52-55.
Question 29
1 out of 1 points

A B cell lymphocyte possesses 100 distinct V regions, 6 J regions, and 6


isotypic possibilities to rearrange for its selection of light chain synthesis.
Assuming no role for TdT, how many distinct idiotypes could be produced by
combining this coding sequence with one heavy chain?
Answers:

3600
600
1006
606
36

Response Idiotype of a light chain is determined solely by the variable


Feedback region of the light chain gene, which consists of V and J
:
segments that can be chosen for somatic recombination. The
isotype choices do not contribute to the idiotype of a B cell. So
to determine the total number of possible idiotypes, one must
multiply V by J, i.e. 100 by 6 to get 600.
As isotypes play no role in antigen binding, the 6 isotypic
possibility can be ignored.
The total number of possibilities is a product of the number of V
and the number of J regions, rather than an exponent of these
regions.
The isotype choices need not be added to the product of V and J
to determine the idiotype number.
The answer 36 does not include the V regions in the calculation
and is therefore incorrect.
References:
Murphy Kenneth, Paul Travers, Mark Walport. Janeway's
Immunobiology, 7th ed., Taylor & Francis, Inc. 2007
Wallace DJ, Hahn B, Hahn BH, Dubois EL. Dubois' Lupus
Erythematosus. Lippincott Williams & Wilkins. 2006:410
Bentolila LA, Olson S, Marshall A, et al. Extensive junctional
diversity in Ig light chain genes from early B cell progenitors of

mu MT mice. J Immunol. 1999 Feb 15;162(4):2123-8


Galler GR, Mundt C, Parker M, et al. Surface mu heavy chain
signals down-regulation of the V(D)J-recombinase machinery in
the absence of surrogate light chain components. J Exp Med.
2004 Jun 7;199(11):1523-32. Epub 2004 Jun 1
Question 30
1 out of 1 points

Case:
A 37-year-old Caucasian woman swims regularly for exercise. She swims 100
laps 4 to 6 times per week. She starts to notice severe right ear pain. She
also notes that her right ear is very itchy. She sees her family doctor and
mentions her symptoms. When he goes to insert the otoscope, he gently
pulls on her ear. This causes her quite a bit of pain. He notes an inflamed
external ear canal, but the tympanic membrane is normal.
Question:
What is the most likely pathogen?

Answers:

Pseudomonas aeruginosa
Streptococcus pneumoniae
Haemophilus influenzae
Branhamella catarrhalis
Corynebacterium diphtheriae

Respons
e
Feedbac
k:

This patient has signs and symptoms of otitis externa, specifically


"swimmer's ear." An infection of the external auditory canal,
usually due to bacteria, is called otitis externa. Symptoms include
ear pain and itching. The finding of an inflamed external ear
canal is consistent with otitis externa. The most likely pathogen
for swimmer's ear is Pseudomonas aeruginosa. Pseudomonas

aeruginosa is a Gram-negative aerobic rod. Other pathogens that


can cause otitis externa
include Peptostreptococcus,Staphylococcus aureus, Bacteroides,
Proteus, and fungi.
Streptococcus pneumoniae, Haemophilus influenzae,
and Branhamella catarrhalis are all causes of otitis
media. Corynebacterium diphtheriae does not infect the ear.
The image included here is a photomicrograph of Pseudomonas
aeruginosa. The image is courtesy of the CDC.

References:
Swimmer's Ear. CDC website. Available at:
http://www.cdc.gov/healthyswimming/swimmers_ear.htm.
Accessed August 10, 2015.
Question 31
1 out of 1 points

An effective vaccine must satisfy three important criteria. The vaccine must
be safe, it must produce protective immunity in most recipients, and it must
generate long-lived immunological memory. Tetanus toxoid vaccines often
contain aluminum salts, which act as what?
Answers:

Adjuvants
Immunotoxins
Streptolysins
Immunophilins
Endotoxins

Response Vaccines are not usually strongly immunogenic on their own Feedback most require the addition of adjuvants, substances that enhance
:
the immunogenicity of antigens. It is thought that adjuvants act
on antigen-presenting cells and reflect the importance of these
cells in initiating immune responses. Substances used as

adjuvants or immunostimulants include:


Oil adjuvants
Freund's complete adjuvant, Freund's incomplete adjuvant
Mineral salts
Aluminum phosphate or hydroxide, calcium alginate
Double-stranded nucleic acids
Poly (IC), Poly (AU)
Other substances
Saponin, levamisole
In humans, the most widely used adjuvants are aluminum
compounds (alums). These form a precipitate with protein
antigens and result in slow release of the antigen. Alums are
present in vaccines such as tetanus toxoid and diphtheria
toxoid.Biodegradable polymers can be used as delayed-release
capsules, dissolving weeks after injection to release a booster
dose of antigen. A glycoside adjuvant called Quil A extracted
from the bark of an Amazonian oak tree readily forms micelles
and can be used to make an immunostimulatory complex
containing immunizing peptides with a built-in adjuvant.
Linking a tumor-specific or tumor-selective monoclonal antibody
to a toxin is a way to create an immunotoxin, a reagent that can
by used to destroy all or part of a tumor. When the labeled
antibody is internalized, the toxin is cleaved from the antibody
in the cell's endocytic compartment, allowing the toxin to
penetrate and kill the cell.
Certain groups of streptococci secrete a variety of streptolysins
or hemolysins that lyse red blood cells and are much more
active weight for weight than hemolysins such as bile salts or
saponin, but which have a more important toxic action on
polymorphs and macrophages. Various hemolysins are released
also by pathogenic staphylococci, and these can kill phagocytes.
The immunophilins are a family of intracellular proteins that
bind to certain compounds forming complexes that interfere
with signaling pathways important for the clonal expansion of
lymphocytes.
are part of the outer membrane of Gram-negative bacteria.
Some of the diseases in which endotoxins may play a part are
typhoid fever, tularemia, plague and brucellosis, and a variety of

hospital-acquired infections caused by opportunistic Gramnegative pathogens. Endotoxin is a complex lipopolysaccharide


that displays a large array of biological effects.Endotoxins
Question 32
1 out of 1 points

A forest worker developed an ulcer on his leg at the site of a tick bite within
1 week after exposure. He complained of fever and chills and had painful
inguinal lymphadenopathy. From the ulcer material, a small Gram-negative,
weakly staining pleomorphic coccobacillus (0.2-0.5 microns x 0.7-1 microns)
was isolated on chocolate agar medium at 37 degrees C. The isolate was
identified by conventional methods. It belonged to a bacterial species
recognized as a Category A bioterrorism agent. Which of the following is a
characteristic of this bacterial species?
Answers:

Requires X and V factors for growth


Tick is the only arthropod vector
Type A strains are more virulent for humans than type B
One of the virulence factors is secreted exotoxin
Is resistant to aminoglycosides

Respons
e

The bacterial agents considered Category A bioterrorism agents


are the zoonotic pathogens Bacillus anthracis, Yersinia pestis,

Feedbac
k:

and Francisella tularensis. They cause anthrax, plague, and


tularema, respectively. All possess high infectivity, and the
infective dose is very low. Inhalation of a very small number of
aerosolized organisms by humans can result in fatal pneumonia
and septicemia.
Bacillus anthracis is a large Gram-positive bacterium and can be
excluded by its morphology. Yersinia pestis may morphologically
simulate Francisella tularensis, though its typical morphology
shows bipolar staining. Bubonic plague caused by Y.pestismay
clinically simulate ulceroglandular tularemia caused
by F.tularensis. Y.pestis is transmited by rat flea, not by tick.
Ticks are important vectors of F.tularensis. The patient's illness is
related to tick bite, and the clinical presentation is in accordance
with ulceroglandular tularemia. Therefore, it can be concluded
that the isolate is a strain of Francisella tularensis.
Francisella tularensis is antigenically homogenous. Based on
biochemical activities, 5 subspecies are currently recognized.
Most of the human infections are caused by F.
tularensis subspecies tularensis, also termed type A,
and F.tularensis subspeciesholartica, also termed type B. Type A
strains are more virulent and are associated with more serious
diseases than those produced by type B. Type A strains ferment
glycerol, contain citrulline ureidase, and cause lethal infection in
rabbits. Type B strains lack the biochemical features and are not
lethal for rabbits. Factors associated with pathogenicity
of F.tularensis include its lipopolysaccharide (LPS), surface
capsule, and genes in the Pathogenicity Island. The unique
features that account for increased virulence
of F.tularensis subspecies tularensis (type A) are not well
understood.
Type A infections are predominant in North America and type B
in Europe and Asia.
F.tularensis does not possess the other properties listed. Not only
ticks, but other arthropods like biting flies and mosquitoes, are
also vectors for the organism. The bacterium does not require X
and V factors for growth. No secreted toxin has been detected
for F.tularensis. The organism is susceptible to aminoglycosides.
Gentamicin and streptomycin are drugs of choice for treatment
of tularemia, especially in severe illness. F.tularensis produces
beta-lactamase. Beta-lactam antibiotics, including third
generation cephalosporins, are not effective against the
bacterium. Doxycycline and quinolones are useful in less serious
disease.

Clinical presentation of tularemia depends on the route of entry


of the organisms. In addition to arthropod bite, transmission can
occur also by direct contact with infected animals or animal
tissue, inhalation of aerosols, or ingestion of contaminated food
or water. Ulceroglandular, pulmonary, oculoglandular,
oropharyngeal, gastrointestinal, and typhoidal are different
clinical forms of tularemia. Airborne and waterborne outbreaks
and laboratory infections have been reported.
Molecular assays have been developed for detection
of F.tularensis in clinical specimens and for genotyping of strains.
References:
Jawetz, Melnick & Adelberg's Medical Microbiology-23rd Ed 2004
copyright McGraw-Hill Co: pp286, 287.
Mandell, Douglas and Bennet's Principles and Practice of
Infectious diseases-6th Ed.2005.Copyright Elsevier Inc; Chapter
322:pp 3607-3612.
CDC. MMWR Weekly Report-FEB 25-2005; 54(7): 170-173.
Clin Microbiol Rev-2002; 15(4); 631-646.
Annual Review of Microbiology-2006; 60:167-185.
Antimicrob Agents Chemother-FEB-2006; 50(2): 824.
Kobe J Med Sci-2007; 53(1): 37-42.
Journal of Bacteriology-JUL-2006; 188(14); 5319-5324.
Journal of Clinical Microbiology-OCT-2005; 43(10): 5355-5358.
Eur Surveill 2006; 11(2): 58-60.
Emerg Infect Dis-DEC-2005; 11(120:1835-1841.)
Question 33
1 out of 1 points

Streptococcus pyogenes causes pharyngitis and scarlet fever and is


classified as
Answers:

Group A
Group B

Group C
Group D
Group G

Response
Feedback:

Lancefield grouping A through H, K, and V is based on


serological properties of pathogenic strains possessing group
specific antigens.
Streptococcus pyogenes is -hemolytic Group A Streptococci .
Group B includes Streptococcus agalactiae.
Group C includes Streptococcus anginosus and Streptococcus
equisimilis.
Group D includes Streptococcus bovis and Enterococcus spp.
Group F includes Streptococcus anginosus.

Question 34
1 out of 1 points

Case:
A 58-year-old woman presents with a history of alcohol abuse and insulin
dependent diabetes. She complains of pain in her right calf and shaking
chills. An examination of the right calf is significant for an area of necrosis
that is black in color with no crepitus present. The patient appears
intoxicicated and has an elevated temperature of 102F. A stat Gram stain
and culture is performed on aspirated material from the wound site. The
Gram stain result is significant for gram-positive cocci in chains (refer to the
image). The patient is admitted to the hospital for surgical debridement of
the wound and empiric antibiotic therapy. The culture results were positive
for 4+ beta hemolytic colonies on aerobic blood agar media that were
catalase negative and had a Lancefield grouping of A.
Question:
What is the most likely diagnosis?

Answers:

Bacteremic pseudomonas gangrenous cellulitis


Streptococcal gangrene
Synergistic necrotizing cellulitis
Necrotizing cutaneous mucormycosis
Clostridial myonecrosis (gas gangrene)

Response Streptococcal gangrene is caused primarily by Streptococcus


Feedback pyogenes. Conditions predisposing individuals to this type of
:
infection are diabetes and abdominal surgery. Patients will
present with prominent pain in the infected area, which will
contain necrosis of subcutaneous tissue and fascia. The area
may appear black or "burned" in appearance. Crepitus is absent.
Patients will present with high fevers and marked systemic
toxicity. The clinical course is very rapid. Streptococcus
pyogenes is an aerobic gram-positive cocci that will occur in
short to long chains. It is beta-hemolytic on blood agar, catalase
negative, and has a Lancefield grouping of "A".

Clostridial myonecrosis (gas gangrene) is an infectious gangrene


caused by Clostridium perfringens and occasionally other
clostridium species. Local trauma to the area is usually the
predisposing factor to this type of infection. Patients present
with prominent pain at the infected area, which has marked
swelling and a yellow-bronzed discoloration. There can also be
green-black patches of necrosis with a serosanguinous
discharge. Crepitus is present at the infected areas. The patient
generally will present with a moderate to high fever and a very
marked systemic toxicity. The clinical course is extremely
rapid. Clostridium perfringens is an anaerobic gram-positive rod
that is described as box-car shaped. Clostridium perfringens will
typically produce a double-zone of hemolysis on blood agar
media. A Gram stain of the serosanguinous discharge will be
positive for numerous gram-positive box-car shaped rods.
Bacteremic pseudomonas gangrenous cellulitis is caused
by Pseudomonas aeruginosa. Conditions predisposing
individuals to this type of cellulitis are burns and
immunosuppression. Patients will present with mild pain at the
infected area, which is a sharply demarcated necrotic area with
black eschar and surrounding erythema. There is no crepitus
present at the site. The patient will generally present with a high
fever and marked systemic toxicity. Clinical course is
rapid. Pseudomonas aeruginosa is an aerobic gram-negative
bacilli that is oxidase positive, grows on MacConkey agar media
as a nonlactose fermenter, and produces a blue-green pigment
with a sweet grape-like odor.
Synergistic necrotizing cellulitis is caused by a mixture of
organisms such as Bacteroides, Peptostreptococcus, Escherichia
coli, and other members of the Enterobacteriaceae. Diabetes is
the most common predisposing factor. Patients will present with
prominent pain at the infected area. The cellulitis produces a
thick, foul-smelling, copious "dishwater" smelling drainage.
Crepitus is often present. The patient will usually have moderate
fever with marked systemic toxicity. The clinical course is rapid.
is caused by fungal organisms. The most common causes of
necrotizing cutaneous mucormycosis are the rhizopus, mucor,
and absidia species. Diabetes and corticosteroid therapy are
predisposing conditions for acquiring necrotizing cutaneous
mucormycosis. Pain at the site of infection is minimal and the
site is described as a central black area with a purple raised
margin. The site can also present as just a black ulcer. The
patient will present with a low-grade fever, and systemic toxicity
can be variable. The clinical course is rapid.Necrotizing

cutaneous mucormycosis
Question 35
1 out of 1 points

The expression of the immune response involves a certain amount of


inflammation, cell infiltration, lymph node swelling, and even tissue
destruction. Sometimes they are very severe. There are signs of an immune
response in nearly all infections. Therefore, it is to be expected that there
will nearly always be some contribution of the immune response to
pathological changes. Sometimes this forms the major part of the disease.
For example, in tuberculosis, much of the lung tissue damage is caused by
the release of destructive enzymes by what?
Answers:

Mast cells
Eosinophils
Dendritic cells
Basophils
Macrophages

Response In tuberculosis the pathological picture is dominated by the


Feedback operation of a strong and persistent cell-mediated immunity
:
response to the invading bacillus. In the classical tubercle a
central zone of bacilli with large mononuclear and giant cells,
often with some necrosis, is surrounded by fibroblasts and
lymphocytes. Mononuclear infiltrations, giant cells, and
granulomatous lesions are characteristic pathological features
of tuberculosis. When macrophages are killed by intracellular
mycobacteria, the lysosomal enzymes and other materials
released from the degenerating cell contribute to chronic
inflammation. Activated macrophages and T cells secrete
cytokines that contribute to the immune pathology. There are no
recognizable toxins formed by tubercle bacilli, and there seems
to be no single antigen or other component that accounts for
virulence.
Mast cells are large cells containing distinctive cytoplasmic
granules that contain a mixture of chemical mediators, including

histamine, that act rapidly to make local blood vessels more


permeable. Mast cells have a distinctive appearance after
staining with the dye toluidine blue that makes them readily
identifiable in tissues.
Eosinophils are terminally differentiated, end-stage leukocytes
that reside primarily in submucosal tissue and are recruited to
sites of specific immune responses. They are granulocytes that
can express Fc receptors when activated. Once these receptors
are triggered they degranulate, releasing leukotrienes that
cause muscle contraction.
Dendritic cells are cells with a distinctive branched morphology
found in skin, mucosa, and lymphoid tissues. These cells trap
antigen in the periphery and migrate to lymphoid tissues, where
they present antigens to immune cells. Because of this, and
because of their ability to deliver activating signals, dendritic
cells are known as professional antigen-presenting cells.
are the smallest circulating granulocytes with the least known
function. They arise in the bone marrow, and following
maturation and differentiation, are released into the blood
circulation. Like mast cells, basophils have receptors on their
cell membranes that bind with high affinity to the Fc portion of
IgE.Basophils
Question 36
1 out of 1 points

Case:
An archaeologist who has been excavating a very old Middle Eastern site
develops a pustule on his hand. The pustule then ruptures to form a black
eschar surrounded by expanding brawny edema.
Question:
What organism is the most likely cause of this condition?
Answers:

Bacillus anthracis
Borrelia burgdorferi
Francisella tularensis
Spirillium minus

Yersinia pestis

Response Anthrax forms extremely stable spores and has been


Feedback encountered in very old (deep) dirt in Israeli and Arabian
:
excavation sites. The causative organism, Bacillus anthracis, is
found in many animal species; humans can acquire the
organism either through contact with the animals or from locally
contaminated soil. The pustule described in this patient is called
a 'malignant pustule' and may be accompanied by
lymphadenopathy. Most cases remain localized, but death can
occur as the result of complications such as bacteremia,
meningitis, and pneumonia.
Borrelia burgdorferi causes Lyme disease. In the 1st stage of
this disease, there is a localized expanding erythematous rash
(rather than an eschar). Lyme disease occurs in the United
States, Europe, and Asia; it corresponds to the distribution of
Ixodid ticks which spread the infection.
Francisella tularensis causes tularemia. The organism can
persist for weeks to months, but does not form spores that
could survive for hundreds or thousands of years. Most human
cases occur in the endemic areas of the United States.
Spirillium minus is 1 of the causes of rat-bite fever. There is no
indication this patient was bitten by a rat.
Yersinia pestis causes bubonic plague. This organism does not
form stable spores that could survive for extended periods of
time.

Question 37
1 out of 1 points

Mice have long served as models for the study of the mammalian immune
system. The recessive nude mutation in mice is caused by a mutation in the
gene for the transcription factor Wnt and in homozygous form causes
hairlessness. In animals with this mutation, the thymus fails to form. This
defect causes the animals to produce

Answers:
"B cells, but few T cells"
"T cells, but few B cells"
No B or T cells
Few macrophages or dendritic cells
Few erythrocytes

Response The importance of the thymus in immunity was first discovered


Feedback through experiments on mice. It was found that surgical removal
:
of the thymus at birth resulted in an immunodeficient mouse.
Much evidence has accumulated since to establish
theimportance of the thymus in T cell development.
In nude mice, the thymic epithelium fails to differentiate, and
the affected individuals produce B cells but few T cells. The
thymus provides a specialized and architecturally organized
microenvironment for the development of mature T cells.
are produced in the bone marrow, and bone marrow provides
the essential microenvironment for development of B cells when
they leave the fetal liver. Dendritic cells arise from the bone
marrow and mature in lymphoid tissues.Macrophages also begin
in bone marrow, and migrate to the periphery to become resting
macrophages. They may undergo changes there that prepare
them to activate T cells in an immune response.Erythrocytes
Question 38
1 out of 1 points

Case:
A 37-year-old Caucasian woman swims regularly for exercise. She swims 100
laps, 4 to 6 times per week. She starts to notice severe right ear pain. She
also notes that her right ear is very itchy. She sees her family doctor and
mentions her complaints. When he goes to insert the otoscope, he gently
pulls on her ear. This causes her quite a bit of pain. He notes an inflamed
external ear canal, but the tympanic membrane is normal.
Question:

What is the classification of the most likely pathogen?


Answers:

Gram-positive aerobic rod


Gram-negative aerobic rod
Gram-negative anerobic rod
Gram-negative cocci
Gram-positive cocci

Respons
e
Feedbac
k:

This patient has signs and symptoms of otitis externa,


specifically, "swimmers ear'. An infection of the external auditory
canal, usually due to bacteria, is called otitis externa. Symptoms
include ear pain and itching. The finding of an inflamed external
ear canal is consistent with otitis externa.
The most likely pathogen for swimmer's ear is Pseudomonas
aeruginosa. Pseudomonas aeruginosa is a Gram-negative aerobic
rod. Other pathogens that can cause otitis externa
include Peptostreptococcus, Staphylococcus aureus, Bacteroides,
Proteus, and fungi.

References:
Swimmer's Ear. CDC website. Available
at: http://www.cdc.gov/healthyswimming/swimmers_ear.htm.
Accessed March 18, 2008.
Question 39
1 out of 1 points

Case:
A previously healthy 24-year-old woman presented with high fever of sudden
onset, severe myalgia, headache, vomiting, diarrhea, and diffuse scarlatiniform rash resembling sunburns. She was on the fourth day of her menses
and was using tampons. Her blood pressure was noted as 70/40mm mercury.
As part of laboratory investigations, blood, urine, and vaginal cultures were
done. According to the microbiology report, heavy growth of staphylococcus
aureus was obtained from vaginal culture. Preliminary report of her blood
culture showed no growth after 24 hrs incubation. Microscopy of urine

sediment showed 15 leucocytes per high power field. Urine culture did not
yield any significant growth.
Question:
Which of the following staphylococcal toxins plays a major role in causing
this patient's illness?
Answers:

Staphylococcal alpha toxin


Panton Valentine leucocidin
Exfoliative toxin
Enterotoxin B
TSST1

Response TSST1 (Toxic Shock Syndrome Toxin 1) plays a major role in


Feedback causing the clinical manifestations of toxic shock syndrome in
:
the patient. Exotoxins constitute essential components of the
virulence mechanisms of staphylococcus aureus. Nearly all
strains secrete hemolysins, nucleases, proteases, lipases,
hyaluronidase, and collagenase, which convert host tissue into
nutrients required for bacterial growth. Some strains produce
additional exoproteins that may be responsible for particular
clinical manifestations. The clinical picture of the patient
referred to in the case is suggestive of menstrual toxic shock
syndrome (mTSS) associated with tampon use. The patient has
vaginal colonization/infection with staphylococcus aureus, and
the multi-system involvement is due to intoxication. TSST1
(toxic shock syndrome toxin -1) is accepted as the cause of
100% of mTSS cases. It is a pyrogenic superantigen produced by
about 20% staphylococcus aureus strains and is the only one
known to cause TSS from intra-vaginal sources, presumably due
to its unique capacity to cross mucosal surfaces. TSST-1
interacts simultaneously with MHC class 2 molecules on the
surface of antigen-presenting cells and T-cell receptors forming
a tri-molecular complex. This induces profound T-cell
proliferation, resulting in massive release of bio-active
cytokines, causing TSS. In patients with mTSS, absent or
insufficient titers of neutralizing antibodies to the toxin have
been observed. The exact role of tampons in mTSS is not clear.
Tampons are not the source of toxigenic staphylococcus aureus.
TSS unrelated to menses can occur and is associated with

conditions like post-surgical and post-partum infections. In


addition to TSST1, some of the staphylococcal enterotoxins with
superantigenic activity have been found to contribute to the
pathogeneses of nonmenstrual TSS.
Alpha toxin of staphylococcus aureus is a heterogenous protein,
which can act on a broad spectrum of eukaryotic cell
membranes. It is a potent hemolysin.
Panton Valentine leucocidin is a cytotoxin that causes leucocyte
destruction and necrosis. About 3% of staphylococcus aureus
strains produce PV leucocidin, and they often produce necrotic
lesions such as furunculosis and necrotizing pneumonia.
Exfoliative toxins are epidermolysins and split the epidermis
between 2 cell layers, resulting in desquamation. There are 2
distinct proteins: epidermolytic toxin A, which is a chromosomal
gene product that is heat stable, and epidermolytic toxin B,
which is plasmid-mediated and heat stable. These are
superantigens and produce generalized desquamation of
Staphylococcal scalded skin syndrome (SSSS).
Enterotoxin B is one of the staphylococcal toxins that causes
food poisoning. There are multiple enterotoxins produced
by staphylococcus aureus. Approximately 50% of strains can
produce 1 or more of them. The toxin is heat stable and can
resist the action of gut enzymes. Food poisoning results from
ingestion of preformed toxin in food that has been contaminated
with staphylococcus aureus. It is a self-limiting condition. The
emetic effect of the toxin depends on the activation of
medullary emetic center through vagal and sympathetic
stimulation. Enterotoxin B also possesses superantigenicity,
which refers to the ability of the toxin to activate T lymphocytes
without regard for the antigen specificity of these cells.

References:
Journal of Clinical Microbiology-SEPT-2005; 43 (9): 4628 -34.
Exp Biol Med -2001; 226 (3): 164-176.
BMJ - OCT 2005 ; 331 ( 7520 ) : 793 - 794.
Postgraduate Medicine - OCT- 2001 ; 110 ( 4 ) : 55 - 62.
Question 40

1 out of 1 points

Vaccines can be divided into two classes: live and inactivated. In general,
inactivated vaccines are less successful than live vaccines. This is partly
explained by the fact that they are usually given by an unnatural route.
Furthermore, when a live vaccine is used, the replicating agent provides an
immunogenic stimulus over many days. To produce the equivalent stimulus
with inactivated vaccine would require a vast dose of antigen, with the risk
of producing severe reactions. This problem is overcome by combining the
vaccine with
Answers:

Adjuvants
Immunotoxins
Streptolysins
Immunophilins
Endotoxins

Response Even conjugate vaccines are not usually strongly immunogenic


Feedback on their own; most require the addition of adjuvants, which are
:
substances that enhance the immunogenicity of antigens. Some
commonly used adjuvants are listed.
Oil Adjuvants
Freund's complete adjuvant and incomplete adjuvant
Mineral Salts
Aluminum phosphate or hydroxide, calcium alginate
Double-stranded nucleic acids
Poly (IC), Poly (AU)
Other substances
Saponin, levamisole
In humans, the most widely used adjuvants are aluminum
compounds (alums). These form a precipitate with protein
antigens and result in slow release of the antigen. Alums are
present in vaccines such as tetanus toxoid and diphtheria
toxoid.Biodegradable polymers can be used as delayed-release
capsules, dissolving weeks after injection to release a booster
dose of antigen. A glycoside adjuvant called Quil A extracted
from the bark of an Amazonian oak tree readily forms micelles
and can be used to make an immunostimulatory complex

containing immunizing peptides with an built-in adjuvant.


Linking a tumor-specific or tumor-selective monoclonal antibody
to a toxin is a way to create an immunotoxin, a reagent that can
by used to destroy all or part of a tumor. When the labeled
antibody is internalized, the toxin is cleaved from the antibody
in the cell's endocytic compartment, allowing the toxin to
penetrate and kill the cell.
Certain groups of streptococci secrete a variety of streptolysins
or hemolysins that lyse red blood cells and are much more
active weight for weight than hemolysins such as bile salts or
saponin, but which have a more important toxic action on
polymorphs and macrophages. Various hemolysins are released
also by pathogenic staphylococci, and these can kill phagocytes.
The immunophilins are a family of intracellular proteins that
bind to certain compounds forming complexes that interfere
with signaling pathways important for the clonal expansion of
lymphocytes.
are part of the outer membrane of Gram-negative bacteria.
Some of the diseases in which endotoxins may play a part are
typhoid fever, tularemia, plague and brucellosis, and a variety of
hospital-acquired infections caused by opportunistic Gramnegative pathogens. Endotoxin is a complex lipopolysaccharide
that displays a large array of biological effects.Endotoxins
Question 41
1 out of 1 points

An 80-year-old female with cellulitis of her lower left leg is seen in the
emergency room. She has a history of diabetes. The toes, heel, and ankle
were swollen and there was a bluish hematoma and blister located medially
at the heel. The ankle was tender to the touch and released a foul smelling
gas. A CBC was notable for an elevated white cell count of 30,000/mm3 with
a left shift. The blister was drained for gram stain and culture. The gram
stain was significant for gram-positive ""box car"" shaped rods with an
absence of neutrophils. The cultures were significant for anaerobic grampositive rods that were lecithinase positive, nonmotile, and produced a
double zone of hemolysis on anaerobic blood agar. What infection does this
patient have?
Answers:

Clostridium perfringens

Bacteroides fragilis
Fusobacterium spp
Veillonella spp
Propionibacterium acnes

Respons
e
Feedback
:

Clostridium perfringens is an anaerobic, gram-positive, sporeforming bacillus. The organism is nonmotile and has a distinctive
"box car" appearance on gram stain. It produces oval central
spores, but they are rarely seen in clinical or cultures. All types
produce lecithinase, which is lytic to neutrophils. Because of the
action of lecithinase on neutrophils, gram stains of blister
aspirates and other drainage characteristically contain grampositive rods but little or no neutrophils. Clostridium
perfringens is the main causative agent of gas gangrene, and
diagnosis is made by microbiological findings, clinical findings,
and the demonstration of myonecrosis at surgery. Surgical,
antibiotic, and hyperbaric oxygen treatments are used in
treating gas gangrene.
Bacteroides fragilis are anaerobic, gram-negative bacilli that
grow in 20% bile; they are resistant to kanamycin, vancomycin,
and colistin and are catalase and indole positive. It is the most
common anaerobic isolate from intra-abdominal abscesses. The
organism is part of the normal gastrointestinal flora, though
usually only 0.5% of the colonic microflora. Anaerobic infections
are usually polymicrobial with a mixture of aerobes and
anaerobes. Metronidazole, clindamycin, and chloramphenicol
are some of the antibiotics used to treat Bacteroides fragilis.
Fusobacterium spp. are long, thin anaerobic gram-negative rods
with pointed ends. They are usually arranged in end-to-end
pairs. The organism is indole variable, catalase negative, grows
in 20% bile, and is sensitive to kanamycin and colistin but
resistant to vancomycin. The organism is a cause of brain
abscesses, sinusitis, odontogenic infections, pleuropulmonary
infections, bacteremia, and endocarditis. Most Fusobacterium
spp. are sensitive to penicillin.
Veillonella spp. is anaerobic gram-negative cocci that are found
as part of the normal flora in the oral cavity, upper respiratory
tract, intestine, and vagina. They have a generally low virulence
but can cause significantly serious infections given the right

circumstances, such as steroid therapy, previous injury, foreign


bodies, and IV drug abuse. They are generally susceptible to
most of the antibiotics used to treat other anaerobic infections.
is a gram positive anaerobic cocci. It is part of the normal skin
flora. Though the organism is usually a contaminant of blood
and wound cultures, it can be the cause of serious lifethreatening infections in the proper conditions. These infections
include brain abscesses, osteomyelitis, subdural empyema,
cerebral shunt infection, infective endocarditis, and parotid and
dental infections.Propionibacterium acnes
Question 42
1 out of 1 points

Case:
Smooth strains (S) of Streptococcus pneumoniae are encapsulated and are
pathogenic. Rough strains (R) are not encapsulated and are generally not
pathogenic. When mice are injected with live S strains they die; when
injected with live R strains, the mice survive. Mice also survive when injected
with dead S strains. Interestingly, when dead S strains and live R strains are
mixed together and then injected into mice, the mice die and colonies of S
and R strains can be isolated from the dead mice.
Question:
What is the most likely explanation for these laboratory results?

Answers:

Transportation
Transformation
Conjugation
Transduction
Transposition

Respons
e
Feedbac

In 1928, an English bacteriologist named Frederick Griffith


published the results of the experiment described above. While
the results were curious and unusual in 1928, this experiment

k:

became a classic example of the genetic event now known


astransformation. In transformation, soluble DNA from a donor
cell (in this case, the dead S strain) is taken up by a recipient
cell of the same species but a different genotype (in this case,
the R strain). The DNA from the donor recombines with the DNA
of the recipient resulting in the expression of genetic
characteristics of the donor (the ability to manufacture capsular
material).
Conjugation and transduction are also important mechanisms of
DNA transfer, but soluble DNA is not involved in these
processes.
Transportation and transposition are unrelated to the genetic
events described above.

References:
Carroll KC, Hobden JA, Miller S, Morse SA, Mietzner TA, Detrick B,
Mitchell TG, McKerrow JH, Sakanari JA. Carroll K.C., Hobden J.A.,
Miller S, Morse S.A., Mietzner T.A., Detrick B, Mitchell T.G.,
McKerrow J.H., Sakanari J.A. Carroll, Karen C., et al.Microbial
Genetics. In: Carroll KC, Hobden JA, Miller S, Morse SA, Mietzner
TA, Detrick B, Mitchell TG, McKerrow JH, Sakanari JA. Carroll K.C.,
Hobden J.A., Miller S, Morse S.A., Mietzner T.A., Detrick B,
Mitchell T.G., McKerrow J.H., Sakanari J.A. Eds. Karen C. Carroll, et
al.eds. Jawetz, Melnick, & Adelbergs Medical Microbiology,
27e. New York, NY: McGraw-Hill; 2015.
Question 43
1 out of 1 points

As part of a study on organisms associated with bacterial vaginosis, samples


of vaginal discharge collected from women clinically diagnosed with the
condition were cultured for aerobic and anaerobic bacteria. About 20% of
the isolates belonged to an obligate anaerobic species of small non-motile
Gram-negative saccharolytic bacillus. These isolates were resistant to
penicillin and produced beta-lactamase. This bacterial species most likely
belongs to what genera?
Answers:

Mycoplasma
Prevotella

Gardnerella
Lactobacillus
Propionibacterium
Mobiluncus

Respons
e
Feedbac
k:

Bacterial vaginosis (BV) is a lower genital tract syndrome that


affects women of reproductive age. Though the pathogenesis of
BV is not well understood, microbes associated with BV mostly
belong to endogenous flora of the vagina.
Commensals of the genital tract of healthy females include
aerobic and anaerobic bacteria, with predominance
of Lactobacillus. In BV, the Lactobacillus that produces H2O2 is
depleted and is replaced by overgrowth of aerobic and anaerobic
flora, including Gardnerella vaginalis, Mobiluncus
curtisii, Mycoplasma hominis, and Prevotella bivia.

Lactobacillus and Propionibacterium, though anaerobes, are


Gram-positive bacilli, and Propionibacterium is a commensal of
skin. Mycoplasma and Gardnerella are not obligate anaerobes.
Therefore, the possibility of the isolates belonging to these 4
genera is excluded.

Mobiluncus species are obligate anaerobes and are known to be


associated with BV. But they are curved motile bacilli and often
appear Gram-variable. Prevotella species are obligately
anaerobic non-motile Gram-negative bacilli. They are
saccharolytic and high percentage of the strains produce betalactamase. So the anerobic bacterial species referred to in the
question is likely to belong to the genus Prevotella.

G.vaginalis is considered the predominant bacterial species


associated with BV. BV is associated with increased risk of
obstetric and gynecological complications and acquisition of HIV
and other STDs.
A commensal or symbiotic relationship has been observed
between Prevotella bivia and G.vaginalis.

P.bivia and P.disiensis are known to be associated with upper


genital tract infections such as tubo-ovarian and pelvic
abscesses in women. Together with other bacterial
species, P.bivia has been isolated from various lesions like
perirectal abscesses, septic arthritis, intracranial abscess,
endocarditis, infected wounds, and paronychia. Based on
experiments in animal models, it is suggested thatP.bivia in
conjunction with an aerobic organism can be more pathogenic.
Molecular methods have been found useful for identification of
the different bacterial species associated with BV
including Prevotella.
Other prevotella species like P.melaninogenicus that occur as
oral indigenous flora are found to be associated with
endogenous periodontal and pulmonary infections.
References:
Jawetz, Melnick & Adelberg's Medical Microbiology-23rd Ed 2004
copyright McGraw-Hill Co: pp 306,307,309,722.
Journal of Clinical Microbiology-OCT-2003; 41(10): 4901-4903.
Journal of Clinical Microbiology-SEPT-2005; 43(9): 4492-4497.
Chemotherapy-MAR-2005;51(1): 9-14.
Expert Rev Ant Infect Ther-DEC-2004; 2(6): 913-922.
Am J Obstet Gynecol-SEPT-2005; 193(3Pt1): 752-756.
Question 44
1 out of 1 points

Diphtheria toxin acts by


Answers:

Inhibiting protein synthesis


Blocking mRNA binding
Producing edema
Increasing translation

Response Diphtheria toxin acts by inhibiting protein synthesis. It contains

Feedback both an active subunit A, which catalyzes the toxic activity, and
:
a subunit B, which mediates receptor binding and membrane
translocation. Separation of the two subunits is required for full
activity of the A subunit on its target protein elongation factor 2
(EF-2), which transfers polypeptidyl transfer RNA from acceptor
to donor sites on the ribosome of the host cell. The specific
action of the A subunit is to catalyze the transfer of the
adenosine ribose phosphate portion of the nicotinamide adenine
dinucleotide (NAD) to EF-2, an enzymatic reaction called ADPribosylation. Covalent attachment of the ADP-ribosyl groups
occurs at an unusual derivative of histidine called dipthamide.
This inactivates EF-2 and shuts off protein synthesis.
Question 45
0 out of 1 points

You are treating an 85-year-old man in a retirement home. Since his bladder
incontinence has worsened, you provide him with a Foley catheter. You
advise his caregiver that bacterial contamination can be avoided by doing
what?
Answers:

Daily cleansing of the meatus with antiseptic solution


Administration of prophylactic antibiotics
Using silicone catheters

Daily cleansing of meatus and catheter with soap and water


Scheduling weekly catheter changes

Response Daily cleansing of meatus and catheter with soap and


Feedback water reduces bacterial counts at the entry point to the bladder.
:
The other answers are incorrect. Daily cleansing of the meatus
with antiseptic solution would irritate the mucosa of the
glans. Administration of prophylactic antibiotics is unnecessary
and carries the risk of developing antibiotic resistances. Latex
catheters need to be changed every 2 weeks, and silicone
catheters need to be changed every 3 months. There is no proof
that the catheter material has any influence in bacterial

contamination. Since every invasive procedure increases the


risk of infection, scheduling weekly catheter changes would be
counterproductive.
Question 46
1 out of 1 points

Autoimmune diseases such as Type 1 diabetes, RA, and MS are common,


often devastating diseases. The key feature of these diseases is the
development and persistence of inflammatory processes in the apparent
absence of pathogens, leading to destruction of the target tissue. Presently
no comprehensive explanation can be given for the onset of autoimmune
disease. As a rule, the chronic activation of a particular type of cell appears
to be crucial for fueling the destructive autoimmune processes. What is this
cell type?
Answers:
T helper cells
T suppressor cells
Macrophages
Dendritic cells
Basophils

Response Adaptive immune responses are initiated by activating antigenFeedback specific T cells, and it is believed that autoimmunity is initiated
:
in the same way. T cell responses to self-antigens can inflict
tissue damage either directly or indirectly. Cytotoxic T-cell
responses and inappropriate activation of macrophages by T
helper cells can cause extensive tissue damage,
whereas inappropriate T cell help to self-reactive B cells can
initiate harmful autoantibody responses. Chronic activation of
these T helper cells can cause sustained autoimmune disease
with lasting tissue damage.
One way by which normal animals may control their immune
responses is by induction of cells that function to suppress the
immune response, so-called T suppressor cells. Results in some
experimental systems have indicated that inhibiting suppressor

T cells can result in autoimmunity.


Macrophages are phagocytes derived from blood monocytes.
The monocyte itself is a small, spherical cell with few
projections, abundant cytoplasm, little endoplasmic reticulum,
and many granules. Following migration of monocytes from the
blood to various tissues, they undergo further differentiation
into a variety of histologic forms, including several kinds of
macrophages.
Dendritic cells are cells with a distinctive branched morphology
found in the T cell areas of lymphoid tissue and other areas.
These cells trap antigen in the periphery and migrate to
lymphoid tissues, where they present antigens to T cells.
Because of this, and because of their ability to deliver activating
signals, dendritic cells are known as professional antigenpresenting cells.
are the smallest circulating granulocytes with the least known
function. Their function is probably similar and complementary
to that of eosinophils and mast cells. They arise in the bone
marrow, and following maturation and differentiation, are
released into the blood circulation. Like mast cells, basophils
have receptors on their cell membranes that bind with high
affinity to the Fc portion of IgE.Basophils
Question 47
1 out of 1 points

Case:
The latest news from Haiti is that a cholera epidemic had killed more than
1,300 people and sickened thousands after a series of devastating
earthquakes. Some scientists say that this outbreak is part of a 49-year-old
global pandemic and likely was brought to the Caribbean country in a single
instance. Other says that it may be impossible to trace how the cholera
came to Haiti. You know that cholera is one of the most rapidly fatal illnesses
known. It is considered a biological weapon.
Question:
Effective controls would require research on a process of signal transduction
similar to the process caused by what toxin?
Answers:

Pertussis toxin

Escherichia coli heat-labile enterotoxin


Shiga toxin
Tetanus toxin
Botulism toxin

Respons
e
Feedbac
k:

Enterotoxigenic Echerichia coli (E. Coli) may produce a heatlabile enterotoxin (LT toxin) that is similar in molecular size,
sequence, antigenicity, and function to cholera toxin. It
recognizes and binds to the same receptors as cholera toxin,
disables GTP-ase function of G-proteins, and causes adenylate
cyclase A to produce large amounts of cyclic AMP, which results
in the loss of fluid and salts across the lining of the gut (normally,
the G proteins quickly return to "off" by hydrolizing GTP to GDP).
Pertussis toxin, upon activation, catalyzes ADP ribosylation of G
proteins, thus blocking the inhibition of adenylate cyclase and
leading to increased cellular concentrations of cAMP.
Shiga toxin is a protein toxin, which has a moiety that binds to
the cell surface and enzymatically active moiety that after entry
into the cytosol inhibits protein synthesis. The toxins can also
cause apoptosis by mechanisms that may be different from the
effect on the protein synthesis. Shigella dysenteriae, some
strains of Escherichia coli, as well as other bacteria can secrete
such toxins and cause serious complications during infections.
Tetanus toxin is Zn++ dependent protease that inhibits
neurotransmission at inhibitory synapses, resulting in spastic
paralysis.
Botulinus toxin is Zn++ dependent protease that inhibits
neurotransmission at neuromuscular synapses, resulting in
flaccid paralysis.

References:
Todar's Online Textbook of Bacteriology, available at:
http://www.textbookofbacteriology.net/proteintoxins.html
Retrieved November 22. 2010.
Question 48

1 out of 1 points

Refer to the diagram of B cell development. In what stage would you expect
to see active RAG?

Answers:
Pro-B cell and pre-B cell
Immature and mature B cell
Pre-B cell and immature B cell
Pro-B and mature B cell
Stem cell and mature B cell

Response Active RAG is needed for V-D-J and V-J recombination of variable
Feedback regions of B cells. The recombination of variable regions occurs
:
at the heavy chain gene in the pro-B cell stage and on the light
chain gene in the pre-B cell stage.
Mature and Immature B cells already have rearranged
immunoglobulins.
At immature stage, the light and the heavy chain genes have
finished somatic recombination and RNA processing begins to
allow expression of IgM and IgD.
At the mature stage, the light and the heavy chain genes have
finished somatic recombination and both IgM and IgD are
expressed.
References:
Murphy Kenneth, Paul Travers, Mark Walport. Janeway's

Immunobiology, 7th ed., Taylor & Francis, Inc. 2007.


Villa A, Sobacchi C, Notarangelo LD, et al. V(D)J recombination
defects in lymphocytes due to RAG mutations: severe
immunodeficiency with a spectrum of clinical presentations.
Blood. 2001 Jan 1;97(1):81-8.
Noordzij JG, de Bruin-Versteeg S, Verkaik NS, et al. The
immunophenotypic and immunogenotypic B-cell differentiation
arrest in bone marrow of RAG-deficient SCID patients
corresponds to residual recombination activities of mutated RAG
proteins. Blood. 2002 Sep 15;100(6):2145-52.
Question 49
1 out of 1 points

Many bacteria, including Neisseria meningitidis, Streptococcus pneumoniae,


and Haemophilus spp., have an outer capsule composed of polysaccharides
that are species- and type-specific for particular strains of bacteria. These
bacteria owe their success to their ability to survive and grow extracellularly,
avoiding uptake by phagocytic cells. What is the most effective defense
against these microorganisms?
Answers:

Neutralization
Pinocytosis
Opsonization
Endocytosis
Apoptosis

Respons
e
Feedbac
k:

One role of antibody is to enable a phagocytic cell to ingest and


destroy bacteria. This is important for the many bacteria that
are resistant to direct recognition by phagocytes; instead, the
phagocytes recognize the constant region of the antibodies
coating the bacteria. The coating of pathogens and foreign
particles by antibodies is opsonization. Macrophages and other
phagocytic cells have receptors for the Fc portion of the
antibody molecule. These receptors provide a "handle" for

phagocytes to bind coated particles.


A simple and direct way in which antibodies can protect from
pathogens or their toxic products is by binding to them,
thereby blocking their access to cells that they may infect or
destroy. This is known as neutralization and is important for
protection against bacterial toxins and against pathogens such
as viruses, which can thus be prevented from entering cells and
replicating.
Phagocytosis is the ingestion of these coated pathogens or
particles by macrophages or other phagocytes, followed by
internalization and destruction by intracellular digestion.
Pinocytosis is the simple cellular uptake of fluid and solutes. It is
a much simpler process than phagocytosis.
Some enveloped viruses enter the cell
using endocytosis, the invagination of the plasma
membrane into smaller virus-sized depressions coated on the
cytoplasmic side with a cellular protein known as clathrin. These
detach from the plasma membrane and become vesicles free in
the cell's cytoplasm.
represents physiological, as opposed to pathological (necrotic),
cell death; it is the cellular equivalent of suicide. Apoptotic cells
are recognized by phagocytes and removed.Apoptosis
Question 50
1 out of 1 points

Case:
A mother brings her 5-year-old son into your office. The boy has papular and
pustular lesions on his face. A serous honey-colored fluid exudes from the
lesions. You suspect impetigo. A gram stain reveals spherical gram-positive
arrangements in irregular-grape like clusters.
Question:
What is the most likely organism causing this patient s condition?
Answers:

Staphylococcus epidermidis
Staphylococcus aureus

Peptostreptococcus
Streptococcus pneumoniae
Haemophilus influenzae

Respons
e
Feedbac
k:

The history and lab findings suggest Staphylococcus aureus as


the causative organism. The most common causes of impetigo is
usually more than likely Staphylococcus aureus, but also at
times may be beta-hemolytic streptococcus group A.
S. aureus causes inflammatory and toxin-mediated diseases.
When a gram stain has been completed S. aureus will appear as
spherical gram-positive cocci arranged in irregular grape-like
clusters, such as what is described above. Inflammatory
diseases that can be caused by an active S. aureus infection
include 1) skin infections, including impetigo, furuncles,
carbuncles, cellulitis, surgical wound infections, eyelid infections,
and postpartum breast infections; 2) septicemia (sepsis) can
originate from any localized lesion, especially wound infection,
or as a result of intravenous drug abuse; 3) endocarditis on
normal or prosthetic heart valves; 4) osteomyelitis and arthritis,
either hematogenous or traumatic; 5) pneumonia in
postoperative patients or following viral respiratory infection,
especially influenza; and 6) abscesses (metastatic) in any organ.
Streptococcus pneumoniae are Gram-positive lancet-shaped
cocci arranged in pairs (diplococci) or short chains. On blood
agar they produce alpha-hemolysis. Virulence factors
of pneumococci are polysaccharide capsules. Pneumococcicause
pneumonia, bacteremia, meningitis, and infections of the upper
respiratory tract such as otitis and sinusitis. Mortality rate is high
in elderly, immunocompromised (especially splenectomized), or
debilitated patients. They should be immunized with the
polyvalent polysaccharide vaccine.
Peptostreptococci are anaerobic Gram-positive cocci. It grows
under anaerobic or microaerophilic conditions and produces
variable hemolysis. Peptostreptococci are members of the
normal flora of the gut and female genital tract and participate
in mixed anaerobic infections of the abdomen, pelvis, and brain.
Staphylococcus epidermidis is part of normal human flora on the
skin and mucous membranes, but can cause infections of
intravenous catheters and prosthetic implants. This organism is
particularly infectious in the inpatient hospital setting in those

patients with compromised immune systems. Gram stain of


these bacterium reveal gram-positive cocci arranged in grape
like clusters. Although the gram stain characteristics are similar
to the correct answer, the patient scenario is inappropriate.
Haemophilus influenzae is the incorrect choice in this scenario
due to the fact that on gram stain characteristics seen include a
gram negative and rod shaped.

References:
1. ODell ML. Skin and Wound Infections: an overview. Am Fam
Physician. 1998 May 15;57(10):2424-2432.
2. Crowley, LV. An Introduction to Human Disease: Pathology and
Pathophysiology Correlations. 8th ed. Boston: MA: Jones and
Bartlett Publishers; 2010: 94-103
3. Hay WW, Levin MJ, Deterding RR, Abzug MJ, Sondheimer JM.
Current Diagnosis & Treatment: Pediatrics. 21st ed. New York:
NY: McGraw Hill Medical; 2012: 412-413.
Wolff K, Johnson RA, Saavedra AP. Fitzpatricks Color Atlas and
Synopsis of Clinical Dermatology. 7th ed. New York, New York:
McGraw Hill Medical; 2013:
Question 51
0 out of 1 points

Case:
A 30-year-old man presents with a 1-hour history of severe nausea and
vomiting. Prior to falling ill, he had been at a party where he ate pudding
(along with other food). Physical examination reveals a normal temperature
with mild, diffuse tenderness of the abdomen. The organism isolated is a
Gram-positive coccus that occurs in grape-like clusters, is catalase and
coagulase-positive, and forms a golden-yellow colony on agar.
Question:
What toxin released by the causative organism is responsible for the
patient's symptoms?
Answers:

Toxic shock syndrome toxin (TSST-1)


Exfoliatin toxin

Enterotoxin
Leukocidin
Alpha toxin

Response Enterotoxin produced by Staphylococcus aureus is an exotoxin;


Feedback it is responsible for the patient's condition. The clinical
:
presentation and the laboratory findings (e.g., a short
incubation period of 1 - 6 hours with predominant emesis) are
suggestive of Staphylococcal food poisoning. S.
aureus organisms are Gram-positive cocci that occur in grapelike clusters. They are catalase and coagulase-positive, and they
form golden-yellow colonies on agar. Staphylococcal food
poisoning results from the ingestion of preformed enterotoxins
on food contaminated with S. aureus. Bacteria growing in
carbohydrates and meat products produce enterotoxins that,
upon ingestion, diffuse into the circulation and cause emesis by
stimulating the vomiting center in the central nervous system.
Toxic shock syndrome toxin (TSST-1) has superantigen activity,
and when expressed systemically it causes life-threatening toxic
shock syndrome. The clinical presentation includes fever,
hypotension, and diffuse macular erythema; there is
involvement of 3 or more of the organ systems (i.e.,
gastrointestinal, renal, hepatic, musculoskeletal, and nervous
system).
Exfoliatin toxin (ET) elaborated by S. aureus causes scalded skin
syndrome; this is manifested as widespread blistering and loss
of the epidermis, revealing a red base. ETA and ETB are the 2
antigenically distinct forms of the toxin. ET has esterase and
protease activity which targets a protein involved in maintaining
epidermal integrity; this causes separation of the epidermis.
Leukocidin is a membrane-damaging toxin expressed by S.
aureus that acts on polymorphonuclear leukocytes.
Alpha toxin, or alpha hemolysin, is the most potent membranedamaging toxin of S. aureus; it causes hemolysis.

References:
References:

Ikeda T, Tamate N, Yamaguchi K, Makino S. Mass Outbreak of


Food Poisoning Disease Caused by Small Amounts of
Staphylococcal Enterotoxins A and H. Appl Environ Microbiol.
2005 May; 71(5): 2793-2795.
Dinges MM, Orwin PM, Schlievert PM. Exotoxins of
Staphylococcus aureus. Clin Microbiol Rev. 2000 Jan; 13(1):1634.

Question 52
1 out of 1 points

A 20-year-old man goes to the primary care doctor presenting with a cough
for a few days with purulent sputum and fever. He has a condition that was
diagnosed with an exam of his sweat chloride values by the quantitative
pilocarpine iontophoresis test. The culture of the sputum shows gram
negative rods and has a blue-green pigment and a sweet odor. What is the
most likely pathogen?
Answers:

Bordetella pertussis
Legionella pneumophila
Listeria monocytogenes
Pseudomonas aeruginosa
Streptococcus pneumoniae

Response Sweat chloride values are abnormal in cystic fibrosis, which is an


Feedback autosomal recessive disorder caused by a mutation in
:
chromosome 7, producing a defective CFTR protein. These
patients are very susceptible to respiratory infection by
Pseudomonas aeruginosa, a gram negative rod that produces
blue-green pigment cultures and a sweet odor.
Bordetella pertussis is the causative organism of pertussis.
Legionella pneumophila causes Legionnaire's disease.Listeriosis
is caused by Listeria monocytogenes. Streptococcus
pneumoniae causes a variety of infections including pneumonia,
meningitis, brain abscess, and osteomyelitis.

References:
Ronald L. Gibson, Jane L. Burns, and Bonnie W. Ramsey.
Pathophysiology and Management of Pulmonary Infections in
Cystic Fibrosis. Am. J. Respir. Crit. Care Med., Oct 2003; 168: 918
- 951
Nele Wellinghausen, Juliane Kthe, Beate Wirths, Anja Sigge,
and Sven Poppert. Superiority of Molecular Techniques for
Identification of Gram-Negative, Oxidase-Positive Rods,
Including Morphologically Nontypical Pseudomonas aeruginosa,
from Patients with Cystic Fibrosis J. Clin. Microbiol., Aug 2005;
43: 4070 - 4075.
O'Malley CA, VandenBranden SL, Zheng XT et al. A day in the
life of a nebulizer: surveillance for bacterial growth in nebulizer
equipment of children with cystic fibrosis in the hospital setting.
Respir Care. 2007 Mar;52(3):258-62.
Question 53
1 out of 1 points

The laboratory you are working in is studying the regulation of


immunoglobulin synthesis. You know that in order to produce the complete
immunoglobulin protein, changes must occur at the DNA level. The
production of complete immunoglobulin proteins occurs because of
Answers: Rearrangement of DNA in the germline
Rearrangement of DNA in the B cells
Splicing of RNA in B cells
Deletion of immunoglobulin genes which are not needed
Reorganization of large chromosome fragments in the germline

Response In the embryo (germline), DNA coding for the variable region of
Feedback the immunoglobulin is physically separated from the DNA
:
coding for the constant region. The heavy and light chains are
coded by two separate pools of DNA sequences. The larger pool
codes for the variable regions, while a smaller pool encodes the
constant regions. In B lymphocyte DNA, a variable gene

sequence can be selected and the DNA rearranged so it


becomes joined to a constant region. This is termed DNA or
gene rearrangement. This "new" gene is then transcribed and
processed to give mRNA, which is translated to give a protein
corresponding to a specific immunoglobulin. This gene
rearrangement allows for an endless number of combinations of
variable and light chain allowing for the generation of diverse
antibodies to antigens to which the body is exposed.
Question 54
1 out of 1 points

Case:
A 37-year-old Caucasian woman swims regularly for exercise. She swims 100
laps 4 to 6 times per week. She starts to notice severe right ear pain. She
also notes that her right ear is very itchy. She sees her family doctor and
mentions her symptoms. When he goes to insert the otoscope, he gently
pulls on her ear. This causes her quite a bit of pain. He notes an inflamed
external ear canal, but the tympanic membrane is normal.
Question:
What is the most likely diagnosis?

Answers:
Otitis externa
Otitis media
Malignant otitis
Otosclerosis
Otorrhagia

Respons
e
Feedbac
k:

This patient has signs and symptoms of otitis externa, specifically


"swimmer's ear". An infection of the external auditory canal,
usually due to bacteria, is called otitis externa. Symptoms include
ear pain and itching. The finding of an inflamed external ear

canal is consistent with otitis externa. The most likely pathogen


for swimmer's ear is Pseudomonas aeruginosa. Pseudomonas
aeruginosa is a Gram-negative aerobic rod. Other pathogens that
can cause otitis externa
include Peptostreptococcus,Staphylococcus aureus, Bacteroides,
Proteus, and fungi.
Otitis media is an infection of the middle ear. In contrast to the
scenario here, the external auditory canal would not be involved,
and the tympanic membrane would more than likely be involved.
Malignant otitis is an invasive otitis externa.
Otosclerosis is calcification that occurs in the ear.
Otorrhagia is bleeding that occurs from the ear.
The included image is a photomicrograph of Pseudomonas
aeruginosa. The image is courtesy of the CDC.

References:
Swimmer's Ear. CDC website. Available at:
http://www.cdc.gov/healthyswimming/swimmers_ear.htm.
Accessed August 10, 2015.
Question 55
1 out of 1 points

An antigen can be defined as any foreign molecule that will stimulate


antibody formation when offered appropriately to the host. The newlyformed antibody will react specifically to the immunizing antigen. The most
immunogenic chemical family is
Answers:

Carbohydrates
Lipids
Nucleic acids
Haptens

Proteins

Respons
e
Feedback
:

Virtually all proteins are immunogenic. Thus, the most


common immune responses are those to proteins. Furthermore,
the greater the degree of complexity of the protein, the more
vigorous will be the immune response to that protein. In general,
proteins are multideterminant antigens and they produce
the strongest responses.
Carbohydrates are potentially, but not always, immunogenic. An
immune response, consisting primarily of antibodies, can be
induced against many kinds of polysaccharide molecules, such
as components of microorganisms and of eukaryotic cells.
Lipids are rarely immunogenic, but an immune response to lipids
may be induced if the lipids are conjugated to protein carriers.
Thus, in a sense, lipids are haptens.
Nucleic acids are poor immunogens by themselves, but they
become immunogenic when they are conjugated to protein
carriers. One important example in clinical medicine of a
response to nucleic acid is the appearance of anti-DNA
antibodies in patients with systemic lupus erythematosus.
are very small compounds such as a benzene ring, a glucose
molecule, or a phosphatidylcholine molecule. These small
molecules by themselves cannot induce an immune response. If
they are covalently attached to an immunogenic substance such
as a protein, they can be recognized by a B cell's antigen
receptor, or antibody. The antigen can then be internalized,
processed and presented to a helper T cell, and specific
antibody can be produced against the haptens by B
cells.Haptens

Question 56
1 out of 1 points

The organism known as a significant cause of septicemia, pneumonia and


meningitis in newborns is
Answers:

Group A Streptococcus
Group B Streptococcus

Group C Streptococcus
Group D Streptococcus
Group G Streptococcus

Respons
e
Feedbac
k:

Group B Streptococcus or Streptococcus agalactiae colonizes the


upper respiratory tract, lower gastrointestinal tract, and vagina.
Infection, with subsequent development of disease in the
neonates can occur in utero, at the time of birth leading to early
onset disease. Early onset disease is characterized by
septicemia, pneumonia and meningitis in newborns.
Late onset infection occurs between 1 week and 3 months after
birth and is commonly manifested as meningitis.

Question 57
1 out of 1 points

The successive stages of B cell development are marked by steps in the


rearrangement and expression of the immunoglobulin genes, as well as by
changes in the expression of cell-surface and intracellular molecules. This
developmental program generates B cells in the fetal liver and continues in
the bone marrow. B cell development is dependent on what structure's nonlymphoid cells?
Answers:
Stroma
Lymph node
Peyer's patch
Spleen
Thymus

Response The development of B lymphocytes is dependent on the nonFeedback lymphoid stromal cells found in the bone marrow; stem cells
:
isolated from the bone marrow and grown in culture fail to

differentiate into B cells unless bone marrow stromal cells are


also present. The stroma thus provides a necessary support for
B cell development. The contribution of the stromal cells is
twofold. First, they form specific adhesion contacts with the
developing B-lineage cells by interactions between cell adhesion
molecules (CAMS) and their ligands. Second, they
provide growth factors. For example, a small chemokine known
as PBSF/SDF-1, which is produced constitutively by bone marrow
stromal cells, has an important role in early stages of B cell
development.
The lymph node is a highly organized lymphoid structure that is
the site of convergence of an extensive system of vessels that
collect the extracellular fluid from the tissues and return it to
the blood. Lymph nodes contain B and T lymphocytes.Peyer's
patches are specialized structures in the gut that collect antigen
from the epithelial surfaces of the gastrointestinal tract.
The spleen is an organ behind the stomach that collects antigen
from the blood, and also collects senescent red blood cells.
The thymus is a central lymphoid organ in the upper chest that
serves as a site for T lymphocytes to undergo maturation. The
lymph nodes, Peyer's patches, and spleen all operate on the
same principal: to trap antigen from sites of infection and
present it to migratory small lymphocytes, thus inducing
adaptive immune responses.
Question 58
1 out of 1 points

Case:
A 15-year-old girl presented with sore throat and low-grade fever of 4 days
duration. She had received all childhood immunizations. On examination, a
grayish white patch was observed on her pharynx. Throat swabs were
collected for microbiological examination. Gram stain of throat swab smears
showed predominance of gram-positive beaded bacilli. Smears stained by
Albert's stain showed long slender bacilli with green colored body and deep
blue prominent granules. The bacteria were observed to have cuneiform
arrangement. Throat swab cultures were done on Loeffler's serum medium,
sheep blood agar and Hoyle's tellurite medium. Bacteria showing the same
morphology as in the throat swab smears grew on Loeffler's serum medium,
producing small disc-like colonies after 12 hours incubation at 37C. On
blood agar small beta hemolytic colonies were seen after 24 hours
incubation and on tellurite medium small grayish black colonies appeared
after 48 hours incubation. The isolated bacterium was catalase positive,

urease negative, pyrazinamidase negative, fermented glucose and maltose


with acid production, and reduced nitrate to nitrite. It was susceptible to
many antibiotics including penicillin and erythromycin and gave positive
Elek's test.
Question:
The pharyngeal infection of the girl is most likely to be due to what bacteria?
Answers:

Corynebacterium xerosis
Corynebacterium diphtheriae
Corynebacterium pseudotuberculosis
Corynebacterium jeikeum
Corynebacterium pseudodiphtheriticum

Respon
se
Feedba
ck:

The pharyngeal lesion of the patient characterized by grayish


white patch is suggestive of diphtheritic pseudomembrane and
the morphology and biochemical characteristics of the isolate are
suggestive of Corynebacterium diphtheriae. Elek's test is an in
vitro immunoprecipitation test used for detection of diphtheria
toxin. A positive test indicates that the isolate is a toxigenic
strain.
Among the other options, Corynebacterium pseudotuberculosis is
the only species known to acquire ability to produce diphtheria
toxin. It is a primary pathogen of animals and can be
differentiated by a positive urease test and negative nitrate
reduction. Corynebacterium xerosis and Corynebacterium
pseudodiphtheriticum are non-toxigenic and generally nonpathogenic. Production of pyrazinamidase (pyrazine carboxyl
amidase) is one of the phenotypic features used for identification
of corynebacterium species. Most non-toxigenic corynebacteria
produce pyrazinamidase. Corynebacterium diphtheriae does not
produce this
enzyme. C.xerosis and C.pseudodiphtheriticum strains are usually
pyrazinamidase positive. Susceptibility to antibiotics helps to
exclude Corynebacterium jeikeium, which is a multiresistant
corynebacterium.
Pharyngeal diphtheria is one of the common clinical
manifestations of infection by C.diphtheriae and grayish white
pseudomembrane formation is characteristic of the disease.

Often marked adenitis, causing swelling of the neck (bull neck


appearance), is seen. The most important virulence factor of the
organism is its exotoxin. Serious systemic complications like
asphyxia due to mechanical obstruction of the respiratory
passage by the pseudomembrane and severe toxemia leading to
myocarditis and post-diphtheritic paralysis may occur. Once
diphtheria is diagnosed clinically, diphtheria antitoxin should be
administered to the patient without waiting for the microbiology
report. Antibiotics are given to eliminate the organisms. In a
patient, clinically diagnosed with pharyngeal diphtheria, culture
for the organisms is done to confirm the diagnosis and to exclude
staphylococcal and streptococcal pharyngitis and throat lesions
associated with other conditions like infectious mononucleosis.
Cultural isolation also helps to differentiate the isolate from
morphologically resembling commensal corynebacteria and to
test for its toxigenicity.
Other clinical forms of diphtheria include laryngeal, nasal, otitic,
conjunctival, and cutaneous. Diphtheria was mainly considered as
a childhood disease and after the introduction of immunization,
the incidence of the disease was found considerably reduced in
developed countries and to some extent in developing countries.
The prolonged and extensive epidemic of diphtheria by toxigenic
diphtheria bacilli in the 1990s in the Soviet Union indicated the
dramatic return of the disease and need for strict adherence to
immunization policies. Recent outbreaks reported occurrence of
majority of cases in adolescents and adults. Those who had
completed childhood immunization against diphtheria also were
found susceptible because of the waning immunity. Sporadic
cases of diphtheria in adults have been reported in the US as
well. Based on these observations, CDC has recommended
booster doses of vaccines containing diphtheria toxoid for all
persons above 7 years of age every 10 years in order to maintain
adequate immunity among all age groups.
The exotoxin of C.diphtheriae is encoded by tox gene and this
gene is conferred by lysogenization by one of the
corynebacteriophages, especially the beta phage. The regulation
of tox gene expression is mediated by DtxR gene on the bacterial
chromosome and is influenced by the iron content in the medium.
Decreased iron concentration inhibits the regulator
gene DtxR and leads to increased toxin production. The toxin can
be detected by various methods. In vitro Elek's
immunoprecipitation test is the conventional method. PCR-based
methods for detection of the tox gene in C.diphtheriae strains
and in clinical specimens have been developed and are in use. An

immunochromatic strip assay for rapid detection of diphtheria


toxin and an enzyme-linked immunosorbant assay (ELISA) test
have also been reported to be useful.
Immunization by diphtheria toxoid induces protective levels of
antitoxin in circulation but it has no antibacterial activity. Nontoxigenic strains of C.diphtheriae also are found to be pathogenic.
Infections include pharyngitis and also invasive diseases like
endocarditis and septic arthritis. Such infections have been more
frequently reported in previously immunized individuals. Invasive
infections are reported to be often caused by certain invasive
clones and are seen in injection drug users and homeless
alcoholics, transmission being facilitated by crowding and
unhygienic living conditions. The pathogenesis of nontoxigenic C.diphtheriae is not clearly understood.
Corynebacterium xerosis is a human commensal
corynebacterium found in the conjunctival sac. It is
pyrazinamidase positive unlike C.diphtheriae and is not known to
produce toxin. Infections caused are very rare, mainly affecting
immunocompromised patients. It has been isolated from patients
with endocarditis with predisposing cardiovascular lesions.
Corynebacterium pseudotuberculosis is of veterinary importance.
It produces caseous lymphadenitis in horses, sheep, and goats.
Cases of human lymphadenitis due to this organism mostly
associated with occupational exposure have been reported,
though very rarely. C.pseudotuberculosis is urease positive,
pyrazinamidase negative.
Corynebacterium jeikeium is an opportunistic pathogen causing
acute invasive infections like endocarditis in immunosuppressed
individuals. The infections are associated with high mortality, as
the bacterium shows high resistance to most of the antibiotics,
responding only to vancomycin.
Corynebacterium pseudodiphtheriticum (C.hofmannii) is a
commensal of the throat. Though it morphologically
resembles C.diphtheriae, it can be differentiated by biochemical
tests. It does not ferment sugars and is urease and
pyrazinamidase positive. It is reported to cause endocarditis in
patients with prosthetic valve replacement and is an uncommon
cause of respiratory infections and community-acquired
pneumonia.
References:
1. Jawetz, Melnick and Adelberg's Medical Microbiology 23rd Ed

2004, McGraw-Hill Co, inc. p212-216.


2. CDC- VPD Surveillance manual 3rd Ed 2002, Chapter 1:
Diphtheria: p1-11; available at
www.cdc.gov/nip/publications/surv-manual/chpt01_dip.pdf
3. Emerg Infect Dis -MAY-2002; 8(5): 516-518.
4. SDJ Med-JUL-2000; 53(7): 281-285.
5. J Clin Micribiol-DEC-2002; 40(12): 4713-4719.
6. J Clin Microbiol-MAY-2006; 44(5): 1625-1629.
7. Br J Biomed Sci-2005-62 (1): 1-4.
8. Vet Res-MAR-APR-2006; 37 (2): 201-218.
9. NHS; National Standard Methods: Identification of
C.diphtheriae, C.ulcerans and C.pseudotuberculosis ;Issue no: 2
dated 3/2/06; page 1-13 available at www.hpastandardmethods.org.uk/documents/bsopid/pdf/bsopid2.pdf.10.Int
ernational - Journal of Systematic and Evolutionary Microbiology2001-51;1723-1728.
10. Nelson Textbook of Pediatrics: 16th ed. Philadelphia: W.B.
Saunders; 2000. p. 779-84.
11. MMWR Morb Mortal Wkly Rep 2003; 52:1285-86.
Question 59
1 out of 1 points

A 25-year-old woman comes to your office complaining that she always


seems to have watery eyes and a runny nose. These symptoms get worse
after she has been outside, especially if she plays in the grass with her 2year-old. Allergy testing reveals that she is highly allergic to several grass
pollens. You suggest that allergy shots, using grass pollens as the antigens,
might be helpful. What is the major principle behind the potential success of
allergy shots?
Answer
s:

The body makes antibodies to the injected antigen


Phagocytic cells are activated to directly remove antigens more
effectively

T cell receptors are activated to recognize native antigens


The injected antigen neutralizes existing antibodies
The specificity of existing antibodies is increased to react with the
injected antigen

Response Allergy shots are given to individuals that have an allergic


Feedback reaction to common allergens such as mold or pollen from
:
grasses, ragweed, and trees. A small amount of the allergen is
injected into the patient. The body starts making antibodies to
the allergen, and this allows the body to fight the allergen,
thereby relieving the symptoms of the allergic reaction. The
mature antibody molecule is composed of 4 polypeptide chains,
2 heavy chains, and 2 light chains. Generation of the mature
antibody molecule specific for a given antigen requires
rearrangement of immunoglobulin genes in the B cells. There is
a "pool" of gene segments that eventually must be brought
together to synthesize the mature antibody molecule. During B
cell development, a complete coding sequence for the 2 Ig
chains is assembled from the pool of gene fragments by a
process called site-specific genetic recombination. Site-specific
recombination alters the relative position of gene sequences in
the chromosome and requires specific enzymes. The gene
sequences needed to form the mature antibody chains are
brought together. Each light and heavy chain consists of
constant and variable regions and also contains 3 hypervariable
regions that are required to form the specific antigen-binding
site. A complete Ig chain can only be synthesized after this
genetic recombination occurs.
There is now compelling evidence that the microenvironment of
mucosal tissues of allergic disease favors class switching to lgE
through alternate splicing in the mRNA. The rate of local lgE
synthesis can easily compensate for the rate of the antibody
dissociation from its receptors on mucosal mast cells. Effective
mechanisms ensure that allergic reactions are confined to
mucosal tissues, thereby minimizing the risk of systemic
anaphlaxis.
Antibodies are only produced after the body comes in contact
with an antigen so the injected antigen does not neutralize
existing antibodies. Also, because of this the specificity of
existing antibodies cannot be increased to react with the
injected antigen. T cell receptors found on the surface of T

lymphocytes recognize antigens bound to major


histocompatibility complex molecules and not native antigens.
Question 60
1 out of 1 points

Case:
A 60-year-old man comes to the emergency room complaining of left foot
pain severe enough to make it difficult for him to walk. The patient also
complains of malaise, chills, and body aches. The patient's medical history is
significant for insulin-dependent diabetes since early childhood. He has a
body temperature of 39.8C and a slightly elevated blood pressure. A CBC is
performed and the results are significant for an elevated white blood cell
count with a left shift. The examination of the left foot is remarkable for
brown to black discoloration with crepitus upon palpation. There was also a
foul smelling thick drainage oozing from a lesion. The drainage is aspirated
for a stat Gram stain and culture. The Gram stain is significant for the
presence of gram negative rods and gram positive cocci. Surgical
debridement is scheduled in the OR and empiric antibiotic therapy is
initiated. The culture is positive for anaerobic gram-negative rods and
anaerobic cocci as well as aerobic gram-negative rods.
Question:
What is this patient's diagnosis?
Answers:

Bacteremic pseudomonas gangrenous cellulitis


Streptococcal gangrene
Synergistic necrotizing cellulitis
Necrotizing cutaneous mucormycosis
Clostridial myonecrosis (gas gangrene)

Response Synergistic necrotizing cellulitis is caused by a mixture of


Feedback organisms such as Bacteroides, Peptostreptococcus, Escherichia
:
coli, and other members of the Enterobacteriaceae. Diabetes is
the most common predisposing factor. Patients will present with
prominent pain at the infected area. The cellulitis produces a
thick, foul-smelling, copious "dishwater" smelling drainage.

Crepitus is often present. The patient will usually have moderate


fever with marked systemic toxicity. The clinical course is rapid.
Clostridial myonecrosis (gas gangrene) is an infectious gangrene
caused by Clostridium perfringens and occasionally other
clostridium species. Local trauma to the area is usually the
predisposing factor to this type of infection. Patients present
with prominent pain at the infected area, which has marked
swelling and a yellow-bronzed discoloration. There can also be
green-black patches of necrosis with a serosanguinous
discharge. Crepitus is present at the infected areas. The patient
generally will present with a moderate to high fever and a very
marked systemic toxicity. The clinical course is extremely
rapid. Clostridium perfringens is an anaerobic gram-positive rod
that is described as box-car shaped. Clostridium perfringens will
typically produce a double-zone of hemolysis on blood agar
media. A Gram stain of the serosanguinous discharge will be
positive for numerous gram-positive box-car shaped rods.
Bacteremic pseudomonas gangrenous cellulitis is caused
by Pseudomonas aeruginosa. Conditions predisposing
individuals to this type of cellulitis are burns and
immunosuppression. Patients will present with mild pain at the
infected area, which is a sharply demarcated necrotic area with
black eschar and surrounding erythema. There is no crepitus
present at the site. The patient will generally present with a high
fever and marked systemic toxicity. Clinical course is
rapid. Pseudomonas aeruginosa is an aerobic gram-negative
bacilli that is oxidase positive, grows on MacConkey agar media
as a nonlactose fermenter, and produces a blue-green pigment
with a sweet grape-like odor.
Streptococcal gangrene is caused primarily by Streptococcus
pyogenes. Conditions predisposing individuals to this type of
infection are diabetes and abdominal surgery. Patients will
present with prominent pain in the infected area, which will
contain necrosis of subcutaneous tissue and fascia. The area
may appear black or "burned" in appearance. Crepitus is absent.
Patients will present with high fevers and marked systemic
toxicity. The clinical course is very rapid. Streptococcus
pyogenes is an aerobic gram-positive cocci that will occur in
short to long chains. It is beta-hemolytic on blood agar, catalase
negative, and has a Lancefield grouping of "A."
is caused by fungal organisms. The most common causes of
necrotizing cutaneous mucormycosis are the rhizopus, mucor,
and absidia species. Diabetes and corticosteroid therapy are

predisposing conditions for acquiring necrotizing cutaneous


mucormycosis. Pain at the site of infection is minimal and the
site is described as a central black area with a purple raised
margin. The site can also present as just a black ulcer. The
patient will present with a low-grade fever, and systemic toxicity
can be variable. The clinical course is rapid.Necrotizing
cutaneous mucormycosis
Question 61
1 out of 1 points

Case:
A 24-year-old man is seen on morning rounds complaining of an extremely
painful wound. The patient had been hospitalized due to blunt traumatic
injuries sustained in a rock climbing accident. The patient appeared toxic
and had an elevated body temperature of 103F, and a respiration rate of
40/minute. The wound is a deep puncture wound acquired in the accident. A
visual examination of the wound is remarkable for a yellow-bronzed
discoloration with marked swelling. Palpitation of the wound is significant for
crepitus. Material is obtained for culture and stat Gram stain. The stat Gram
stain is significant (refer to the image). The patient is scheduled for
immediate debridement of the wound and placed on penicillin therapy. The
culture is subsequently reported as positive for 4+ of an anaerobic grampositive rod that produced a double zone of hemolysis on anaerobic blood
agar media.
Question:
What is the most likely diagnosis?

Answers:

Bacteremic pseudomonas gangrenous cellulitis


Streptococcal gangrene
Synergistic necrotizing cellulitis
Necrotizing cutaneous mucormycosis
Clostridial myonecrosis (gas gangrene)

Response Clostridial myonecrosis (gas gangrene) is an infectious gangrene


Feedback caused by Clostridium perfringens and occasionally other
:
clostridium species. Local trauma to the area is usually the
predisposing factor to this type of infection. Patients present
with prominent pain at the infected area, which has marked
swelling and a yellow-bronzed discoloration. There can also be
green-black patches of necrosis with a serosanguinous
discharge. Crepitus is present at the infected areas. The patient
generally will present with a moderate to high fever and a very
marked systemic toxicity. The clinical course is extremely
rapid. Clostridium perfringens is an anaerobic gram-positive rod
that is described as box-car shaped. Clostridium perfringenswill

typically produce a double-zone of hemolysis on blood agar


media. A Gram stain of the serosanguinous discharge will be
positive for numerous gram-positive box-car shaped rods.
Bacteremic pseudomonas gangrenous cellulitis is caused
by Pseudomonas aeruginosa. Conditions predisposing
individuals to this type of cellulitis are burns and
immunosuppression. Patients will present with mild pain at the
infected area, which is a sharply demarcated necrotic area with
black eschar and surrounding erythema. There is no crepitus
present at the site. The patient will generally present with a high
fever and marked systemic toxicity. Clinical course is
rapid. Pseudomonas aeruginosa is an aerobic gram-negative
bacilli that is oxidase positive, grows on MacConkey agar media
as a nonlactose fermenter, and produces a blue-green pigment
with a sweet grape-like odor.
Streptococcal gangrene is caused primarily by Streptococcus
pyogenes. Conditions predisposing individuals to this type of
infection are diabetes and abdominal surgery. Patients will
present with prominent pain in the infected area, which will
contain necrosis of subcutaneous tissue and fascia. The area
may appear black or "burned" in appearance. Crepitus is absent.
Patients will present with high fevers and marked systemic
toxicity. The clinical course is very rapid. Streptococcus
pyogenes is an aerobic gram-positive cocci that will occur in
short to long chains. It is beta-hemolytic on blood agar, catalase
negative, and has a Lancefield grouping of "A".
Synergistic necrotizing cellulitis is caused by a mixture of
organisms such as Bacteroides, Peptostreptococcus, Escherichia
coli, and other members of the Enterobacteriaceae. Diabetes is
the most common predisposing factor. Patients will present with
prominent pain at the infected area. The cellulitis produces a
thick, foul-smelling, copious "dishwater" smelling drainage.
Crepitus is often present. The patient will usually have moderate
fever with marked systemic toxicity. The clinical course is rapid.
is caused by fungal organisms. The most common causes of
necrotizing cutaneous mucormycosis are the rhizopus, mucor,
and absidia species. Diabetes and corticosteroid therapy are
predisposing conditions for acquiring necrotizing cutaneous
mucormycosis. Pain at the site of infection is minimal and the
site is described as a central black area with a purple raised
margin. The site can also present as just a black ulcer. The
patient will present with a low-grade fever, and systemic toxicity
can be variable. The clinical course is rapid.Necrotizing

cutaneous mucormycosis
Question 62
1 out of 1 points

Several pathogenic microorganisms possess surface structures called


capsules. Capsules act as one of the virulence factors of the organism. They
are known to protect the infecting pathogen from phagocytosis and attack of
anti-microbials. This structure can be readily demonstrated in wet films by
negative staining technique using India ink. The presence of a capsule is
often environmentally determined. Capsular substances of organisms may
vary in their chemical nature. Given below is a list of capsulated organisms.
Which of these possesses a polypeptide capsule?
Answers:

Klebsiella pneumoniae
Cryptococcus neoformans
Bacillus anthracis
Streptococcus pneumoniae
Neisseria meningitidis
Streptococcus pyogenes
Haemophilus influenzae

Respons
e
Feedback
:

Bacillus anthracis produces a polypeptide capsule. It is a


polymer of gamma-d-glutamic acid. The capsule is encoded on a
large plasmid pXO2. It is formed in vivo and contributes to
pathogenicity by enabling the bacilli to evade the host immune
defenses and thereby to provoke septicemia. It is very weakly
immunogenic and therefore does not favor an immune
response. M'Fadyean's polychrome methylene blue stain is
useful for demonstrating capsulated anthrax bacilli in fresh
smears of clinical specimens such as vesicular fluid, lymph node
aspirates, CSF and blood. The bacilli appear dark blue
surrounded by pink capsule. In vitro, capsule is formed when
anthrax bacilli are grown on bi-carbonate containing medium or
in the presence of 5-10% carbon dioxide. Encapsulated anthrax
bacilli produce mucoid colonies. In the context of developing

more effective anthrax vaccines, it is suggested that antibodies


to capsule may be critical in development of complete immunity
to anthrax exposure. The expression of B. anthracis capsule
biosynthesis genes (capBCAD) and regulation of capsule
synthesis are found to be highly dependent upon three
regulatory genes, atxA, located on the virulence plasmid pXO1,
and acpA and acpB, located on pXO2. The virulence plasmids of
B. anthracis are not self-transmissible, but can be transferred by
conjugative plasmids originating in a closely related species, B.
thuringiensis.
Klebsiella pneumoniae has a large capsule consisting of
polysaccharide (K antigen) and produce highly mucoid colonies.
Based on the K antigens, K. pneumoniae strains are classified
into several serotypes. There are different serotypes specifically
associated with upper respiratory tract infections and urinary
tract infections. Capsular polysaccharides of certain types cross
react with those of H. influenzae and S. pneumoniae.
Cryptococcus neofomans is a yeast which possesses a large
polysaccharide capsule. The organism causes meningitis,
pulmonary infections, and disseminated infections, especially in
immunocompromised patients.
Streptococcus pneumoniae (pneumococcus) possesses a
polysaccharide capsule. Antigenic specificity of the capsular
polysaccharide is used to classify streptococcus pneumoniae
into > 90 types. Neufeld reaction (capsule swelling reaction)
was first described in pneumococci by adding specific antiserum
to the capsulated organism. Mutational loss of enzymes in the
biosynthesis of capsular polysaccharide can result in the smooth
to rough variation. Pneumococcal vaccines contain capsular
antigens of most common pathogenic capsular types of the
bacterium.
Neisseria meningitidis produces a polysaccharide capsule. At
least 13 groups of meningococci are identified depending on the
antigenic specificity. Groups A, B, C, Y, and W-135 are the most
important groups causing meningitis. Groups A and C are often
associated with epidemics of meningitis. International outbreak
of meningococcal disease associated with Hajj pilgrimage was
caused by group W-135. Meningococcal vaccine containing
specific capsular polysaccharides of groups A, C, Y, and W-135 is
currently used for prophylaxis during epidemics. Group B
capsular substance is less immunogenic. Poor immunogenicity
has been attributed to its resemblance to host antigens. An
effective vaccine against group B is not yet available.

Streptococcus pyogenes capsule is composed of hyaluronic acid,


the same polymer as found in human connective tissue. It can
serve as an antigenic disguise that prevents recognition of the
streptococcus by phagocytes or immune system of the host.
Haemophilus influenzae has capsular polysaccharide capsule.
Based on the immunogenicity, six typesa, b, c, d, e and fare
recognized. Type b is an important agent of bacterial meningitis
in small children. H. influenzae strains, which are associated
with respiratory infections like chronic bronchitis, do not possess
capsule and are non-typeable.
Reference:
Ann Rev Microbiol-2001; 55: 647-671.
Jawetz, Melnick and Adelberg's Medical Microbiology, 23rd ed.,
McGraw-Hill, pp 31,231,235,251,279,301-303,647.
Mandell, Douglas, and Bennett's Principles and Practice of
Infectious Diseases, 6th ed., Churchill Livingstone, pp 2485,
2499- 2502.
Postgrad Med J - APR- 2002; 78 (918): 216 - 24.
Lancet 2000; 356: 2159.
WHO Weekly Epidemiol Rec - 2001; 76: 141-2.
Adv Drug Deliv Rev - JUN- 2005; 57 (9): 1266 - 92.
Curr Top Microbiol Immunol 2002; 271: 143-164.
J Bacteriology-2004-JAN- 186(2): 307- 315.
Question 63
1 out of 1 points

A 5-year-old boy is taken to a physician after his insect bite gets infected.
The doctor takes a specimen from the lesion for culture and a catalasepositive, Gram-positive bacteria growing in clusters is isolated. What else is
a feature of this organism?
Answers:

Can make spores


Produces gamma-hemolysis on blood agar
Can only grow anaerobically

Produces a potent endotoxin


Can ferment mannitol and grow on high salt

Response
Feedback:

The boy most likely presents with impetigo. The characteristics


of the bacteria suggest that it is Staphylococcus aureus. This
pathogen is known to grow on 7.5% salt and can ferment
mannitol.
S. aureus does not make spores.
S. aureus is usually beta-hemolytic, occasionally alphahemolytic, but not gamma-hemolytic.
S. aureus is a facultative anaerobe, not strict anaerobe.
Since this is a Gram positive pathogen, it does not produce
endotoxin (Lipopolysaccharide).
References:
Murray PR., Rosenthal KS., Pfaller MA. Medical Microbiology.
5th edition. Philadelphia, PA, Elsevier Inc, 2005.
Bernard P. Management of common bacterial infections of the
skin. Curr Opin Infect Dis. 2008 Apr;21(2):122-8.
Durupt F, Mayor L, Bes M, Reverdy ME, et al. Prevalence of
Staphylococcus aureus toxins and nasal carriage in furuncles
and impetigo. Br J Dermatol. 2007 Dec;157(6):1161-7. Epub
2007 Oct 4.

Question 64
1 out of 1 points

In the accompanying diagram of a bacterial cell envelope, identify the


structure marked 5.

Answers:

Nucleoid
Peptidoglycan layer
Outer membrane
Inner cell membrane
Mesosomes

Response Arrow 5 points to the outer membrane, which is the structure


Feedback found outside the peptidoglycan layer/cell wall. This structure is
:
anchored non-covalently to lipoprotein molecules (Braun's
lipoprotein), which in turn, are covalently linked to the
peptidoglycan. Like other membranes, the outer membrane is
composed of proteins and phospholipids. Unlike other
membranes, it contains an additional molecule,
lipopolysaccharide, which displays endotoxin activity. The
lipopolysaccharides of the Gram-negative cell envelope form
part of the outer leaflet of the outer membrane structure, while
the lipoprotein and the phospholipids form the inner leaflet of
the outer membrane bi-layer. The outer membrane possesses

several major outer membrane proteins; the most abundant is


called porin. A capsule is often seen extending beyond this
structure and contributes to bacterial virulence.
Peptidoglycan is a major component of the bacterial cell wall,
particularly in gram-positive species. These complex
carbohydrate polymers provide structural integrity to the cell
and are a major factor in the Gram stain reaction.
The inner cell membrane surrounds and contains the bacterial
cytosol.
Nucleoid refers to the bacterial DNA and associated proteins
that appear within discrete areas of the cytosol. Unlike
eukaryotic cell nuclei, the nucleoid is not membrane bound.
Mesosomes are invaginations of the plasma membrane/inner
cell membrane. These structures are much more prominent in
Gram-positive than in Gram-negative organisms.
References:
1. Murray, Third edition Chapter 4; Dr. Alvin Fox; CELL
ENVELOPE, SPORES AND MACROMOLECULAR BIOSYNTHESIS.
2. Todar's Online Textbook of Bacteriology: BACTERIAL
STRUCTURE IN RELATIONSHIP TO PATHOGENICITY; 2002
Kenneth Todar University of Wisconsin-Madison Department of
Bacteriology.
Question 65
1 out of 1 points

Case:
A 25-year-old woman who handled rodents in a pet shop was admitted to
the hospital with a 4-day history of fever, headache, and polyarthralgia
followed by maculo-papular cutaneous rashes on her extremities. She did
not recall any specific instance of animal bite. Blood cultures taken on
admission, after 72 hours of incubation showed flocculent puff ball-like
growth of long chains of Gram-negative bacilli. Subcultures on sheep blood
agar grew smooth gray colonies 1-2 mm diameter, of pleomorphic Gramnegative bacilli appearing as chains and non-branching filaments with beadlike swellings. The organism was catalase, oxidase, urease, and indole
negative.
Question:

What is the most likely isolate?


Answers:

Borrelia burgdorferi
Fransicella tularensis
Pasteurella multocida
Spirillum minus
Streptobacillus moniliformis

Respons
e
Feedbac
k:

Among the listed bacteria Streptobacillus moniliformis is the


only one that possesses the characteristics of the isolate. As the
name indicates the bacilli are arranged in chains, and the
filaments with bead-like swellings give a necklace-like
(moniliformis) appearance. Streptobacillus moniliformis is one
of the 2 causative agents of rat bite fever (RBF), the other
being Spirillum minus.
The patient had the risk of occupational exposure to rats and
other rodents and presented with typical clinical manifestations
of RBF. While handling animals in the pet shop, minor scratches
and bites may be commonplace and may not be recalled.
Streptobacillus moniliformis is a natural parasite of rodents. It is
found in the nasopharynx and oropharynx of small rodents,
especially rats. Rats are carriers; they do not show any
symptoms. RBF may result after a bite or scratch from a carrier
animal. Infections due to ingestion of water or food
contaminated with rat excreta have also been reported.
Epidemics of water and milk borne infections have been
documented. This form of RBF is known as Haverhill fever.
S.moniliformis infection can cause clinical illness without the
classic signs and symptoms of RBF. The bacterium may produce
septic arthritis or polyarthritis mimicking rheumatoid arthritis.
Fever and rashes may be absent.
Manifestations such as pericarditis, endocarditis, myocarditis,
meningitis, pneumonia, and focal metastatic abscesses have
been reported. Immunocompromised persons are more
susceptible to such complications. Rare cases of fatal fulminant
sepsis caused by S.moniliformis have been reported in
previously healthy adults also.

Laboratory diagnosis includes direct microscopic detection of


pleomorphic gram-negative bacilli in smears of blood or joint
fluid or skin lesions stained by Giemsa, Wayson, or Gram's
method, and cultural isolation of S.moniliformis from the clinical
specimens. The organism is fastidious and isolation is not
always easy. Sodium polyanethol sulfonate, an additive used in
many commercial blood culture bottles for inhibiting the
antibacterial activity of human blood may impede the growth
ofS.moniliformis. It has been observed that non-specific initial
clinical presentation and difficulties in culturing may sometimes
lead to misdiagnosis of RBF.
The bacterium is a non-motile highly pleomorphic gramnegative rod, growing as tangled chains with beaded or
fusiform swellings. It readily develops into cell wall-deficient Lforms.
New methods have been developed and found helpful for
identifying S.moniliformis. These include gas-liquid
chromatography analysis of the cellular fatty profile of the
isolate, and molecular methods like 16S rRNA gene sequencing
and polymerase chain reaction (PCR) amplification. PCR can be
used for detection of S.moniliformis DNA in clinical samples
from patients and also for screening saliva of rats for the
organism. Molecular methods can be used to identify the source
of infection even in the absence of known rat bite.
Enzyme-linked immunosorbent assay (ELISA) test is used for
demonstration of specific antibodies.
S.moniliformis is susceptible to a variety of antibiotics including
penicillin. Treatment with penicillin G is recommended for
streptobacillary RBF. For those allergic to penicillin, tetracycline
and streptomycin are alternatives. Mortality of untreated RBF is
reported as 10-13%.
Spirillum minus: This causes a type of RBF called Sodoku, seen
mostly outside the U.S. The bacterium is short, with 2 or 3
regular spirals, is actively motile with amphitrichous flagella
and does not grow on laboratory media. It is gram-negative, but
better visualized by Giemsa or silver stains. RBF cause is
associated with suppuration of rat bite wound, regional
lymphadenopathy, and fever of relapsing type.
Pasteurella multocida is a gram-negative, non-motile bacillus,
catalase, oxidase, and indole positive. This bacterium is carried
in the upper respiratory tract of a variety of animals, dogs, cats,
rats, cattle, and sheep. Human infections are rare. Infection

may occur following animal bites and may cause local


suppuration, meningitis, respiratory tract infections,
appendicitis, appendicular abscess, or endocarditis.
Francisella tularensis is a minute capsulated fastidious gramnegative bacillus 0.3-0.7micronsx0.2microns, in size. The
bacterium causes a serious and occasionally fatal zoonotic
disease tularemia. It is considered a bioterrorism agent.
Infection may be transmitted directly from the reservoir hosts
or by arthropod vectors. Clinical presentation depends on the
route of infection. Ulceroglandular, typhoidal, oropharyngeal or
gastrointestinal, and pneumonic forms can occur. Pneumonic
form results from inhalation of the bacteria and is rare. But this
is the likely form if F.tularensis is used as abioterrorism agent. A
live attenuated vaccine is available for immunization of persons
at high risk.
Borrelia burgdorferi is a spirochete 20-30 microns x 2-3 microns
in size. It causes Lyme borreliosis that is transmitted from
infected animals by ticks. Early manifestations are local lesion
at the site of tick bite called erythema migrans associated with
flu-like symptoms and most common late manifestations are
arthralgia and arthritis.
References:
Jawetz, Melnick & Adelberg's medical microbiology 23rd Ed
2004; McGraw-Hill company Inc; pp 275-276.
Netherlands Journal of Medicine-SEPT-2005; 63(8): 319-321.
Ann Rheum Dis-2003; 62:894-896.
BMC Infectious Diseases-JUNE 11-2007; 7:56.
Emerg Infect Dis-JUNE- 2006;12 (6): 1037-1038.
MMWR Morbidity and Mortality Report (CDC)-JAN 7-2005;
53(51): 1198-1202.
Journal of Clinical Pathology-NOV-2005; 58(11): 1215-1216.
MMWR Morbidity and Mortality Weekly Report-FEB 25-2005;
54(7): 170-173.
Emerg Infect Dis-DEC-2005; 11(12): 1835-1841.
Mandell, Douglas & Bennett's Principles and Practice of
Infectious Diseases 6th Ed 2005, copyright Elsevier Inc:
pp2708-2710

Elliott SP. Rat bite fever and Streptobacillus moniliformis. Clin


Microbiol Rev. 2007 Jan;20(1):13-22.
Question 66
1 out of 1 points

The activation of individual cells in an immune response often requires at least


2 independent signals provided by neighboring cells. B cell activation, in
particular, is usually carried out by a previously activated T cell. In some
individuals, B cells cannot be properly stimulated because of a deficiency in
CD40 signaling. Which of the following describes the phenotypes of an
individual with this syndrome?
Choice

IgM

IgG

normal

normal

normal

Answers:

A
B
C
D
E

Respons
e
Feedbac

Hyper-IgM syndrome is characterized by normal or elevated serum


concentrations of IgM and IgD, and absent or very low serum
concentrations of IgG, IgA, and IgE (this is due to a lack of class

k:

switching). There is also normal or increased number of B cells and


normal number and distribution of CD4+ and CD8+ T-cell subsets.
The option "A" lack of all isotypes would be seen in Brutton's
agammaglobulinemia.
The option B would be seen in Wiskott-Aldrich disease.
The option "C" does not represent a clinical disorder.
An increase in only a single isotype such as IgA may be seen in
multiple myeloma.
References:
1. Parham, P. The Immune system. 2nd ed., New York, NY: Garland
Science, 2005
2. Melegari A, Mascia MT, Sandri G, et al. Immunodeficiency and
autoimmune phenomena in female hyper-IgM syndrome. Ann N Y
Acad Sci. 2007 Aug;1109:106-8.
3. Nagaraj N, Egwim C, Adler DG. X-linked hyper-IgM syndrome
associated with poorly differentiated neuroendocrine tumor
presenting as obstructive jaundice secondary to extensive
adenopathy. Dig Dis Sci. 2007 Sep;52(9):2312-6. Epub 2007 Apr 12.
Question 67

1 out of 1 points

The rash produced by poison ivy, is caused by a T cell response to a


chemical in the poison ivy leaf. As the chemical is delivered by contact with
the skin, this is called ""contact dermatitis"". An important test in
determining which antigen is responsible for contact dermatitis is
Answers:
Patch test
Isoelectric focusing
SDS-PAGE
ELISA
FACS

Response Allergic contact dermatitis is an example of delayed-type


Feedback hypersensitivity, or type IV hypersensitivity reactions. These
:
responses are caused by specific effector T cells. The agents
causing the reaction are usually of low molecular weight, and
are not immunogenic in their own right; instead they are highly
reactive molecules that bind covalently to skin or tissue
proteins.
Some antigens are readily absorbed when applied directly to the
skin in the patch test. Others are poorly absorbed, and have to
be applied on cellulose discs, taped to the skin. The choice of
antigens is dependent upon the patient's history of exposure.
Many patients are sensitive to several antigens. The time taken
for a positive reaction to develop, depends on the rate of
absorption, and can vary from two to seven days. A positive
result is indicated by a red, edematous, itchy, and indurated
lesion in the area of contact.
Isoelectric focusing or IEF is an electrophoretic method for the
separation of amphoteric molecules, according to their
isoelectric points, in a stabilized pH gradient. It can be
described as electrophoresis in a pH gradient, set up between a
cathode and an anode, with the cathode at a higher pH than the
anode. Under the influence of the electrical force, the pH
gradient is established and the proteins migrate and focus
(concentrate) at their isoelectric points.
Separation of proteins on the basis of size, or molecular weight,
is frequently accomplished with SDS-PAGE, or polyacrylamide
gel electrophoresis, in the presence of a detergent called SDS.
SDS binds relatively homogeneously to proteins, conferring a
charge that allows the electrophoretic field to drive protein
migration through the gel.
Enzyme-linked immunosorbent assay or ELISA is a direct binding
assay to detect the presence of a specific antibody, or to
measure its quantity in a sample. An antigen or a specific
antibody can be detected by a reaction that converts a colorless
substrate into a colored reaction product.
A powerful tool for defining and counting lymphocytes is the
fluorescence-activated cell sorter (FACS). This instrument is
used to study the properties of cell subsets identified using
monoclonal antibodies to cell-surface proteins.
Question 68

1 out of 1 points

Researchers have found a link between human type II diabetes and the gene
for a protein kinase by breeding mice that lacked the gene for the kinase.
This is accomplished using what technique?
Answers:

Transgenesis
Gene duplication
Recombination
Molecular mimicry
Gene conversion

Respons
e
Feedback
:

The functions of genes have been studied by observing the


effects of spontaneous mutations in whole organisms and, more
recently, by analyzing targeted mutations in cultured cells. The
advent of gene cloning and in vitro mutagenesis makes it
possible to produce specific mutations in whole animals. Mice
such as those in the case mentioned above are termed
knockout mice because a piece of their DNA has been turned
off, or knocked out, via replacement or disruption using
artificial or non-mouse DNA in the process of recombination.
This occurs as the mouses DNA incorporates the new DNA into
its current DNA. Recombination can create new alleles by
reassorting discrete polymorphic regions.
Mice with altered copies of a gene in their genome can now be
generated by transgenesis. Transgenic animals can be produced
using DNA microinjection, retrovirus-mediated gene transfer, or
embryonic stem cell-mediated gene transfer. Transgenesis
allows scientists to study the impact of a newly discovered
gene.
Gene duplication is the creation of an extra copy of a gene. This
is a key mechanism in evolution. Once a gene is duplicated, the
identical genes can undergo changes and diverge to create 2
different genes.
If a microbial antigen is very similar to normal host antigens, the
immune response to this antigen may be weak or absent, giving
a degree of tolerance. The mimicking of host antigens by

microbial antigens is referred to as molecular mimicry.


Some new alleles are the result of point mutations but many
arise by gene conversion, a process in which one sequence is
replaced in part by another from a homologous gene.
Question 69
1 out of 1 points

The use of recombinant DNA to introduce new genes into organisms is a


significant part of the revolution in molecular biology. This transfer can be
mediated by a retrovirus or embryonic stem cells. What is the process of
incorporation of jumping gene into a genome of an organism?

Answers:

Transgenesis
Antigenic shift
Antigenic drift
Gene fusion
Transposon mutagenesis

Respons
e
Feedbac
k:

Transposon mutagenesis involves the insertion of a transposon


into a gene. This inactivates the host gene, leading to a mutant
phenotype and also confers the phenotype associated with the
transposon gene. Transposons are mobile genetic elements that
move about in the genome, either removing themselves to a
new location or duplicating themselves for insertion elsewhere.
Transposon insertion causes gene disruption in the organism.
The pseudonyms for transposons include mobile elements,
jumping genes, and genomic parasite. Transposons also occur
naturally in the bacterial genome. Transposons generally
demonstrate vertical transmission, i.e. the new incorporations
are inheritable by the offspring. Transposons are useful in the
cloning of genes, production of transgenic organisms, and in the
identification of regulatory elements.
To produce transgenic animals, a cloned gene is introduced into
the animal genome by microinjection into the pronucleus of a

fertilized egg, which is then implanted into the uterus of a


pseudopregnant female. In some of the eggs, the injected DNA
becomes integrated randomly into the genome, giving rise to an
animal that has an extra genetic element of known structure,
the transgene. This technique allows one to study the impact of
a newly discovered gene on development, to identify the
regulatory regions of a gene required for its normal tissuespecific expression, to determine the effects of its over
expression or expression in inappropriate tissues, and to find out
the impact of mutations on gene function.
Antigenic shift is the appearance of a new assortment of genes.
This type of re-assortment happens rarely and can be
devastating, since shift produces a virus strain with a
combination of surface antigen proteins to which the human
population will have little or no immunity. Antigenic shift is
believed to be the explanation for the occurrence of strains of
microorganisms, such as the influenza virus associated with
large scale epidemics.
Antigenic drift is the change that occurs at the molecular level to
effect a change in the antigenicity of a bacteria or
virus. Antigenic drift is caused by point mutations in the genes
encoding for hemagglutinin and neuraminidase of the influenza
virus. Every 2 to 3 years, a variant arises with mutations that
allow the virus to evade neutralization by antibodies in the
population. As new variants appear, they replace the previous
variant so that within about 4 years a given individual can be reinfected with an antigenic variant that has been gradually
generated by infection of other individuals. This results in local
epidemics.
Gene fusion is the process of formation of a hybrid gene by
translocation, interstitial deletion, or chromosomal inversion.
References:
1. Kidwell, M.G. (2005). Transposable elements. In
http://www.answers.com/main/ntquery - The Evolution of the
Genome (ed. T.R. Gregory), pp. 165-221. Elsevier, San Diego.
2. http://www.ncbi.nlm.nih.gov/entrez/query.fcgi, 2005 Jan;
137(1):168-75. Epub 2004 Dec 23.
3. Andras Nagy, Norbert Perrimon, Suzanne Sandmeyer & Ronald
Plasterk;Tailoring the genome: the power of genetic
approaches; Nature Genetics 33, 276 - 284 (2003.)
4. Heikki Vilen, Juha-Matti Aalto, Anna Kassinen, Lars Paulin and

Harri Savilahti A; Direct Transposon Insertion Tool for Modification


and Functional Analysis of Viral Genomes; Journal of Virology,
January 2003, p. 123-134, Vol. 77, No. 1.
Question 70
1 out of 1 points

There are infections in which antibodies directed against the infectious agent
react with normal host cells. For example, in atypical pneumonia caused by
Mycoplasma pneumoniae, antibodies to heart, lung, brain, and red blood cells
may be formed. The antibodies to red blood cells (called cold agglutinins) can
cause autoimmune hemolytic anemia. What is this phenomenon called?
Answers:

Anergy
Apoptosis
Molecular pathogenesis
Molecular mimicry
Antigenic variation

Re
sp
on
se
Fe
ed
ba
ck:

In molecular mimicry, an antigen of a microorganism resembles a selfmolecule and can induce a cross-reactive autoimmune response. Several
autoimmune diseases occur after specific infections and are thought to be
triggered by the infection. Some examples are shown in the table.
Infection
Group A Streptococcus
Chlamydia trachomatis
Salmonella typhimurium Shigella flexneri Campylobacter jejuni
Borrelia burgdorferi

Disease susceptibility in these cases is determined by MHC genotype.


Other environmental initiators of autoimmunity can act like infections by
causing tissue damage, such as sunlight in lupus erythematosus.
Anergy is a state of unstable metabolic arrest affecting lymphocytes that
can lead to apoptosis. It occurs when a lymphocyte receives an antigenic
signal without the normally necessary co-stimulatory second signal.
Apoptosis represents physiological as opposed to pathological (necrotic)
cell death - it is the cellular equivalent of suicide. Apoptotic cells are
recognized by phagocytes and removed.
Most infectious agents show a significant degree of host specificity,
causing disease only in one or a few related species. The molecular
mechanism of host specificity is an area of research interest known
as molecular pathogenesis.
One way in which an infectious agent can evade immune surveillance is
by altering its antigens; this is particularly important for extracellular
pathogens, against which the principal defense is the production of
antibody against their surface structures.Antigenic variation can allow
pathogens to escape form the immune response.
Question 71
1 out of 1 points

Many bacteria cause disease by secreting proteins called bacterial toxins,


which damage or disrupt the function of the host's cells. Most toxins are
active at nanomolar concentrations; for example, a single molecule of
diphtheria toxin can kill a cell. To have an effect, a toxin must interact with a
specific molecule that serves as a receptor on the surface of the target cell.
The erythrogenic/pyrogenic toxins A and C of Streptococcus pyogenes,
staphylococcal enterotoxins, and staphylococcal toxic shock syndrome toxin
are examples of what?
Answers:

Immunotoxins
Superantigens
Immunophilins
Toxoids
RTX toxins

Response Superantigens are a distinct class of antigens that have a


Feedback special mode of binding that enables them to stimulate very
:
large numbers of T cells. They bind to the MHC class II
molecules, outside the antigen-binding groove, on antigenpresenting cells, and then bind the T cell receptor. This mode of
stimulation does not prime an adaptive immune response
specific for the pathogen. Instead it causes a massive
production of pro-inflammatory cytokines by CD4 T cells. These
cytokines have two effects on the host: systemic
toxicity and suppression of the adaptive immune response. Both
of these effects contribute to microbial pathogenicity.
Linking a tumor-specific or tumor-selective monoclonal antibody
to a toxin is a way to create an immunotoxin, a reagent that can
by used to destroy all or part of a tumor. When the labeled
antibody is internalized, the toxin is cleaved from the antibody
in the cell's endocytic compartment, allowing the toxin to
penetrate and kill the cell.
The immunophilins are a family of intracellular proteins that
bind to certain compounds forming complexes that interfere
with signaling pathways important for the clonal expansion of
lymphocytes.
Diphtheria and tetanus toxins are among the bacterial toxins in
which the toxic and the receptor-binding functions of the
molecule are on two separate chains. Toxoids are modified toxin
molecules that lack toxic activity but retain the receptor-binding
site, so that immunization with the toxoid induces neutralizing
antibodies effective in protecting against the native toxin.
One group of toxins has been designated RTX toxins (repeats in
toxin) by virtue of a common structural feature - the presence of
an array of a nine amino acid repeat to which calcium binds
thereby activating the toxins which form membrane pores of
varying sizes. They constitute the largest group of bacterial
pore-forming toxins and are widespread among Gram-negative
pathogens.
Question 72
1 out of 1 points

The interaction of self-molecules and immature B cells is important in the


development of self-tolerance in the B cell lineage: cells with potential
reactivity to self are prevented from responding. This can happen in two
ways. If an immature B cell is exposed to a self-molecule expressed on the
surface of bone marrow cells, it dies by apoptosis. In contrast, if the
immature B cell is exposed to a non-cell surface molecule (soluble antigen)
in the bone marrow, what does the cell go into?
Answers:

Apoptosis
Anergy
Latency
Persistence
Tolerance

Response When soluble antigen binds an immature B cell, the cell


Feedback is inactivated but not killed. This state of non-reactivity is
:
called anergy, and such B cells are defined as anergic. Anergy is
a state of unstable metabolic arrest that can lead to apoptosis.
The inactivation of immature B cells with potential reactivity to
self is also known as negative selection.
Apoptosis represents physiological as opposed to pathological
(necrotic) cell death - it is the cellular equivalent of suicide.
Apoptotic cells are recognized by phagocytes and removed.
Some viruses can enter a state known as latency in which the
virus is not being replicated. In the latent state, the virus does
not cause disease, but because there are no viral peptides to
signal its presence, the virus is not eliminated. Latent infections
can be reactivated.
Persistence is the state where viruses can be found continuously
in the individual, and they may be shed more or less
continuously, often for many years, without causing further
disease. After recovery from typhoid, for instance, bacteria
sometimes persist for long periods in the gall bladder.
is an immunologically specific reduction in the immune
response to a given antigen. It can be due to a primary lack of
responsiveness or to an active suppression of immune
response.Tolerance

Question 73
1 out of 1 points

Case:
A 33-year-old man presents with a wound on his left hand; 3 days before, the
patient obtained the wound from a human bite during a fight in a bar and
now has increasing pain and redness. Examination of the hand reveals
inflammation and erythema encompassing the entire hand. The wound itself
has a dark peripheral border with purulent exudates; they are visually
apparent. An aspirate of deep purulent material is collected for Gram stain
and culture. The wound is cleaned and treated with an antiseptic. The
patient is given ciprofloxacin as a course of treatment, and he is given
instructions to call if the symptoms worsen. The culture Gram stain is
significant for many white blood cells and many Gram-negative rods (see
image 1b). The culture is positive for growth of an organism that only grows
on chocolate agar media. The colonies are approximately 1 to 2 mm in
diameter after 48 hours at 37C, and they have clear centers surrounded by
spreading growth. The colonies form pitting of the agar surface and smell
strongly of hypochlorite. Several days later, the colonies begin to assume a
very light yellow hue.
Question:
The patient has a wound infection due to what organism?

Answers:

Haemophilus influenza
Pasteurella multocida
Moraxella catarrhalis
Vibrio parahaemolyticus
Eikenella corrodens

Respons
e
Feedbac
k:

Eikenella corrodens, which was formerly called Bacteroides


ureolyticus, is a facultative anaerobic, Gram-negative bacterium;
it is a normally occurring inhabitant of the oral cavity, and it is
often one of the pathogens associated with infected human bite
wounds. Characteristically, the organism will pit the agar and
produce a strong hypochlorite odor. It is a member of the HACEK
group, which includes organisms such as Haemophilus
aphrophilus, Actinobacillus
actinomycetemcomitans,Cardiobacterium hominis, Eikenella
corrodens, and Kingella kingae; along with other organisms in the
group, it can be a cause of infective endocarditis.
Haemophilus influenza is a facultative anaerobic, Gram-negative
coccobacillus that requires the presence of X factor (hemin) and
V factor (nicotinamide adenine dinucleotide [NAD]) for growth.
For this reason, the organism will only grow on chocolate agar
media, which is made with blood that has been heated to induce
lysing of the red blood cells and in turn releases the needed
factors into the agar media. The organism has a characteristic
musty smell on chocolate agar media.
Pasteurella multocida is a Gram-negative, oxidase-positive rod
that will characteristically grow on blood agar plate (BAP) and
chocolate agar, but not on MacConkey agar. The organism is part
of the normal oral flora of dogs and cats, and it is associated with
cat and dog bite wound infections. The organism is commonly
mistaken for Haemophilus species, but it is easily distinguished
by its ability to grow on BAP and chocolate agar
media; Haemophilus species will only grow on chocolate agar
media.
Moraxella catarrhalis, which was formerly called Branhamella
catarrhalis, causes acute, localized infections such as otitis

media, sinusitis, and bronchopneumonia as well as lifethreatening systemic diseases including endocarditis and
meningitis. The organism is a Gram-negative diplococci; it is
oxidase positive, has a slight pink pigment, is catalase positive,
and is usually positive for the production of the beta-lactamase
enzyme. The organism characteristically produces butyrate
esterase, and the detection of this enzyme will provide a rapid
identification.
Vibrio vulnificus normally lives in warm seawater and is part of a
group of vibrios that are called "halophilic" because they require
salt. It is a Gram-negative rod that is oxidase positive and motile
by means of polar flagella. Characteristically, it is lactose
positive. Infections with the organism are generally due to
ingestion of raw oysters and/or exposure of traumatic wounds to
infected marine animals or contaminated water. TCBS (thiosulfate
citrate bile salts sucrose) agar is the agar media of choice in
isolating the organism.

References:
Kristine M. Bisgard, DVM, MPH; Sue Bath, MPH; Pam Srivastava,
MS; Margaret Cortese, MD Chapter 2: Haemophilus influenzae
type b Invasive Disease, VPD Surveillance Manual, 3 rd Edition,
2002,Chapter 2, Haemophilus influenzae type b: 2 - 1
Peel MM. Dog-associated bacterial infections in humans: isolates
submitted to an Australian reference laboratory, 1981-1992.
Pathology 1993;25:379-84.
Talan DA, Citron DM, Abrahamian FM, Moran GJ, Goldstein EJ.
Bacteriologic analysis of infected dog and cat bites. N Engl J Med
1999;340:85-92.
http://www.cdc.gov/std/Gonorrhea/lab/Mcat.htm, Moraxella
catarrhalis.
http://www.cdc.gov/ncidod/dbmd/diseaseinfo/vibriovulnificus_g.ht
m, Vibrio vulnificus
Talan DA, Abrahamian FM, Moran GJ, et al. Clinical presentation
and bacteriologic analysis of infected human bites in patients
presenting to emergency departments. Clin Infect Dis. 2003 Dec
1;37(11):1481-9. Epub 2003 Nov 7.
Millar BC, Moore JE. Emerging issues in infective endocarditis.
Emerg Infect Dis [serial on the Internet]. 2004 Jun [date cited].
Available from: http://www.cdc.gov/ncidod/EID/vol10no6/03-

0848.htm.
Sheng WS, Hsueh PR, Hung CC, et al. Clinical features of patients
with invasive Eikenella corrodens infections and microbiological
characteristics of the causative isolates. Eur J Clin Microbiol Infect
Dis. 2001 Apr;20(4):231-6.
Udaka T, Hiraki N, Shiomori T, et al. Eikenella corrodens in head
and neck infections. J Infect. 2007 Apr;54(4):343-8. Epub 2006
Sep 7.
Question 74
1 out of 1 points

Active immunity is acquired when exposure to an immunogenic stimulus


triggers an immune response. Artificial active immunization involves the
deliberate administration of an immunogen in the form of a vaccine.
However, many purified or synthetic antigenic determinants show poor
immunogenicity. Combinations of antigenic subunits and appropriate
immunostimulatory compounds may provide a safe and effective vaccine.
The immunostimulatory compounds are known as
Answers:
Adjuvants
Immunotoxins
Streptolysins
Immunophilins
Endotoxins

Response Even conjugate vaccines are not usually strongly immunogenic


Feedback on their own; most require the addition of adjuvants, which are
:
substances that enhance the immunogenicity of antigens. Some
commonly used adjuvants are listed.
Oil Adjuvants
Freund's complete adjuvant and incomplete adjuvant
Mineral Salts
Aluminum phosphate or hydroxide, calcium alginate
Double-stranded nucleic acids

Poly (IC), Poly (AU)


Other substances
Saponin, levamisole
In humans, the most widely used adjuvants are aluminum
compounds (alums). These form a precipitate with protein
antigens and result in slow release of the antigen. Alums are
present in vaccines such as tetanus toxoid and diphtheria
toxoid.Biodegradable polymers can be used as delayed-release
capsules, dissolving weeks after injection to release a booster
dose of antigen. A glycoside adjuvant called Quil A extracted
from the bark of an Amazonian oak tree readily forms micelles
and can be used to make an immunostimulatory complex
containing immunizing peptides with an built-in adjuvant.
Linking a tumor-specific or tumor-selective monoclonal antibody
to a toxin is a way to create an immunotoxin, a reagent that can
by used to destroy all or part of a tumor. When the labeled
antibody is internalized, the toxin is cleaved from the antibody
in the cell's endocytic compartment, allowing the toxin to
penetrate and kill the cell.
Certain groups of streptococci secrete a variety of streptolysins
or hemolysins that lyse red blood cells and are much more
active weight for weight than hemolysins such as bile salts or
saponin, but which have a more important toxic action on
polymorphs and macrophages. Various hemolysins are released
also by pathogenic staphylococci, and these can kill phagocytes.
The immunophilins are a family of intracellular proteins that
bind to certain compounds forming complexes that interfere
with signaling pathways important for the clonal expansion of
lymphocytes.
are part of the outer membrane of Gram-negative bacteria.
Some of the diseases in which endotoxins may play a part are
typhoid fever, tularemia, plague and brucellosis, and a variety of
hospital-acquired infections caused by opportunistic Gramnegative pathogens. Endotoxin is a complex lipopolysaccharide
that displays a large array of biological effects.Endotoxins
Question 75
1 out of 1 points

Case:
A 67-year-old man was treated for bacterial pneumonia and prescribed

erythromycin for treatment. He presents a week later with severe diarrhea,


lower abdominal cramping, fever, nausea, and dehydration. Stool
examination and enzyme immunoassay of the patient's serum indicate
infection by Clostridium difficile, which is a Gram-positive bacilli. The patient
is prescribed metronidazole for the infection.
Question:
What component of this infection is the cause of the symptoms in the
patient?
Answers:

Capsule
Cell wall
Flagella
Spores
Toxins

Response Toxins released by the organism are responsible for the patient's
Feedback symptoms.
:
The symptoms of a C. difficile infection are due to the release
of toxins produced by the organism into the body of humans. C.
difficile produces 2 types of toxins, A and B; they enter the
intestinal mucosal cells via specific receptors on their cell wall
and result in inflammation of the colon, mucosal damage, and
fluid and mucous secretions. These changes cause diarrhea.
Toxin A has enterotoxicity and toxin B has cytotoxicity. A strain
of C. difficile, NAP 1, is found to produce another type of toxin
known as binary toxin. Along with toxins, colonization of the
organism is also found to be an essential factor in the onset of
symptoms.
The cell wall of C. difficile does not disrupt the intestinal
mucosal cells. The cell wall of the organism helps in adhering to
the mucosal cells at specific sites and in colonization of the
organism at the site of adherence. The organisms have various
proteins on their cell wall that function as adhesins and attach
the organism at particular sites on the surface of mucosal cells.
The proteins of the cell wall may help the organism in invading
the cells. These proteins can be targeted when producing
vaccines against C. difficile.

Capsules are generally not produced by Clostridium species, but


a capsule-like substance has been found around C. difficile cells.
The capsules were found to be composed of glycocalyx and
were detected by antibody stabilization methods. These
structures protect the organism from phagocytosis during
infection and help in colonizing on the mucosal layer of the
intestine.
A few strains of C. difficile are known to have flagella, which
help the cells in motility. In this organism the flagella are also
known to possess virulence factors, which result in production of
antibodies against them in infected humans. The flagella are
found to help the organisms in adhering to the cells of the
intestinal mucosa. The flagella of C. difficile are also found to be
involved in cross-reactions among serogroups.
Spores produced by Gram-positive bacteria are resistant to
adverse conditions like high temperature, antibiotics,
disinfectants, etc. These spores can survive in an environment
for months or years without germination. C. difficile is a major
source of nosocomial infections pertaining to its spores. The
spores enter the human body when inhaled or through the oral
route. They then geminate in the small intestine, thereby
producing viable cells of C. difficile and starting the cycle of
infection.

References:
Anne-Judith Waligora, Claire Hennequin, Peter Mullany et al.,
Characterization of a Cell Surface Protein of Clostridium
difficile with Adhesive Properties. Infect Immun. 2001 April,
69(4):2144-53.
Albert Tasteyre, Tuomo Karjalainen, Vronique Avesani et al.,
Phenotypic and Genotypic Diversity of the Flagellin Gene (fliC)
among Clostridium difficile Isolates from Different Serogroups. J
Clin Microbiol, 2000 September, 38(9): 3179-86.
Rebecca H. Sunenshine and L. Clifford McDonald, Clostridium
difficile-associated disease: New challenges from an established
pathogen. Cleve Clinic J Med, 2006 February, 73(2):187-97.
Kuehne SA, Cartman ST, Heap JT, Kelly ML, Cockayne A, Minton
NP. The role of toxin A and toxin B in Clostridium difficile
infection. Nature. 2010 Sep 15.
Yoo J, Lightner AL. Clostridium difficile Infections: What Every

Clinician Should Know. Perm J. 2010 Summer;14(2):35-40.


Question 76
1 out of 1 points

Case:
A 20-year-old woman on the orthopedic floor develops redness and
inflammation of a knee that had ACL repair done. The knee is inflamed and
tender to the touch. The surgical sites are significant for a white, pus-like
drainage. There is no significant odor present, which would be indicative of a
possible anaerobic infection. Drainage material was collected for Gram stain
and culture. The Gram stain result report had many neutrophils present and
many Gram-positive cocci. The culture grew colonies on blood agar that
were beta-hemolytic (refer to the image) and catalase/coagulase-positive.
Question:
This patient has what infection?

Answers:

Escherichia coli
Pseudomonas aeruginosa
Streptococcus pyogenes
Staphylococcus aureus
Streptococcus pneumoniae

Response Staphylococcus aureus is the most common Gram-positive cocci


Feedback to cause clinical disease in humans. The organism is a common
:
cause of surgical wound infections acquired in a hospital. It can
also cause pneumonia, sepsis, peritonitis, sore throats, and food
poisoning. S aureus is a Gram-positive cocci appearing clusters,
and it is catalase and coagulase-positive.
Escherichia coli is a Gram-negative bacillus and is a member of
the Enterobacteriaceae group. It is the number 1 significant
clinical isolate in the microbiology laboratory. The organism is
90% lactose-positive and thus grows pink on MacConkey agar, is
bile-positive on MacConkey agar, and is indole-positive and
oxidase-negative. The organism is found to cause a variety of
infections such as urinary tract infections, septicemia, and
wound infections.
Pseudomonas aeruginosa is a lactose-negative bacillus (has no
pink color on MacConkey) that is indole-negative and oxidasepositive. It is a non-fermenter and not a member of the
Enterobacteriaceae group. On MacConkey, it frequently has a
slight blue-green color and is said to have a grape-like odor. The
organism is a common cause of nosocomial infections and is
usually multiply resistant to a variety of antibiotics. It can cause
eye infections, urinary tract infections, wound infections,
septicemia, and ear infections.
Streptococcus pyogenes is a Gram-positive cocci that appears in
pairs or chains under the microscope. The organism is betahemolytic on blood agar, catalase-negative, and is Lancefield
grouped as A. The organism is a major cause of pharyngitis,
especially in children where the infection can progress to
rheumatic fever. It can cause necrotizing fasciitis in which
severe damage is done to muscle tissue. The disease can
progress into a toxic shock-like syndrome leading to death.

Necrotizing fasciitis usually begins at an inconspicuous site of


entry, such as a small vesicle. The host is prone to this infection
due to some type of immunocompromised state.
is a Gram-positive cocci that is catalase-negative. On blood
agar, it is alpha hemolytic and the colonies (typically) are
perfectly round with an indentation in the center of the colony.
The colonies are bile soluble and susceptible to optochin (Pdisc). The organism is a major cause of pneumonia, meningitis,
otitis media, wound infections, and septicemia.Streptococcus
pneumoniae
Question 77
1 out of 1 points

There is an outbreak of a respiratory disease in a hotel. One of the guests, a


45-year-old man, develops a dry cough and complains of difficulty breathing.
He has a fever and weakness. He sees a local doctor. His AP chest X-ray
shows bilateral pulmonary infiltrates. Tests are done, and he is told that he
has pneumonia. The tests are positive for Legionella. He is diagnosed with
Legionnaires' disease. What is the responsible organism?
Answers:
Aerobic Gram negative rod
Gram negative cocci
Gram positive rod
Gram positive cocci
Anaerobic Gram negative rod

Response Legionella is an aerobic Gram negative rod. Water distribution


Feedback systems of large buildings is considered to be a source of
:
infection. Risk factors for the development
of Legionella pneumonia include cigarette smoking, chronic lung
disease, and immunosuppression. In addition, it is seen more
frequently in males than females. It tends to affect those middle
aged or older more than young people. There is no association
with blood type and infections.

References:
Andreoli, Tomas E., M.D., Carpenter, Charles, C.J., M.D., Griggs,
Robert C., M.D., Benjamin, Ivor J., M.D. Andreoli and Carpenter's
Cecil Essentials of Medicine. 7th edition. Philadelphia, PA. 2007.
Legionella: Drinking Water Health Advisory. United States
Enviornmental Protection Agency. March 2001.
Question 78
1 out of 1 points

Case:
A 15-year-old boy was admitted to a healthcare facility presenting with a 2day history of nausea, vomiting, diarrhea, headache, and a continued fever
with 39.5C (103.1F) temperature. The patient was found to have rosecolored spots on his abdomen. The physician suspected it to be a case of
Salmonella infection based on the symptoms. Microscopic examination of
the stool, subjected to Gram staining, revealed Gram negative bacilli. The
organism did not show any motility and produced mauve colonies on
CHROMagar Salmonella (CAS) media and black transparent colonies on
Hektoen enteric agar (HEA), Salmonella-shigella (SS) agar, and xyloselysine-desoxycholate agar (XLD). On XLD agar, yellow colored zones
surrounded the colonies.
Question:
What is the causative organism in this patient?
Answers:

Salmonella enteritidis
Salmonella gallinarum
Salmonella paratyphi
Salmonella typhi
Salmonella typhimurium

Respons
e
Feedbac
k:

The patient has been infected by Salmonella


typhi. Salmonella species are a group of non-spore forming,
facultative anaerobic, Gram negative bacilli that is predominantly
motile found in the intestinal tracts of humans and other warm

blooded animals. They are responsible for the outbreak of food


poisoning instances. S. typhi infection causes typhoid with
symptoms like abdominal pain, headache, loss of appetite, and in
some cases appearance of rose-colored spots across the
abdomen. Diagnostic test for all the species
of Salmonella involve microscopic examination of stool for the
detection of organisms, and later biochemical tests are used to
identify the species. All species of Salmonella produce mauve
colonies on CAS medium.S. typhi and paratyphi produce black
colored colonies on XLD, SS, and HEA media as a result of
hydrogen sulfide (H2S) production, but only S.
typhi produces yellow colored halos around its colonies on XLD
media due to fermentation of the sugar xylose present in it.
The best way to prevent Salmonella infections is by properly
cooking food, good personal hygiene, and heating food to
appropriate temperatures or prompt refrigeration.
Salmonella enteritidis is transmitted to humans from infected
hens via raw or undercooked eggs and causes gastroenteritis.
These organisms contaminate the eggs before the shells are
formed in the infected hens, resulting in transmission of the
organisms to humans. Infection by S. enteritidis results in
diarrhea and fever. These organisms too form mauve colonies on
CAS, but serological tests confirm the infection by S. enteritidis.
Preventive measures include eating well-boiled eggs and
avoiding consumption of cracked eggs.
Salmonella gallinarum is found to infect only chickens, turkeys,
and other animals. This organism causes fowl typhoid, resulting
in mortality or drop in egg production by the birds. These
organisms have not been found to infect humans.
Salmonella paratyphi infection results in symptoms similar to
typhoid. The infected person develops rose-colored spots on the
chest. The organism is transmitted by direct or indirect contact
with feces of the infected person or by contaminated food and
water. Differential test includes growing the organism on CAS and
XLD media. S. paratyphi do not produce yellow halos around
colonies on XLD media. Preventive measures involve hygienic
practices when handling food.
Salmonella typhimurium are generally found to infect mice, but
can be transmitted to humans from contaminated foods of
animal origin and cause salmonellosis. Symptoms include
diarrhea, fever, headache, abdominal pain, vomiting. In rare
cases, blood is seen in stools. Symptoms are evident 12-72 hours
after ingestion of the organism and may last for 4-7 days.

Diagnosis involves microscopic examination of stool and


serological tests. Preventive measures include proper handling of
food.

References:
Susan Maddocks, Tom Olma, and Sharon Chen, Comparison of
CHROMagar Salmonella Medium and Xylose-LysineDesoxycholate and Salmonella-Shigella Agars for Isolation
of Salmonella Strains from Stool Samples. J Clin Microbiol, 2002
Aug, 40(8):2999-3003.
Typhoid fever. Centers for Disease Control and Prevention,
Division of Bacterial and Mycotic Diseases, available at
http://www.cdc.gov/ncidod/dbmd/diseaseinfo/typhoidfever_g.htm
Last accessed February 16, 2007.
Salmonella enteritidis. Centers for Disease Control and
Prevention, Division of Bacterial and Mycotic Diseases.available
at, http://www.cdc.gov/ncidod/dbmd/diseaseinfo/salment_g.htm.
Last accessed Feb 16, 2007
Water-related diseases, Typhoid and paratyphoid enteric fevers,
World Health Organization, available at
http://www.who.int/water_sanitation_health/diseases/typhoid/en/.
Last accessed February 21, 2007.
Poppe C, Smart N, Khakhria R et al., Salmonella typhimurium
DT104: a virulent and drug-resistant pathogen. Can Vet J. 1998
Sep, 39(9):559-65.
Source for the Table: Typhoid fever, Centers for Disease Control
and Prevention, Division of Bacterial and Mycotic Diseases,
available at
http://www.cdc.gov/ncidod/dbmd/diseaseinfo/typhoidfever_g.htm.
Last accessed February 16, 2007.
Renuka K, Sood S, Das BK, Kapil A. High-level ciprofloxacin
resistance in Salmonella enterica serotype Typhi in India. J Med
Microbiol. 2005 Oct;54(Pt 10):999-1000.
Question 79
1 out of 1 points

Case:
A 25-year-old G1P1 woman, who recently delivered a healthy newborn,
presents with right-sided breast pain and tenderness and associated malaise
and fever. The patient is currently breastfeeding. Physical inspection of the
right breast reveals breast erythema and tenderness. The patient has no
known comorbidities and denies significant family medical problems. No
medical complications were incurred during or after the pregnancy. The
patient denies palpitations, shortness of breath, and syncope; she has been
compliant with all of her postnatal obstetrical appointments. The patient's
vitals are as follows: pulse 78, respirations 16, blood pressure 102/68 mm
Hg, and temperature 101.2F.
Question:
What organism most likely accounts for the patient's clinical symptoms?
Answers:

Staphylococcus epidermidis
Streptococci
Escherichia coli
Enterococci
Staphylococcus aureus

Respons
e
Feedbac
k:

Mastitis is a condition that reflects cellulitis of the periglandular


tissue in breastfeeding mothers, typically due to infection
with Staphylococcus aureus. Clinical symptoms of mastitis
include breast pain, redness, high fever, and flu-like symptoms.
Physical findings include focal breast erythema, swelling, and
tenderness; fluctuance suggests a breast abscess. Diagnosis
rests on breast milk cultures and complete blood count analysis.
If fluctuance is present on physical examination, then a breast
ultrasound can be evaluated with the use of a focus ultrasound.
Treatment for mastitis involves continued breastfeeding and
treatment with the antibiotics dicloxacillin or erythromycin. The
treatment of a breast abscess is incision and drainage.
Although Staphylococcus aureus is the most common organism
responsible, Staphylococcus epidermidis and streptococci are
occasionally isolated.
Escherichia coli and enterococci are much less commonly
implicated in mastitis.

References:
Dixon JM. ABC of breast diseases. Breast infection. BMJ. Oct
8 1994;309(6959):946-9.
Barbosa-Cesnik C, Schwartz K, Foxman B. Lactation
mastitis. JAMA. Apr 2 2003;289(13):1609-12.
Bland & Copeland. The Breast: Comprehensive Management of
Benign and Malignant Disorders. 3rd. Saunders; 2004.
Mass S. Breast pain: engorgement, nipple pain and mastitis. Clin
Obstet Gynecol. Sep 2004;47(3):676-82.
Question 80
1 out of 1 points

Antibodies are secreted protein products of B cells. The antibody molecule


has two separate functions: the first is to bind to the pathogen that elicited
the immune response, and the second is to recruit various cells and
molecules to destroy the pathogen once the antibody is bound to it. What is
the ability of an antibody to distinguish its immunogen from other antigens?
Answers:

Amount
Isotype
Specificity
Affinity
Heavy chain

Response Antibodies are large molecules composed of heavy chains and


Feedback light chains. The most important characteristics of an antibody
:
response are the specificity, amount, isotype or class, and
affinity of the antibodies produced. The specificity determines
the ability of the antibody to distinguish the immunogen from
other antigens. The amount of antibody is a function of the
number of responding B cells, their rate of synthesis, and the
persistence of the antibody after production.
The isotypiccomposition of an antibody determines the

biological functions these antibodies can perform and the sites


in which the antibody can be found. The strength of binding of
the antibody to its antigen is called its affinity. All of these
parameters of the humoral response help to determine the
capacity of that response to protect the host from infection.
Question 81
1 out of 1 points

There exist multiple levels of control of the immune response. The effects of
helper T cells are balanced by those of functional suppressor T cells. One of
the key mechanisms for controlling the lymphocyte response to antigen is
the induction of
Answers:

Anergy
Apoptosis
Latency
Tolerance
Molecular mimicry

Response Tolerance is a state in which no specific antibody is made to an


Feedback antigen, while normal immune responsiveness to other antigens
:
remains. T cells from a tolerant animal will suppress the immune
response to the antigen if the cells are adoptively transferred to
another animal; the cells responsible for this transfer
are suppressor T cells. Tolerance can be induced any time in the
ontogeny of the cell, provided it expresses a receptor for the
antigen.
When soluble antigen binds an immature B cell, the cell is
inactivated but not killed. This state of non-reactivity is
called anergy, and such B cells are defined as anergic. Anergy is
a state of unstable metabolic arrest that can lead to apoptosis.
Apoptosis represents physiological as opposed to pathological
(necrotic) cell death - it is the cellular equivalent of suicide.
Apoptotic cells are recognized by phagocytes and removed.
Some viruses can enter a state known as latency in which the

virus is not being replicated. In the latent state, the virus does
not cause disease, but because there are no viral peptides to
signal its presence, the virus will not be eliminated. Latent
infections can be reactivated.
If a microbial antigen is very similar to normal host antigens, the
immune response to this antigen may be weak or absent. The
mimicking of host antigens by microbial antigens is referred to
as molecular mimicry. There is evidence that antibodies formed
against microorganisms sometimes cross-react with host tissues
and cause disease.
Question 82
1 out of 1 points

Coagulase enzyme is produced by


Answers:

Staphylococcus epidermidis
Staphylococcus saprophyticus
Streptococcus pyogenes
Staphylococcus aureus
Streptococcus pneumoniae

Response Staphylococcus aureus produces enzyme coagulase that


Feedback converts fibrinogen to fibrin. This organism possesses bound
:
and free forms of coagulase. Coagulase bound to the
staphylococcal cell wall can directly convert fibrinogen to
insoluble fibrin and cause the organism to clump together.
However, cell free coagulase causes the staphylococci to clump
by reacting with a globulin plasma factor to form
staphylothrombin, which catalyzes the conversion of fibrinogen
to insoluble fibrin.
Question 83
1 out of 1 points

A 4-year-old girl, adequately immunized for her age, has a history of


repeated pyogenic infections. She had normal antibody responses following
childhood immunizations. A defect in what cells best explains the cause of
her infections?
Answers:

B-lymphocytes
T lymphocytes
Eosinophils
Macrophages
Neutrophils
Basophils

Respons
e
Feedbac
k:

Pyogenic infections are generally caused by bacteria. The entry


of bacteria into the body triggers a number of responses:
activation of the complement pathway and phagocytosis
by neutrophils and macrophages. Macrophages play a major role
in chronic granulomatous infections,
and neutrophils predominate in acute pyogenic infections.
An adequate response to childhood immunizations indicates an
adequate capability of antibody production, and recovery from
viral infections like measles indicates appropriate cell-mediated
immunity. Therefore, the defect must lie in the neutrophils.

Question 84
1 out of 1 points

The binding of peptide is an important step in the assembly of stable major


histocompatibility complex (MHC) class I molecules. When the supply of
peptides into the endoplasmic reticulum (ER) is disrupted, newly synthesized
MHC class I molecules are held in the ER in a partially folded state. The
folding of an MHC class I molecule depends on its association first with 2microglobulin and then with peptide, and this process involves a number of
accessory proteins with a chaperone-like function. Only when peptide is
bound is the MHC class I molecule released from the ER and allowed to
reach the surface of the cell. In humans, the newly synthesized MHC class I
chains bind to what chaperone protein?

Answers:

Caspase
Vascular addressin
Granzyme
Calnexin
Selectin

Response The lectin calnexin is an integral membrane protein of about 90


Feedback kDa in the ER. Many resident ER proteins act as molecular
:
chaperones and participate in the proper folding of polypeptides
and their assembly into multisubunit proteins. Studies indicate
that calnexin associates with the MHC class I heavy chains,
partial complexes of the T cell receptor and B cell membrane
immunoglobulin, but not with completed receptor complexes.
Calnexin is a chaperone that retains incompletely or improperly
folded proteins in the ER.
Apoptosis, programmed cell death, is triggered by a variety of
stimuli. The activation of specific proteases
called caspases (cysteine-aspartic-acid-proteases) is an early
event during apoptosis, which can be detected even before DNA
fragmentation. The caspases convey the apoptotic signal in a
proteolytic cascade, with caspases cleaving and activating other
caspases that then degrade other cellular targets that lead to
cell death.
Many CAMs involved in immune responses have been identified.
They include selectins and integrins, which bind to mucin-like
molecules called vascular addressins, glycoproteins that are
expressed on the surface of vascular endothelial cells.
Killing by cytotoxic T cells occurs by two pathways. One involves
the action of cytotoxic substances contained in granules inside
the T cell. The major constituents of the granules involved in
target-cell killing are perforin and granzymes. Granzymes pass
into the target cell through the pores created by polyporphyrin
molecules and interact with intracellular components of the
target cell to induce apoptosis.
are cell-surface molecules with a common core structure,
distinguished from each other by the presence of different
lectin-like domains in their extracellular portion. They are
important for leukocyte homing to specific tissues, and can be

expressed either on leukocytes or on vascular


endothelium.Selectins
Question 85
1 out of 1 points

In the oncology unit of a tertiary care hospital, an outbreak of nosocomial


infections was observed. Blood stream and urinary tract infections were
predominant. The causative agent of the outbreak was a multiple drug
resistant (MDR), catalase-negative, Gram-positive coccus that is known to be
a major etiological agent of nosocomial infections. As part of infection
control measures, surveillance cultures were carried out to detect patients
colonized by this nosocomial pathogen. What specimen is ideal for this
purpose?
Answers:

Nasal swab
Swab from axilla
Blood
Urine

Rectal swab

Respons
e
Feedback
:

The major MDR Gram-positive cocci associated with outbreaks


of nosocomial infections are Methicillin Resistant Staphylococcus
Aureus (MRSA) and Vancomycin Resistant Enterococcus (VRE).
Of these, catalase-negative Gram-positive coccus indicates VRE.
VRE colonization primarily occurs in the lower gastro-intestinal
tract and frequently precedes infection. Therefore of the listed
specimens, rectal swab is the ideal specimen to be collected for
detecting patients colonized by this pathogen.
Urine and blood samples are tested when there is clinical
suspicion of infection.
MRSA colonizes in various body sites. Frequent colonization
occurs in anterior nares and skin, and swabs from these sites
are most commonly used for surveillance screening. Swabs from
multiple sites including rectum have been used for MRSA

surveillance, especially in neonates.


Prevalence of VRE has rapidly increased throughout the health
care systems in various countries, including the U.S. Reservoirs
of transmission mainly consist of colonized and infected
patients. The hospital environment may also be an important
reservoir. Most common mechanism of patient-to-patient
transmission is by the transient carriage of the organisms on the
hands of the health care workers.
Predisposing factors for colonization/infection are prolonged
hospital stay, severe underlying disease, immunosuppression,
abdominal or cardiothoracic surgery, and therapy with multiple
antibiotics and/or vancomycin.
Recommended measures to control transmission include
surveillance cultures for rapid identification of colonized and
infected patients, contact precautions, hand hygiene, prudent
use of antibiotics, and decontamination of the environment and
equipments.
Vancomycin resistance is much more common in Enterococcus
fecium than in E. fecalis. Higher rates of resistance to penicillin
and ampicillin are also found among E. faecium.
Enterococci show intrinsic chromosome-mediated resistance to
cephalosporins, semisynthetic penicillins (oxacillin, nafcillin),
clindamycin and Trimethoprim/Sulfa, and low level resistance to
aminoglycosides.
Acquired resistance can develop to a variety of agents, including
vancomycin, and is mediated by genes encoded on plasmids or
transposons.
Several VRE phenotypes have been identified. Of these, VanA
and VanB are the types commonly encountered in human
infections. VanA phenotype shows high level resistance to the
glycopeptides, vancomycin, and teicoplanin. VanB shows
moderate-to-high level resistance to vancomycin and
susceptibility to teicoplanin.
The VanA resistance determinant has been well studied. It is
identified as a cluster of genes including vanA gene packed
within a transposon that is carried on a plasmid, which can be
transferred via conjugation. This horizontal transfer is found to
play a major role in the nosocomial spread of VanA VRE.
Vancomycin resistance in VanA strains is based on the
manufacture of the depsipeptide D-alanyl-D- lactate (mediated
by a ligase) and its incorporation into the peptidoglycan layer of

the cell wall in place of the natural vancomycin target D-alanylD-alanine.


In addition to the phenotypic studies, characterization of the
strains by pulsed-field-gel-electrophoresis (PFGE) to detect
clonal patterns and determination of the van genotype by
polymerase chain reaction (PCR) assays are found useful in
epidemiological studies of nosocomial infections by VRE.
References:
Jawetz, Melnick & Adelberg's Medical Microbiology -23rd Ed
2004; Copyright McGraw-Hill Co:
Mandell,Douglas & Bennett's Principles and Practice of Infectious
diseases.6th Ed; 2005:2411-2417.
BMC Infect Dis-JUN-2007; 7:52.
Ann Acad Med Singapore- -2007; 36:379-383.
Infect Cont Hosp Epid-OCT-2006; 27(10): 1081-1087.
Indian J Pediatr-Mar-2004; 71(3): 229-239.
Mayo Clin Proc-APR-2006; 81(4): 529-536.
J Clin Microbiol-NOV-2006; 44(11):4234-4236.
Question 86
1 out of 1 points

What genetic event mediates the increased speed and efficiency of the
spread of specific plasmids among bacteria?
Answers:

Recombination
Transformation
Transduction
Hfr transfer
Conjugation

Response Conjugation is one of the major reasons for the increase in the

Feedback number of bacteria exhibiting multiple antibiotic resistance.


:
Conjugation is the direct transfer of bacterial plasmid DNA from
a donor cell to a recipient bacterium cell and is the most
efficient manner of horizontal gene spread. The prototypical
conjugative plasmid is the F-plasmid or F-factor. There can be
only 1 copy of the F-plasmid in a given bacterium and bacteria
that possess a copy are called F-positive and as such function as
donor cells. Cells that lack F-plasmids are called F-minus and
function as recipient cells. An important pre-requisite for
conjugative transfer is the intimate association between cell
surfaces of the interacting donor and recipient cells. Eventually,
all of the cells become donors after conjugation.

Recombination is incorrect. Recombination is the process of DNA


sequence recognition and insertion not the direct transfer of
bacterial plasmid DNA from one cell to another.

Transformation is incorrect. Transformation involves the uptake


of DNA into a host cell and its integration into the genome.
Bacterial transformation is referred to as a stable genetic
change brought about by the uptake of "naked" DNA (DNA
without associated protein or cells) to increase the quantity of
DNA in the cell.

Transduction is incorrect. Transduction involves the injection of


foreign DNA by a bacteriophage virus into the host bacterium.

Hfr transfer is incorrect. Hfr refers to high frequency of


recombination where the F-plasmid is incorporated into the
chromosome and during conjugation a portion of the
chromosome is taken into a recipient cell by the F-plasmid.

References:
Grohmann E., M. Gunther, M. Espinosa. Conjugative plasmid
transfer in gram + bacteria. Microbiol. Mol. Biol. Rev. Jun 2003,
67: 277-301.
Question 87

1 out of 1 points

Case:
A 38-year-old man presents with nausea, vomiting, diarrhea, and abdominal
cramping for the past 2 hours. The patient reveals that he had consumed
meat that had been left uncovered for a night. Laboratory examination
revealed that the meat was contaminated with circular, clustered organisms,
which were found on Gram staining to be Gram-positive. Fairly large, yellow
colored colonies were formed when the organism was grown on mannitol
agar media in a temperature range of 15 to 45C. The organism showed
positive results for coagulase enzyme activity and showed resistance to
penicillin and methicillin antibiotics.
Question:
What organism was the causative agent of the food poisoning?
Answers:

Clostridium perfringens
Enterococcus faecalis
Escherichia coli
Staphylococcus aureus
Salmonella species

Respons
e
Feedbac
k:

The causative agent of food poisoning in this case


is Staphylococcus aureus. S. aureus is a Gram-positive, aerobic,
circular (cocci) organism which appears to be in pairs or in
grape-like clusters on microscopic examination. S. aureus is
found as normal human body flora, colonized in the nasal
passages. The organism is pathogenic pertaining to its ability to
produce and release toxins. It is resistant to high
salt concentrations, especially sodium chloride (NaCl), and grows
on mannitol salt agar forming large, yellow colored colonies. S.
aureus grows well in a temperature range of 15 to 45C. The
majority of the strains of S. aureus show positive results for
coagulase test, but it is not a differential diagnostic test to detect
the organism, as few strains of S. epidermidis also show positive
results for the test. S. aureus is resistant to penicillin and
methicillin. Individuals with S. aureus food poisoning have loss of
fluid due to diarrhea; therefore, they are advised to drink plenty

of water and electrolyte solutions. The preventive measures


include proper washing of hands and maintaining good hygiene
when preparing or handling foods.
Clostridium perfringens is a Gram-positive, anaerobic, spore
forming bacilli (rod shaped). C. perfringens is commonly found as
a soil microorganism. It is also found in the feces of infected
persons and thus becomes a source of infection. C.
perfringens food poisoning is due to the ingestion of the
organism and is associated with intense abdominal cramps and
diarrhea. The diagnostic tests include stool examination to
detect the toxin or the organism in the infected person's feces.
The organism can grow in a wide range of temperatures, and it
are cultured in anaerobic liquid media in laboratories. There is no
specific treatment for C. perfringens food poisoning, but the
preventive measures include cooking food at high temperatures,
rapid cooling of cooked foods, and hygienic handling of foods.
Enterococcus faecalis, previously known as Streptococcus
faecalis, is a Gram-positive, circular, facultative anaerobe. It is a
normal inhabitant of the intestinal tract and female genital tract.
It may cause endocarditis, urinary tract infections,
bacteremia, wound infections, catheter-related infections, and
intra-abdominal/pelvic infections.
Escherichia coli are a common flora of the human body and are
generally found in the gut of humans. E. coli are Gram-negative,
non-spore forming bacilli. Most strains of E. coli are nonpathogenic, but strains called enterohaemorrhagic E. coli(EHEC)
are responsible for foodborne illness due to the toxins they
release. The symptoms include abdominal cramps and diarrhea,
which may lead to hemorrhagic colitis. The organism grows in a
temperature range of 7 to 50C, but the optimum temperature at
which the organism grows best is 37C. Diagnosis of the infection
is generally by microscopic examination of the stool from the
infected person. Preventive measures include practicing hygiene
when preparing food and using boiled and cooled water for
consumption.
Salmonella species are non-spore forming, Gram-negative bacilli.
Salmonella are found in the intestinal tract of humans and other
warm and cold-blooded animals. They are responsible for
causing acute gastroenteritis as a result of foodborne ingestion
or intoxication. They are transmitted to the humans via
contaminated water and food. Salmonella typhi and Salmonella
paratyphi are responsible for typhoid and typhoid-like fever in
humans. Measures to prevent infection include hygienic

practices when handling food.

References:
References:
Kenneth Todar, Staphylococcus, Todar's Online Textbook of
Bacteriology, http://textbookofbacteriology.net/staph.html. Last
updated 2006.
Foodborne Pathogenic Microorganisms and Natural Toxins
Handbook, U.S. Food and Drug Administration,available at
http://www.cfsan.fda.gov/~mow/chap3.html. Last accessed Feb
15, 2007.
Weinstein MP, Mirrett S, Kannangara S et al., Multicenter
evaluation of use of penicillin and ampicillin as surrogates for in
vitro testing of susceptibility of enterococci to imipenem. J Clin
Microbiol, 2004 Aug, 42(8):3747-51.
du Toit M, Franz CM, Dicks LM, and Holzapfel WH. Preliminary
characterization of bacteriocins produced by Enterococcus
faecium and Enterococcus faecalis isolated from pig faeces. J
Appl Microbiol, 2000 Mar, 88(3):482-94.
World Health Organization, Enterohaemorrhagic Escherichia coli
(EHEC), http://www.who.int/mediacentre/factsheets/fs125/en/.
Last updated May 2005.
Vidal R, Vidal M, Lagos R, Levine M, and Prado V, Multiplex PCR
for diagnosis of enteric infections associated with diarrheagenic
Escherichia coli. J Clin Microbiol, 2004 Apr, 42(4):1787-9.
Stuhlmeier R, Stuhlmeier KM. Fast, simultaneous, and sensitive
detection of staphylococci. J Clin Pathol. 2003 Oct;56(10):782-5.
Le Loir Y, Baron F, Gautier M. Staphylococcus aureus and food
poisoning. Genet Mol Res. 2003 Mar 31;2(1):63-76.
Question 88
0 out of 1 points

Case:
A 50-year-old man presents to the emergency room with clinical signs of
toxicity. He has an elevated temperature of 40C. He has difficulty answering
questions and maintaining attention. He complained of a sore left calf

muscle, which had a vesicle on it that was raised and swollen. Previous to
the appearance of the vesicle he had some localized pruritus that he used
hydrocortisone cream to treat. Blood was drawn for laboratory tests and the
results were significant for an elevated prothrombin time; elevated white
blood cell count with a left shift and a decreased platelet count; an elevated
creatine kinase (CPK); and an elevated creatinine. The patient was taken to
the operating room where gross purulence was found to be tracking up
toward the thigh area from the vesicle on the calf. Debridement was initiated
and ischemic dead muscle was noted that was interspersed with viable
muscle tissue. Tissue was sent to the microbiology laboratory for a stat
Gram stain and culture. The Gram stain was positive for gram-positive cocci
in chains. The culture after 24 hours was positive for beta hemolytic colonies
that were Gram-positive cocci in chains; catalase-negative; and a Lancefield
grouping of A. Refer to the image.
Question:
The patient has a diagnosis of necrotizing fasciitis due to what agent?

Answers:

Streptococcus agalactiae
Clostridium perfringens
Bacillus cereus
Streptococcus intermedius group (millerii)
Streptococcus pyogenes

Staphylococcus aureus

Respons
e
Feedback
:

Streptococcus intermedius group (millerii) is a Gram-positive


cocci that is catalase negative, esculin positive, hippurate
negative, Voges-Proskauer positive, and produces a caramel-like
odor. Colonies often require CO2 and when they do grow they
are sometimes mistaken for anaerobic streptococci. Colonies
can be beta, alpha, or gamma hemolytic on blood agar media.
Lancefield antigenic serologic reactions are of little value in
identifying these organisms because any one of a number of
antigens may be exhibited. The organism has been documented
as causing oral infections, bacteremia and endocarditis, thoracic
infections, central nervous system infections, and abdominal
infections. In drug addicts the organism can cause a
subcutaneous abscess/cellulitis that can be mistaken as
necrotizing fasciitis.
Streptococcus agalactiae is a Gram-positive cocci that is
catalase negative, beta hemolytic on blood agar media, and has
a Lancefield grouping of B. The organism produces CAMP factor,
a thermostabile extracellular protein that results in synergistic
hemolysin on sheep blood agar with the B-lysin
of Staphylococcus aureus (seen in 98-100% of group B
streptococcal isolates). The organism can cause a variety of
infections such as neonatal meningitis, pneumonia,
osteomyelitis, endocarditis, and skin/soft tissue infections.
Skin/soft tissue infections are usually found in patients with
predisposing factors such as diabetes mellitus. Necrotizing
fasciitis can occur due to Streptococcus agalactiae. Ampicillin
and penicillin G are the drugs of choice with vancomycin as an
alternative for penicillin allergic adults.
Clostridium perfringens is a Gram-positive anaerobic rod that
produces spores, have a boxcar appearance on Gram stain,
produce lecithinase, and produce a double zone of hemolysis on
blood agar. The organism can cause a variety of infections such
as bacteremia, pleuropulmonary infections, biliary tract
infections, intra-abdominal infections, food poisoning, clostridial
myonecrosis (gas gangrene); including various other soft tissue
infections (crepitant cellulitis, suppurative myositis, and
common soft tissue infections in which they are often a
component of polymicrobial flora).
Bacillus cereus is an aerobic Gram-positive rod that forms
spores; it is beta hemolytic on blood agar. The organism can

cause a variety of infections, such as bacteremia, endocarditis,


food poisoning, ophthalmitis, osteomyelitis, soft tissue
infections, and necrotizing fasciitis. Necrotizing fasciitis has
been reported in a leukemic patient and a patient with sickle cell
disease. As in all cases of necrotizing fasciitis, antibiotic therapy
alone is not sufficient in treating the infection and multiple
surgical debridement is usually required. Amputation is
sometimes necessary.
Streptococcus pyogenesis a Gram-positive cocci that is catalase
negative, beta hemolytic on blood agar, and had a Lancefield
grouping of A. The organism can cause an infection called
necrotizing fasciitis (streptococcal gangrene). The infection
involves the deeper tissues and fascia. Extensive and rapid
spreading of necrosis, as well as, gangrene of the skin and
underlying structures is characteristic. The infection usually
begins at a site of trivial or inapparent trauma. Prompt antibiotic
treatment and surgical measures to contain the infection
(amputation/debridement) is necessary to prevent life
threatening complications, such as toxic shock. Mortality rates
are generally high.
is a Gram-positive cocci that is catalase positive and coagulase
positive. On Gram stain it morphologically appears in clusters.
The organism is usually beta hemolytic on 5% sheep blood agar
and the colonies have a yellowish color. Staphylococcus
aureus can cause a variety of infections. It can cause toxic shock
syndrome (TSS), which can be broken down to menstrual TSS
and nonmenstrual TSS. TSS was first described in 1978 in
children and later in 1980 and 1981 when the introduction of a
new hyperabsorbable tampon produced an epidemic
of Staphylococcus aureus TSS. Product changes have since then
reduced the incidence. Patients with menses-associated TSS are
usually young women between the ages of 15-25 years of age
that are using tampons during their menstrual period.
Symptoms include severe myalgias, fever, vomiting, and
diarrhea. The patient usually presents as listless and confused
and may rapidly develop severe hypotension with hypovolemic
shock. Conjunctival inflammation and a "sunburn rash" can
develop within a few hours. Fluid replacement and IV oxacillin or
nafcillin are used to treat the patient.Staphylococcus aureus
Question 89
1 out of 1 points

A 23-year-old male admitted to the hospital due to multiple injuries acquired


in a motorcycle accident is referred to you because of the development of a
progressing infection to the leg after 2 days hospitalization. Physical
examination of the leg reveals a necrotic wound infection involving the deep
tissues. There is substantial redness and purulent tracking up the thigh area
of the leg. Surgical debridement was advised, and the patient was
immediately taken to surgery. Surgical debridement of the wound was
extensive and involved muscle tissue. Muscle tissue was sent to the
laboratory for a stat Gram stain and culture. The Gram stain report was
called to the floor and was significant for the presence of gram-positive rods
and large numbers of white blood cells. Necrotizing fasciitis was the
diagnoses based on the clinical findings and Gram stain result. Twenty-four
hours later the cultures were positive for an aerobic gram-positive rod that
was beta hemolytic on 5% sheep blood agar. Refer to the image. What is the
probable cause of this necrotizing fasciitis?

Answers:

Streptococcus agalactiae
Clostridium perfringens
Bacillus cereus
Streptococcus intermedius group (millerii)
Streptococcus pyogenes
Staphylococcus aureus

Respons
e
Feedback
:

Streptococcus intermedius group (millerii) is a gram-positive


cocci that is catalase negative, esculin positive, hippurate
negative, Voges-Proskauer positive, and produces a caramel-like
odor. Colonies often require CO2 and when they do grow they
are sometimes mistaken for anaerobic streptococci. Colonies
can be beta, alpha, or gamma hemolytic on blood agar media.
Lancefield antigenic serologic reactions are of little value in
identifying these organisms because any one of a number of
antigens may be exhibited. The organism has been documented
as causing oral infections, bacteremia and endocarditis, thoracic
infections, central nervous system infections, and abdominal
infections. In drug addicts the organism can cause a
subcutaneous abscess/cellulitis that can be mistaken as
necrotizing fasciitis.
Streptococcus agalactiae is a gram-positive cocci that is
catalase negative, beta hemolytic on blood agar media, and has
a Lancefield grouping of B. The organism produces CAMP factor,
a thermostabile extracellular protein that results in synergistic
hemolysin on sheep blood agar with the B-lysin
of Staphylococcus aureus (seen in 98-100% of group B
streptococcal isolates). The organism can cause a variety of
infections such as neonatal meningitis, pneumonia,
osteomyelitis, endocarditis, and skin/soft tissue infections.
Skin/soft tissue infections are usually found in patients with
predisposing factors such as diabetes mellitus. Necrotizing
fasciitis can occur due to Streptococcus agalactiae. Ampicillin
and penicillin G are the drugs of choice with vancomycin as an
alternative for penicillin allergic adults.
Clostridium perfringens is a gram-positive anaerobic rod that
produces spores, have a boxcar appearance on Gram stain,
produce lecithinase, and produce a double zone of hemolysis on
blood agar. The organism can cause a variety of infections such
as bacteremia, pleuropulmonary infections, biliary tract
infections, intra-abdominal infections, food poisoning, clostridial
myonecrosis (gas gangrene); including various other soft tissue
infections (crepitant cellulitis, suppurative myositis, and
common soft tissue infections in which they are often a
component of polymicrobial flora).
Bacillus cereus is an aerobic gram-positive rod that forms
spores, and is beta hemolytic on blood agar. The organism can
cause a variety of infections such as bacteremia, endocarditis,
food poisoning, ophthalmitis, osteomyelitis, soft tissue

infections, and necrotizing fasciitis. Necrotizing fasciitis has


been reported in a leukemic patient and a patient with sickle cell
disease. As in all cases of necrotizing fasciitis, antibiotic therapy
alone is not sufficient in treating the infection and multiple
surgical debridement is usually required. Amputation is
sometimes necessary.
Streptococcus pyogenes is a gram-positive cocci that is catalase
negative, beta hemolytic on blood agar, and had a Lancefield
grouping of A. The organism can cause an infection called
necrotizing fasciitis (streptococcal gangrene). The infection
involves the deeper tissues and fascia. Extensive and rapid
spreading of necrosis, as well as, gangrene of the skin and
underlying structures is characteristic. The infection usually
begins at a site of trivial or inapparent trauma. Prompt antibiotic
treatment and surgical measures to contain the infection
(amputation/debridement) is necessary to prevent life
threatening complications, such as toxic shock. Mortality rates
are generally high.
is a gram-positive cocci that is catalase positive and coagulase
positive. On Gram stain it morphologically appears in clusters.
The organism is usually beta hemolytic on 5% sheep blood agar
and the colonies have a yellowish color. Staphylococcus
aureus can cause a variety of infections. It can cause toxic shock
syndrome (TSS), which can be broken down to menstrual TSS
and nonmenstrual TSS. TSS was first described in 1978 in
children and later in 1980 and 1981 when the introduction of a
new hyperabsorbable tampon produced an epidemic
of Staphylococcus aureus TSS. Product changes have since then
reduced the incidence. Patients with menses-associated TSS are
usually young women between the ages of 15-25 years of age
that are using tampons during their menstrual period.
Symptoms include severe myalgias, fever, vomiting, and
diarrhea. The patient usually presents as listless and confused
and may rapidly develop severe hypotension with hypovolemic
shock. Conjunctival inflammation and a "sunburn rash" can
develop within a few hours. Fluid replacement and IV oxacillin or
nafcillin are used to treat the patient.Staphylococcus aureus
Question 90
1 out of 1 points

Case:
An 18-year-old man reports to your office complaining of a painful nodule
located at the nape of his neck. He reports periods of fever and malaise for
the past 3 days. An examination reveals a large nodule that is hard on
palpation and is surrounded by smaller pus-draining red papules. The patient
has a fever of 38.9C and an elevated WBC of 12,000/mm3 with a 50% band
on the differential. Material is obtained by aspiration of the nodule that is
thick and creamy. A Gram stain of the material is positive (refer to the
image). The culture grew out in 18 hours 4+ beta hemolytic colonies on
aerobic blood agar media that were catalase positive and coagulase
positive. The patient had been placed on antibiotics with instructions to treat
the area with warm compresses periodically. A follow-up visit for the
following week was scheduled.
Question:
What is the most likely diagnosis?

Answers:

Impetigo
Folliculitis
Carbuncle

Furuncle
Ecthyma

Respons
e
Feedbac
k:

Folliculitis is a pyoderma that is located within the hair follicles


and apocrine regions. It can be caused by Staphylococcus
aureus, Pseudomonas aeruginosa,
Enterobacteriaceae, and Candida species. Staphylococcus
aureus is the most common cause of folliculitis. Lesions are
small (2-5 mm) and erythematous, with a central pustule
topping the papules. Folliculitis due to Pseudomonas
aeruginosa is usually acquired from swimming pools or
whirlpools that are contaminated and are not adequately
chlorinated. Pseudomonas aeruginosa produces pruritic
papulourticarial lesions, which appear within 48 hours of
exposure and go on to pustule formation. The sites of
predilection are the hips, buttocks, and axillae with a sparing of
the soles and palms. When contracted in a whirlpool, the
folliculitis is limited to the trunk below the upper chest or neck,
depending on the water level.
Impetigo
is a vesicular superficial infection of the skin that later becomes
crusted. The predominant etiological agents that cause impetigo
are Streptococcus pyogenes and Staphylococcus aureus.
Impetigo infections are most common during the summer when
it is hot and humid. The infection is highly communicable. The
infection is spread among family members and preschool
children, especially in conditions of overcrowding and poor
hygiene. Exposed areas are the most common sites of lesions.
Pruritus is common, and scratching of the lesions can cause
infections to spread. If material from a vesicle is obtained for
Gram stain, gram-positive cocci are usually seen. Initial vesicular
lesions may appear like early varicella, tinea
circunata, andHerpes simplex infections.
Carbuncle
is a large and deep inflammatory nodule that is extensive and
extends into the subcutaneous fat of areas covered by thick,
inelastic skin. It is usually located at the nape of the neck, on
the back, or in the thighs. The patient presents with fever and
malaise. Occasionally the patient will present acutely ill.
Drainage occurs externally along the course of multiple hair
follicles. In cases where the carbuncle produces a large amount

of undrained pus, cellulitis, or bacteremia, there is a leukocytosis


present. Osteomyelitis, endocarditis, or other metastic foci can
occur due to unpredictable blood stream invasion. Blood stream
invasion can also be promoted by the manipulation of the
lesions. Staphylococcus aureus is the most common cause.
Furuncle
is a deep inflammatory nodule that usually develops due to a
preceding folliculitis. It is similar to a carbuncle, except that it is
not as extensive. As with the carbuncle, Staphylococcus
aureus is the most common cause. Furuncles appear as red
nodules that are firm yet tender and become painful as the
infection progresses. Furuncles will be found in areas of the body
where there is friction, perspiration, and hair follicles. These
areas include the neck, axillae, face, and buttocks. Predisposing
factors include diabetes, obesity, neutrophil function defects,
and treatment with corticosteroids.
Ecthyma
is similar to impetigo in that the lesions that appear are small
vesicles on exposed areas of the body. The difference with
ecthyma is that the lesions will penetrate through the
epidermis. Streptococcus pyogenes is the causative agent. The
lesions are most commonly found on the lower extremities
(especially in the elderly and children). The lesions appear as
"punched-out" ulcers that are covered with greenish-yellow
crusts that extend deep in the dermis. They are surrounded by
raised violaceous margins. Streptococcus pyogenes can be seen
on Gram stains of lesions and is an aerobic gram-positive cocci
that will occur in short to long chains. It is beta-hemolytic on
blood agar, catalase negative, and has a Lancefield grouping of
"A".
Question 91
1 out of 1 points

A 25-year-old white male presents to the emergency room after a work


related accident 10 hours ago. A soiled, discolored wound on his left thigh is
hurting badly. There is massive edema around it and you feel crepitation.
What does the microorganism causing your suspected diagnosis look like?
Answers: Gram positive rods with peritrichous flagella, oval spores

Gram positive, thin rods with peritrichous flagella, round spores


Gram positive, big rods, middle or ends distended, oval spores
Gram negative, plump, coccoid rods with peritrichous flagella
Gram negative, very small, motionless rods

Response The patient is presenting with gas gangrene. Gram positive, big
Feedback rods, middle or ends distended, oval spores are clostridium
:
perfringens, which can be found in soil. When they get in
contact with wounds they produce toxins. The latter cause
muscle necrosis, edema, and produce gas. Gas gangrene has a
lethality of about 50%.
Gram-positive rods with peritrichous flagella, oval
spores are Clostridium botulinum and cause botulism.
Gram-positive thin rods with peritrichous flagella, round
spores are clostridium tetani. They produce toxins, which cause
tetanus, an infectious disease usually contracted by wounds.
Gram negative, plump coccoid rods with peritrichous
flagella, are Escherichia coli, which can cause a whole variety of
infections, for example UTI, peritonitis, appendicitis, sinusitis,
otitis, tropical diarrhea, and meningitis in infants and toddlers.
Gram negative, very small, motionless rods are Brucellaceae,
which can cause a variety of infections, like influenza, ulcus
molle, endocarditis, meningitis, sepsis, and conjunctivitis.
Question 92
0 out of 1 points

A stool specimen from a patient with gastroenteritis grows pink colonies on a


McConkey medium. This pathogen is also found to efficiently cleave a
disaccharide into glucose and galactose. What is the causative agent of the
patient's disease?
Answer The pathogen is E. coli grown under the conditions of low lactose
s:
and low glucose
The pathogen is E. coli grown under the conditions of high lactose

and low glucose


The pathogen is Salmonella grown under the conditions of high
lactose and low glucose
The pathogen is Shigella grown under the conditions of low lactose
and high glucose
The pathogen is Shigella grown under the conditions of no glucose
and low lactose

Respons The correct response is that the pathogen is E.coli grown under
e
the conditions of high lactose and low glucose.
Feedback
E. coli is a lactose fermenter. When placed on a McConkey
:
medium, it will cleave lactose and produce acid; this turns the
medium pink. Glucose and galactose are byproducts of lactose.
For lactose to be broken down, the lac operon has to be turned
on; this requires a high lactose concentration to inhibit the lacoperon inhibitor, and low glucose levels to allow efficient
initiation of lac operon transcription.
High glucose would inhibit the lac operon.
Salmonella and Shigella will produce white colonies on
McConkey medium.

References:
Goering Richard, Hazel Dockrell, Mark Zuckerman, Derek
Wakelin, Ivan Roitt, Cedric Mims, Peter Chiodini. Mim's Medical
Microbiology, 4th edition, Elsevier Limited. 2008.
Winn WC, Koneman EW, Allen SD, et al. Koneman's Color Atlas
and Textbook of Diagnostic Microbiology. Lippincott Williams &
Wilkins, 2006:1451.
Narang A, Pilyugin SS. Bistability of the lac operon during growth
of Escherichia coli on lactose and lactose+glucose. Bull Math
Biol. 2008 May;70(4):1032-64. Epub 2008 Feb 2.
Pommerville JC, Alcamo IE. Alcamo's Fundamentals of
Microbiology. Jones & Bartlett Publishers, 2004:212.
Murray RK, Granner DK, Mayes PA, et al. Harper's illustrated

biochemistry. McGraw-Hill Professional, 2003:378.


Question 93
1 out of 1 points

A 28-year-old man comes to the primary care doctor with a lesion on his left
arm that is getting worse. It has been there for 3 days. He also says that he
has muscle aches, fatigue, malaise, but he thinks that it is because he was
on a camping trip last week and all that he ate was a rabbit and some
chocolate bars. On physical exam, there is an ulcer with transparent
exudates and an axillary lymphadenopathy. The rest of the exam is normal.
What bacteria is the most likely cause of his infection?
Answers:

Bordetella pertussis
Borrelia Burgdorferi
Brucella
Francisella tularensis
Rickettsia rickettsii

Response
Francisella tularensis is a Gram negative rod that causes
n;;;Feedbac tularemia (rabbit fever). It is transmitted to humans by ticks
k:
(A. americanum, D. variabilis, and D. Anderson) or flies but
also by eating uncooked infected meat, contaminated water,
or through skin lesions when skinning rabbits that are
infected. The ulceroglandular manifestation is most common.
Symptoms such as flu appear 3 to 5 days after the inoculation
and depending on the source of infection the patient can also
show skin ulcers, pneumonia, pericarditis, and regional
lymphadenopathy. The diagnosis is done by serology, but also
a PCR of the ulcer is available. Although a vaccine has been
around for 50 years, its use has not yet been approved. The
treatment is streptomycin and gentamicin.
Borrelia burgdorferi is a spirochete transmitted to humans by
a deer tick (Ixodes scapularis). It causes Lyme disease. There
are 3 stages of this disease:
Stage 1 (early localized): usually the first 10 days after the
bite. It is a localized erythematous macule or papule, pale in

the center (erythema migrans), present at the site of the bite,


accompanied by flu-like symptoms.
Stage 2 (early disseminated): A few weeks later. Usually
neurological symptoms such as facial nerve palsy or
meningitis, and cardiac symptoms such as arrhythmias or
pericarditis are seen.
Stage 3 (late chronic disease): Develops even years later with
symptoms like arthritis, fickettsia and cardiomegaly.
The diagnosis is mainly clinical. The treatment depends on the
severity and the stage of the disease. Doxycycline is used in
early stages.
Rickettsia fickettsia is a rickettsia that causes Rocky Mountain
spotted fever. It is transmitted to humans by the wood tick
(Dermacentor andersoni) and the dog tick (Dermacentor
variabilis). Typical symptoms include flu appearing 5 to 7 days
after the inoculation and a rash (pink macules) that begins in
palms and soles, disseminating to the rest of the body and
disappearing with pressure. Later on ulceration and
ecchymosis can be seen. The neurologic and circulatory
systems can be also involved. Treatment is done with
tetracycline, doxycycline, and chloramphenicol.
Bordetella pertussis is a gram-negative coccobacillus. The
main symptoms are paroxysmal cough and vomiting. It is
transmitted by direct contact with respiratory discharge of
infected persons.
Brucella is a group of bacteria found in animals such as dogs,
goats, sheep, cattle, deer, or pigs. Humans become infected
(Brucellosis) when they come in contact with the animals or
their products (milk). Brucellosis can produce abscesses in
many different organs such as liver and spleen. The treatment
depends on the type of presentation.
References:
1. Bratton RL, Corey R. Tick-borne disease. Am Fam Physician.
2005 Jun 15;71(12):2323-30. Review. PMID: 15999870
[PubMed indexed for MEDLINE]
2. ARNE TRNVIK and MAY C. CHU New Approaches to
Diagnosis and Therapy of Tularemia Ann. N.Y. Acad. Sci., Jun
2007; 1105: 378 404.
3. J. WAYNE CONLAN and PETRA C.F. OYSTON Vaccines
Against Francisella Tularensis Ann. N.Y. Acad. Sci., Jun 2007;
1105: 325 350

4. Farlow J,. Wagner DM, Dukerich M et al Francisella tularensis


in the United States. CDC website, available at
http://www.cdc.gov/ncidod/eid/vol11no12/05-0728.htm, last
accessed Jan 6. 2006
Bvb
b
842/
+2415444444444444444412222222222
/[q ]124+
/[2

Question 94 k[;LO1 out of 1 points

Verotoxin
Answers:

Is an endotoxin produced by E.coli (O157:H7 serotype)


Is an exotoxin produced by E.coli (O157:H7 serotype)
Is produced by Streptococcus viridans
Is produced by Clostridium tetani

Response Verotoxin is an exotoxin produced by strains of E.coli with the


Feedback: O157:H7 serotype. These strains cause bloody diarrhea and are
the cause of outbreaks associated with eating undercooked
hamburgers. The toxin is named for its toxic effect on Vero
(monkey) cells in culture.
Question 95
1 out of 1 points

A 30-year-old woman and her 10-year-old son present with a vesicular


outbreak on the skin (arms, legs, and face). There is no history of insect
bites/infestation, exposure to chemicals, or use of medications. Vital signs
are normal for both patients. The vesicles are described as itchy and red.
The appearance of the condition seemed to coincide with a heat wave that's
been impacting the area for the past 4 days. Vesicle material was obtained
for Gram stain and culture. The Gram stain was positive for the presence of
Gram-positive cocci in clusters and moderate white blood cells (refer to the

image). Antibiotics were prescribed along with instructions to take care of


the infected skin areas. The culture was significant for 4+ Gram-positive
aerobic cocci that were beta-hemolytic on blood agar media, catalasepositive, and coagulase-positive. What is the diagnosis?

Answers:
Impetigo
Folliculitis
Carbuncle
Furuncle
Ecthyma

Respons
e
Feedbac
k:

Impetigo is a vesicular superficial infection of the skin that later


becomes crusted. The predominant etiological agents that cause
impetigo are Streptococcus pyogenes and Staphylococcus
aureus. Impetigo infections are most common during the
summer when it is hot and humid. The infection is highly

communicable. The infection is spread among family members


and preschool children, especially in conditions of overcrowding
and poor hygiene. Exposed areas are the most common sites of
lesions. Pruritus is common, and scratching of the lesions can
cause infections to spread. If material from a vesicle is obtained
for Gram stain, Gram-positive cocci are usually seen. Initial
vesicular lesions may appear like early varicella, tinea
circinata,and Herpes simplex infections.
Folliculitis is a pyoderma that is located within the hair follicles
and apocrine regions. It can be caused by Staphylococcus
aureus, Pseudomonas aeruginosa,
Enterobacteriaceae, and Candida species. Staphylococcus
aureus is the most common cause of folliculitis. Lesions are
small (2-5 mm) and erythematous, with a central pustule
topping the papules. Folliculitis due to Pseudomonas
aeruginosa is usually acquired from swimming pools or
whirlpools that are contaminated and are not adequately
chlorinated. Pseudomonas aeruginosa produces pruritic papulourticarial lesions, which appear within 48 hours of exposure and
go on to pustule formation. The sites of predilection are the hips,
buttocks, and axillae, with a sparing of the soles and palms.
When contracted in a whirlpool, the folliculitis is limited to the
trunk below the upper chest or neck, depending on the water
level.Carbuncle is a large and deep inflammatory nodule that is
extensive and extends into the subcutaneous fat of areas
covered by thick, inelastic skin. It is usually located at the nape
of the neck, on the back, or in the thighs. The patient presents
with fever and malaise. Occasionally, the patient will present
acutely ill. Drainage occurs externally along the course of
multiple hair follicles. In cases where the carbuncle produces a
large amount of undrained pus, cellulitis, or bacteremia, there is
a leukocytosis present. Osteomyelitis, endocarditis, or other
metastatic foci can occur due to unpredictable blood stream
invasion. Blood stream invasion can also be promoted by the
manipulation of the lesions. Staphylococcus aureus is the most
common cause.
Furuncle is a deep inflammatory nodule that usually develops
due to a preceding folliculitis. It is similar to a carbuncle, except
that it is not as extensive. As with the carbuncle, Staphylococcus
aureus is the most common cause. Furuncles appear as red
nodules that are firm yet tender and become painful as the
infection progresses. Furuncles will be found in areas of the body
where there is friction, perspiration, and hair follicles. These
areas include the neck, axillae, face, and buttocks. Predisposing
factors include diabetes, obesity, neutrophil function defects,

and treatment with corticosteroids.


is similar to impetigo in that the lesions that appear are small
vesicles on exposed areas of the body. The difference with
ecthyma is that the lesions will penetrate through the
epidermis. Streptococcus pyogenes is the causative agent. The
lesions are most commonly found on the lower extremities
(especially in the elderly and children). The lesions appear as
"punched-out" ulcers that are covered with greenish-yellow
crusts that extend deep in the dermis. They are surrounded by
raised violaceous margins. Streptococcus pyogenes can be seen
on Gram stains of lesions, and it is an aerobic Gram-positive
cocci that will occur in short to long chains. It is beta-hemolytic
on blood agar, catalase-negative, and has a Lancefield grouping
of "A".Ecthyma
Question 96
1 out of 1 points

In his research on antibody diversity, Karl Landsteiner studied the immune


response to small molecular weight organic compounds. They are not
immunogenic by themselves, but antibodies can be raised against them
when they are bound covalently to a protein carrier. Once the tiny molecule
is made into a conjugate, it becomes an antigenic determinant capable of
interaction with an appropriate antibody binding site expressed by a B cell.
What are these organic compounds called?
Answers:

Proteasomes
Lewis antigens
Haptens
Epitopes
Idiotypes

Response Haptens are very small compounds such as a benzene ring, a


Feedback glucose molecule, or a phosphatidylcholine molecule. These
:
small molecules by themselves cannot induce an immune
response. If they are covalently attached to an immunogenic
substance such as a protein, they can be recognized by a B

cell's antigen receptor or antibody. The antigen can then be


internalized, processed and presented to a helper T cell, and
specific antibody can be produced against the haptens by B
cells. Landsteiner's work with haptens defined physical size as
one of the characteristics that determine whether or not a
substance will be able to induce an immune response.
Proteins in cells are constantly being degraded and replaced
with newly synthesized proteins. A major part in cytosolic
protein degradation is played by a large, multicatalytic protease
complex called the proteasome.
Lewis antigens are part of a blood group system separate from
the primary ABO system. Lewis antigens are glycoproteins and
are present in secretions as well as on the red blood cells.
Many of the antigens that provoke an immune response are
proteins. Antibody binds to specific regions of the protein
molecule known as epitopes. Epitopes are usually composed of
amino acids from different parts of the sequence that have been
brought together by protein folding.
Since individual antibodies differ in their variable regions, one
can raise antibodies against unique sequence variants, which
are called idiotypes. These sequence variants are phenotypic
markers.
Question 97
1 out of 1 points

The primary humoral immune response to an antigen usually consists of


antibody molecules made by plasma cells derived from a relatively large
number of different precursor B cells. The secondary response derives from far
fewer, high-affinity precursor B cells, which have undergone significant
expansion. During the secondary response and subsequent responses, the cell
surface receptors and antibodies show extensive somatic hypermutation, and
the antibodies produced increase in
Answers:

Amount
Quantity
Affinity
Variability

Recombination

Re B cells that respond to antigen increase in frequency after the primary


sp exposure to antigen and produce antibody of higher average affinity than
on unprimed B cells. Refer to the table.
se
Fe
Characteristic
ed
ba
Isotype of Antibody Produced
ck
:
Affinity of Antibody
Somatic Hypermutation

These antibodies are produced by memory B cells that have already


switched from IgM to more mature isotypes and express IgG, IgA, or IgE
on their surface, as well as a higher level of MHC class II molecules than is
characteristic of nave B cells. Increased antibody affinity for antigen and
increased levels of MHC class II expression facilitate antigen uptake and
presentation, and allow memory B cells to initiate their critical interactions
with armed helper T cells at lower doses of antigen.

Question 98
1 out of 1 points

Case:
A 12-year-old boy was seen in the pediatric clinic for 2-day history of severe
sore throat, pain and difficulty in swallowing, and high fever. On examination
his temperature was 39.5C and his tonsils were swollen and showed
yellowish spots of exudates. The anterior cervical lymph glands were
enlarged and tender. Throat swabs were collected and Rapid strep test was
done. The test was positive. Culture of throat swab on sheep blood agar
medium grew Gram-positive cocci in chains. The colonies were small with
large zones of beta hemolysis. The bacterium isolated was sensitive to
bacitracin. Presumptive identification was Streptococcus group A
(Streptococcus pyogenes).

Question:
Which of the Strep pyogenes antigens listed below is detected by the rapid
strep test?
Answers:

Hyaluronic acid capsule


Streptolysin O
Streptolysin S
M protein
Cell wall carbohydrate antigen

Respons Rapid strep test is used for diagnosing sore throat caused by
e
group A streptococcus. The test detects the cell wall
Feedbac carbohydrate antigen of the bacterium in the clinical sample.
k:
Serological grouping of hemolytic streptococci is based on the
nature of cell wall carbohydrate antigen (C substance). Rebecca
Lancefield introduced the grouping in 1933 and the serogroups
are also known as Lancefield's groups. Groups A-U (without I and
J) have been identified. Majority of human streptococcal
infections are caused by group A streptococcus (GAS).
The group specific C carbohydrate antigen is an integral part of
the cell wall. The cell wall is composed of an outer layer of
protein and lipoteichoic acid, a middle layer of group specific
carbohydrate, and an inner layer of peptidoglycan. The
carbohydrate antigen has to be extracted by chemical methods
to be detected using group specific antisera. Extraction of the
antigen can be done by treating centrifuged culture with hot
acids, by enzymatic lysis of streptococcal cells, or by autoclaving
cell suspensions. The conventional methods of capillary or agar
gel precipitation tests with group specific sera are used to
identify the extracted carbohydrate antigen. Isolates of hemolytic
streptococci belonging to other groups can also be identified by
these methods. Co-agglutination and latex agglutination methods
also have been used for grouping streptococci.
The serological specificity of the group-specific carbohydrate is
determined by an amino sugar. For example, group-specific
carbohydrate for group A streptococci is a polysaccharide chain
consisting of repeat units of rhamnose capped by N-acetyl
glucosamine molecules (rhamnose-N-acetyl glucosamine). The

group specific carbohydrate for group B streptococcus is


rhamnose-glucosamine polysaccharide and for group C,
rhamnose-N-acetyl galactosamine.
Rapid antigen detection test done directly with the throat swab
material also involves acid extraction of the group specific
carbohydrate antigen of GAS from the clinical sample and
identification of the antigen by immunological reaction. Different
technologies are used like latex agglutination, enzyme
immunoassay, optical immunoassay, and
immunochromatographic assay. All are reported to have good
specificity. False positive results are unusual and may be
produced by the presence of Streptococcus milleri group of
bacteria in the throat that express the group A carbohydrate
antigen. Sensitivity of the test may vary with the method used.
Though cultural isolation is the gold standard, positive rapid strep
test is generally accepted for diagnosis of pharyngitis/tonsillitis
caused by GAS. Early detection of GAS in throat swabs helps
early administration of antibiotics and early prevention of
suppurative complications and spread to others. When rapid test
is negative, back up cultures help confirm diagnosis. Culture is
required for performing susceptibility tests. Though no significant
change in susceptibility to penicillin has been observed,
erythromycin resistant strains and outbreaks due to such strains
have been reported.
Recently, rapid antigen detection tests based on molecular
methods (chemiluminiscent gene probes and polymerase chain
reaction) have been developed, which require special equipment.
Streptococcal sore throat is the most common infection caused
by hemolytic streptococcus. Virulent group A streptococcus
adheres to the pharyngeal epithelium by means of lipoteichoic
acid covering the surface pili and causes diffuse pharyngitis or
localized tonsillitis. The bacterium is responsible for a variety of
human infections ranging from milder diseases like pharyngitis
and impetigo to severe life-threatening infections such as
bacteremia, necrotizing fasciitis, and streptococcal toxic shock
syndrome.
Various structural components of Streptococcus pyogenes show
antigenic cross reaction with different tissues of the human body.
Antigenic relationships have been observed between Group A
carbohydrate and human cardiac valvular glycoproteins. The
reactivity is related to N-acetyl glucosamine moiety present in
both structures. Antigenic cross reactions with human tissues are
considered to be of importance in the pathogenesis of acute

rheumatic fever and acute glomerulonephritis, the nonsuppurative sequelae of GAS infection. In these conditions the
tissue damage produced is of immunological nature.
Hyaluronic acid capsule: Group A streptococci produces
hyaluronic acid capsule better noticeable in young cultures. It is
composed of equimolar concentrations of N-acetyl glucosamine
and glucuronic acid and is structurally identical to the hyaluronic
acid of mammalian tissues. When present, the capsule inhibits
phagocytosis. It is not antigenic in humans.
Streptolysin O: Streptolysins O and S are hemolysins and extra
cellular products of hemolytic streptococci. Streptolysin O is
oxygen labile. It is rapidly inactivated in presence of oxygen,
hence the name. It resembles the oxygen-labile hemolysins of
Clostridium perfringens and Cl.tetani and contributes to the
virulence of the bacterium. Following Strep pyogenes infection,
antibody is produced to streptolysin O. This antibody (antistreptolysin O) inhibits hemolysis by streptolysin O. In rheumatic
fever and acute glomerulonephritis, a retrospective diagnosis of
streptococcal infection is helpful. This is done by demonstrating
high levels of antibodies to streptococcal toxins. Quantitative
estimation of anti streptolysin O (ASO titer) is a standard
serological procedure for such retrospective diagnosis. An ASO
titer in excess of 160-200 units is considered significant and
suggestive of either recent or recurrent infections with
streptococci.
Streptolysin S is the cytolytic factor, which causes the beta
hemolytic zone surrounding the colonies on blood agar medium.
It is elaborated in presence of serum, hence the name. It is
oxygen stable. Streptolysin S is not immunogenic in humans and
neutralizing antibodies are not evoked during infection. Sera from
persons with past infection with streptococcus pyogenes do not
neutralize streptolysin S. Non-specific inhibitors present in human
sera may inhibit its activity. Streptolysin S is also considered as
an important virulence determinant of group A streptococcus.
M protein: This is a major virulence factor of Strep pyogenes and
is associated with resistance to phagocytosis and adherence to
host cells. It is one of the protein antigens identified in the outer
part of the cell wall. The other proteins are T and R. M protein is
antigenic. Based on the M protein, GAS can be sero- typed. There
are 2 major structural classes of M protein class I and class II.
Class I M protein is thought to be a virulence determinant for
rheumatic fever as the antigenic domains of this cross-react with
cardiac muscle.

A few M serotypes (5, 14, 18, and 24) have been identified with
outbreaks of acute rheumatic fever. A recent study of
streptococcal pharyngitis in children conducted in the US has
shown marked decrease in M-protein rheumatogenic types and
increase in non-rheumatogenic types of S.pyogenes isolates. It is
suggested that the near-disappearance of acute rheumatic fever
in the US could be due to the replacement of rheumatogenic
types of S.pyogenes by non-rheumatogenic types.
The association of M types with certain conditions is as follows:
1. Streptococcal toxic shock syndrome-M protein types 1 and 3,
which produce pyrogenic, exotoxins.
2. Strains belonging to M types 12, 2, 4, and 49 are known to be
nephritogenic.
3. Skin infections by strep pyogenes M types 49, 57, and 59-61;
strain characterization of GAS is mainly based on identification of
M protein and is of importance in epidemiological and
surveillance studies. To identify the M protein type, a genotyping
system (emm typing) has been developed. It is based on the
sequence of emm gene, which encodes for M protein. This
method gives better strain identification and could be used also
for identifying isolates non-typeable by M serotyping.
Most common emm types in the US are reported to be 1, 28, 12,
3, and 11.
References:
1. Jawetz, Melnick & Adelberg's medical microbiology 23rd Ed
2004; Copyright: Mc Graw-hill companies; pp 31,231-239.
2. Clin Microbiol Reviews-JULY-2004;17(3):571-580.Expert Rev Mol
Diagn-SEPT-2006;6(5):761-766.
3. .CDC;on strep throat: dated OCT 11,2005 available at
www.cdc.gov/ncidod/dbmd/diseaseinfo/groupastreptococcal_t.ht
m.
4. American Family Physician-FEB-2003;67(4):880,883-884.
5. J Med Microbiol-FEB-2006; 55 (Pt): 157-164.
6. Clin Infect Dis-AUG 1-2002; 35(3): 268-276.
7. Pediatriics-APR-2004;113(4):924-926.
8. Clinical Infectious Diseases-FEB 15-2006;42(4):441-447.
Question 99

1 out of 1 points

A 35-year-old male known IV drug abuser presents to the emergency room


with a necrotic infection of the right forearm. Examination of the right arm
revealed inflammation and erythema extending from the fingers to the
upper arm. An irregular area of necrosis is noted that is approximately
20mm long and 10mm in width. The necrosis appears to involve the fascia
and musculature. The patient has an elevated body temperature of 40C
with a respiratory rate of 40/min. A tissue specimen was obtained for culture
and Gram stain. The patient was treated with antibiotics and debridement
was necessary to contain the infection. The Gram stain was significant (refer
to the image) for gram-positive cocci in chains. At 24 hours the culture was
positive for 4+ gram-positive cocci that were alpha hemolytic; pinpoint in
size; catalase negative; esculin positive; and had a caramel-like odor. The
patient has a diagnosis of necrotizing fasciitis due to what agent?

Answers:

Streptococcus agalactiae
Clostridium perfringens
Bacillus cereus
Streptococcus intermedius group (millerii)
Streptococcus pyogenes

Staphylococcus aureus

Respons
e
Feedback
:

Streptococcus intermedius group (millerii) is a gram-positive


cocci that is catalase negative, esculin positive, hippurate
negative, Voges-Proskauer positive, and produces a caramel-like
odor. Colonies often require CO2 and when they do grow they
are sometimes mistaken for anaerobic streptococci. Colonies
can be beta, alpha, or gamma hemolytic on blood agar media.
Lancefield antigenic serologic reactions are of little value in
identifying these organisms because any one of a number of
antigens may be exhibited. The organism has been documented
as causing oral infections, bacteremia and endocarditis, thoracic
infections, central nervous system infections, and abdominal
infections. In drug addicts the organism can cause a
subcutaneous abscess/cellulitis that can be mistaken as
necrotizing fasciitis.
Streptococcus agalactiae is a gram-positive cocci that is
catalase negative, beta hemolytic on blood agar media, and has
a Lancefield grouping of B. The organism produces CAMP factor,
a thermostabile extracellular protein that results in synergistic
hemolysin on sheep blood agar with the B-lysin
of Staphylococcus aureus (seen in 98-100% of group B
streptococcal isolates). The organism can cause a variety of
infections such as neonatal meningitis, pneumonia,
osteomyelitis, endocarditis, and skin/soft tissue infections.
Skin/soft tissue infections are usually found in patients with
predisposing factors such as diabetes mellitus. Necrotizing
fasciitis can occur due to Streptococcus agalactiae. Ampicillin
and penicillin G are the drugs of choice with vancomycin as an
alternative for penicillin allergic adults.
Clostridium perfringens is a gram-positive anaerobic rod that
produces spores, have a boxcar appearance on Gram stain,
produce lecithinase, and produce a double zone of hemolysis on
blood agar. The organism can cause a variety of infections such
as bacteremia, pleuropulmonary infections, biliary tract
infections, intra-abdominal infections, food poisoning, clostridial
myonecrosis (gas gangrene); including various other soft tissue
infections (crepitant cellulitis, suppurative myositis, and
common soft tissue infections in which they are often a
component of polymicrobial flora).
Bacillus cereus is an aerobic gram-positive rod that forms
spores, and is beta hemolytic on blood agar. The organism can

cause a variety of infections such as bacteremia, endocarditis,


food poisoning, ophthalmitis, osteomyelitis, soft tissue
infections, and necrotizing fasciitis. Necrotizing fasciitis has
been reported in a leukemic patient and a patient with sickle cell
disease. As in all cases of necrotizing fasciitis, antibiotic therapy
alone is not sufficient in treating the infection and multiple
surgical debridement is usually required. Amputation is
sometimes necessary.
Streptococcus pyogenes is a gram-positive cocci that is catalase
negative, beta hemolytic on blood agar, and had a Lancefield
grouping of A. The organism can cause an infection called
necrotizing fasciitis (streptococcal gangrene). The infection
involves the deeper tissues and fascia. Extensive and rapid
spreading of necrosis, as well as, gangrene of the skin and
underlying structures is characteristic. The infection usually
begins at a site of trivial or inapparent trauma. Prompt antibiotic
treatment and surgical measures to contain the infection
(amputation/debridement) is necessary to prevent life
threatening complications, such as toxic shock. Mortality rates
are generally high.
is a gram-positive cocci that is catalase positive and coagulase
positive. On Gram stain it morphologically appears in clusters.
The organism is usually beta hemolytic on 5% sheep blood agar
and the colonies have a yellowish color. Staphylococcus
aureus can cause a variety of infections. It can cause toxic shock
syndrome (TSS), which can be broken down to menstrual TSS
and nonmenstrual TSS. TSS was first described in 1978 in
children and later in 1980 and 1981 when the introduction of a
new hyperabsorbable tampon produced an epidemic
of Staphylococcus aureus TSS. Product changes have since then
reduced the incidence. Patients with menses-associated TSS are
usually young women between the ages of 15-25 years of age
that are using tampons during their menstrual period.
Symptoms include severe myalgias, fever, vomiting, and
diarrhea. The patient usually presents as listless and confused
and may rapidly develop severe hypotension with hypovolemic
shock. Conjunctival inflammation and a "sunburn rash" can
develop within a few hours. Fluid replacement and IV oxacillin or
nafcillin are used to treat the patient.Staphylococcus aureus
Question 100
1 out of 1 points

A 55-year-old male presents to your office complaining of chronic cough,


fatigue, fever, and chills. His history is significant for smoking and alcohol
and abuse. His body temperature is 38.7C, and respiration rate is 25/min.
He describes left-sided pleuritic chest pain. There were crackles heard over
the left lower lung, and right lower lobe. A chest radiograph was ordered and
demonstrated left lower lobe infiltrates. A CBC was significant for an
elevated white cell count of 28,000 /ul with a marked left shift. Sputum was
produced by the patient that was green in color. The sputum was sent to the
laboratory for culture and Gram stain. The Gram stain had many WBC and
moderate numbers of gram-positive diplococci. The culture was significant
for 4+ gram-positive cocci that were alpha hemolytic. A plate was streaked
out with a suspension of the isolate and had an oxacillin disc and a P-disc
placed on the inoculated plate. The results were (refer to the image). These
results indicate that the organism causing the pulmonary infection in this
patient is what?

Answers:

Escherichia coli
Pseudomonas aeruginosa
Streptococcus pyogenes

Staphylococcus aureus
Streptococcus pneumoniae

Response Streptococcus pneumoniae is a gram-positive cocci that is


Feedback catalase negative. On blood agar it is alpha hemolytic and the
:
colonies typically are perfectly round with an indentation in the
center of the colony. The colonies are bile soluble and
susceptible to optochin (P-disc). The organism is a major cause
of pneumonia, meningitis, otitis media, wound infections, and
septicemia.
Staphylococcus aureus is a gram-positive cocci that appears as
clusters. The organism is catalase and coagulase positive. It is
the most common gram-positive cocci to cause clinical disease
in humans. The organism is a common cause of surgical wound
infections acquired in a hospital. It can also cause pneumonia,
sepsis, peritonitis, sore throats, and food poisoning.
Escherichia coli is a gram-negative bacillus and is a member of
the Enterobacteriaceae group. It is the number one significant
clinical isolate in the microbiology laboratory. The organism is
90% lactose positive and thus grows pink on MacConkey agar,
bile positive on MacConkey agar, and indole positive and
oxidase negative. The organism is found to cause a variety of
infections such as urinary tract infections, septicemia, and
wound infections.
Pseudomonas aeruginosa is a lactose negative bacillus (has no
pink color on MacConkey) that is indole negative and oxidase
positive. It is a nonfermenter and not a member of the
Enterobacteriaceae group. On MacConkey it many times has a
slight blue-green color and is said to have a grape-like odor. The
organism is a common cause of nosocomial infections and is
usually multiply resistant to a variety of antibiotics. It can cause
eye infections, urinary tract infections, wound infections,
septicemia, and ear infections.
is a gram-positive cocci that appears in pairs or chains under
the microscope. The organism is beta hemolytic on blood agar,
catalase negative, and is Lancefield grouped as A. The organism
is a major cause of pharyngitis, especially in children where the
infection can progress to rheumatic fever. It can cause
necrotizing fasciitis where severe damage is done to muscle
tissue. The disease can progress into a toxic shock-like
syndrome leading to death. Necrotizing fasciitis usually begins

at an inconspicuous site of entry such as a small vesicle. The


host is prone to this infection due to some type of
immunocompromised state.Streptococcus pyogenes
Thursday, March 31, 2016 2:43:14 PM EDT

Vous aimerez peut-être aussi